1. Trang chủ
  2. » Cao đẳng - Đại học

Sach Toan hoc Tap san Toan hoc

131 15 0

Đang tải... (xem toàn văn)

Tài liệu hạn chế xem trước, để xem đầy đủ mời bạn chọn Tải xuống

THÔNG TIN TÀI LIỆU

Thông tin cơ bản

Định dạng
Số trang 131
Dung lượng 1,41 MB

Nội dung

Chỉ có đúng p lớp mà mỗi lớp chứa đúng n phần tử giống hệt nhau (tương ứng với n cách tô toàn bộ vòng tròn bằng một màu duy nhất). Đây là một đẳng thức rất đơn giản nhưng nó lại có nhiều[r]

(1)

Tuyển tập số vấn đề chọn lọc

(2)(3)

Lời nói đầu

Cuốn sách nhỏ "Tuyển tập số tốn sơ cấp chọn lọc www.diendantoanhoc.net" q đặc biệt mà BTC kỳ thi VMEO II dành tặng cho bạn thành viên tham gia đoạt giải Đây quà mùa hè mà Nhóm Quản Lý muốn dành tặng cho tất bạn học sinh chun tốn nói riêng bạn u thích tốn sơ cấp nói chung Trong sách giới thiệu với bạn 250 tốn thuộc chủ đề lớn tốn phổ thơng bao gồm Số Học, Tổ Hợp, Hình Học, Giải Tích Đại Số Kèm theo đề toán khoảng 20 viết chuyên đề nhỏ xoay quanh toán Số Học, Tổ Hợp Trong viết cố gắng thể đầy đủ thảo luận bạn diễn đàn tốn Một số viết chưa post lên diễn đàn mà trao đổi riêng thành viên giới thiệu tài liệu Chúng tơi vui mừng biết rằng, trao đổi riêng phổ biến bạn thành viên Đây thực mong muốn lớn người điều hành diễn đàn Số Học Tổ Hợp chủ đề thú vị đẹp đẽ toán sơ cấp Tuy nhiên để viết tài liệu hai chủ đề điều không dễ Đối với Số Học lựa chọn nhiều chủ để nhỏ dựa khung toán có diễn đàn, kiến thức Số Học đưa vào viết nhỏ, bạn đọc qua viết tìm hiểu kỹ lý thuyết số sơ cấp sách chuyên khảo hơn, giới thiệu hai sách: An introduction to the theory of number G.H.Hardy & E.M.Wright Elementary theory of number Sierpinsky Bản điện tử hai sách giới thiệu diễn đàn Về Tổ Hợp, chủ trương lựa chọn chủ đề cách tương đối rời rạc, cho khơng nên khiến bạn phải tiếp thu kiến thức tổ hợp cách giáo khoa Đối với toán tổ hợp chúng tơi cho vẻ đẹp tốn có ý nghĩa cao tới việc nhận thức người Do chúng tơi cố gắng lựa chọn tốn tổ hợp đẹp đẽ để kích thích tính tìm tịi bạn đọc Hai sách sơ cấp tổ hợp không nên bỏ qua 102 combinatorial problem Titu Andrecscu & Zuming Feng Extrenal combinatorics Stasys Jukna

Tất nhiên chủ đề Hình Học, Giải Tích Đại Số thú vị, nội dung ấn phẩm diễn đàn Và ấn phẩm diễn đàn chủ yếu xây dựng dựa thảo luận bạn nên hi vọng thời gian tới cịn có nhiều chủ đề thú vị chất lượng ngày cao

Cuốn sách nhỏ đời dựa cộng tác nhiều bạn thành viên Đó bạn K09, TuanTS, lehoan, NDTPX, clmt, anhminh, neverstop, bk2004, chuyentoan, camum,

(4)

hungkhtn lovepearl_maytrang Bạn camum lựa chọn hầu hết tốn giải tích, mục tổ hợp lehoan tuyển chọn với cộng tác NDTPX, tốn hình học MrMATH soạn với giúp đỡ nhiệt tình bk2004, chuyentoan nhận nhiều ý kiến bạn neverstop Cuối toán số học lựa chọn K09 lehoan, sau TuanTS MrMATH có nhiều thảo luận để hồn thiện thảo Trong q trình tuyển chọn chúng tơi nhận có nhiều tốn sáng tạo bạn thành viên Trong thời gian tới mong điều phát huy

Cuốn sách soạn phần mềm PCTEX version 5.0, gói vntex giới thiệu bạn tamnd File cài đặt chương trình gói lệnh bạn dowload mạng khơng q khó khăn Nếu có thắc mắc việc sử dụng TEX bạn giải tham khảo sách tác giả Nguyễn Hữu Điển (sách cho Viện Tốn Học ấn hành), ngồi bạn tham gia diễn đàn TEX www.viettug.com trao đổi với thành viên có kinh nghiệm soạn thảo diễn đàn

Mặc dù cố gắng việc kiểm tra thảo, chúng tơi bỏ sót số lỗi Mọi ý kiến đóng góp nội dung lần hình thức xin gửi địa mail nqk_mrmath@yahoo.com Chúng xin chân thành cám ơn hứa cố gắng trong việc thiết kế ấn phẩm

Thay mặt Ban Biên Tập

a

MrMATH

www.diendantoanhoc.net

(5)

Cộng tác viên

Trong thời gian hoàn thành thảo, thực giới thiệu sách nhỏ khơng hồn tồn tất nhóm CTV làm Trên thực tế nhóm CTV hồn thiện hầu hết đề mục cho ba nội dung Hình Học, Giải Tích Đại Số Tuy nhiên việc giới thiệu đồng thời tất 5chủ đề có lẽ khơng phù hợp với mục đích Bản liệt kê không nêu lên hết CTV công việc họ, dù tra cứu đủ dùng cho bạn đọc.Trong ấn phẩm tiếp nối sách nhỏ này, công việc CTV giới thiệu đầy đủ chi tiết

a Trần Nam Dũng (namdung) GV ĐHKHTN ĐHQG TP Hồ Chí Minh: [1]

a Trần Quốc Hoàn (K09) SV K50 CA Đại Học Công Nghệ Hà Nội: [2], [3.6], [3.8]

a Trần Mạnh Tuấn (TuanTS) SV K9 CNTN ĐHKHTN ĐHQG Hà Nội: [2], [3.2], [3.3],[3.4] a Lê Hồng Quý (lehoan) HS lớp 12 chuyên toán ĐHSP Vinh: [6], [7.2], [7.3], [7.7]

a Trần Đức Anh (camum) SV năm hệ CLC ĐHSP Hà Nội: [10]

(6)(7)

Mục lục

I Một số chủ đề Số Học 9

1 Tổng hai bình phương 11

2 Các đề tốn số học chọn lọc 17

3 Một số chủ đề số học chọn lọc 23

3.1 Số bập bênh 23

3.2 Định lýF ermatnhỏ ứng dụng đẹp 26

3.3 Một số tính chất hàm tổng chữ số 27

3.4 Hai ứng dụng phương trìnhP ell 30

3.5 Định lý phần dư Trung Hoa 32

3.6 Biểu diễn số 34

3.7 Một dạng phương trìnhDiophante đặc biệt 37

3.8 Số nguyên phức 40

3.8.1 Các khái niệm mở đầu 40

3.8.2 Thuật toán Euclidvà ước chung lớn hai số nguyên phức 41

3.8.3 Số phức nguyên tố vấn đề phân tích số nguyên phức 43

3.8.4 Sử dụng số nguyên phức để giải số toán 44

3.9 Phương trìnhCarmichael 45

3.10 Một số toán khác 47

4 Tổng nghịch đảo 53 II Một số chủ đề Tổ Hợp 59 5 Bổ đề Sperner 61 5.1 Bao lồi 63

5.2 Bổ đề KKM 64

5.3 Chứng minh định lý điểm bất động Brower 64

6 Các đề toán tổ hợp chọn lọc 65 7 Một số chủ đề tổ hợp chọn lọc 71 7.1 Bài toán Rubik lục lăng 71

(8)

7.2.1 Bất biến 73

7.2.2 Nửa bất biến 75

7.3 Phương pháp phân nhóm 78

7.4 Vai trò số đặc biệt 81

7.5 Hai toán phủ hình vng 84

7.6 Câu hỏi mở tính chất chùm đường tròn 86

7.7 Định lí Konig-Hall 88

7.8 Định lýErdos -Skerezes 90

7.9 Một số tốn khác 92

8 Góc màu 95 8.1 Khái niệm góc màu 95

8.2 Mở rộng toán 6người 99

8.3 Phương pháp hàm đếm vài ứng dụng 103

8.4 Mở rộng đề thi IMO1992 105

III Một số tốn khác 109

9 Hình Học 111

10 Giải Tích 117

(9)

Phần I

Một số chủ đề Số Học

(10)(11)

Chương 1

Tổng hai bình phương

Trần Nam Dũng

Giới thiệu.Định lýF ermat Euler là viên ngọc tuyệt vời Tốn Học kỷ 17−18 Từ thời phổ thơng đọc chứng minh (của Lagrange) đây, ngây ngất trước vẻ đẹp Nhiều năm đọc lại viết của GS.T ikhomirov trên tạp chí Kvant, tơi lại tiếp tục bất ngờ với chứng minh kết cũ Quá thích thú với bài báo, tơi dịch Tiếng Việt nhiều lần truyền vẻ đẹp phép chứng minh thần diệu trong đến hệ học sinh Hôm nay, xin dành tặng bạn thành viên diễn đàn www.diendantoanhoc.net bản dịch này.

Các bạn để ý xem số nguyên tố 3,5,7,11,13,17,19 Các số 5,13 17 biểu diễn dạng tổng hai bình phương:

5 = 12 + 22 13 = 22+ 32 17 = 12+ 42.

Còn số lại3,7,11,19 khơng thể biểu diễn Có thể cách giải thích điều hay khơng? Có, ta có định lý sau đây:

Định lý F ermat Euler. Điều kiện cần đủ để số nguyên tố lẻ biểu diễn dạng tổng hai bình phương số dư phép chia số cho là1

Trong trường hợp ban đầu củapcó thể kiểm tra tính đắn định lý này5 = 4.1+1, 13 = 4.3 + 1, 17 = 4.4 + = 4.0 + 3, = 4.1 + 3, 11 = 4.2 + 19 = 4.4 +

Đôi chút lịch sử định lý. Ai người phát điều này, nào? Vào dịp Noel năm 1640 (trong thư đề ngày 25.12.1640) nhà toán học vĩ đại P ierre de F ermat

(1601-1665) thông báo cho M ersenne, bạn thân củaDescartesvà "liên lạc viên" nhà bác học đương thời "Mọi số nguyên tố có số dư phép chia cho4 1đều biểu diễn cách dạng tổng hai bình phương" Thời chưa có tạp chí tốn học, tin tức trao đổi qua thư kết thông thường thông báo mà không kèm theo chứng minh

(12)

Thực sau gần 20 năm sau thư đó, thư gửi cho Carcavi, gửi vào tháng năm 1659, F ermat tiết lộ ý tưởng phép chứng minh định lý Ơng viết ý tưởng phép chứng minh dùng phương pháp xuống thang, cho phép từ giả thiết định lý không với p = 4k+ 1, suy khơng với số nhỏ hơn, cuối ta đến số 5, mà rõ ràng mâu thuẫn

Những cách chứng minh Euler (1707-1783) tìm khoảng 1742-1747 Hơn nữa, để tỏ rõ vị trí củaF ermat, người mà ơng kính trọng, Eulerđã tìm phép chứng minh dựa theo ý tưởng F ermat Vì vậy, ta gọi định lý định lý F ermat Euler

Những kết tốn học thường có tính chất chung ta đến nhiều đường khác nhau, cơng chúng từ nhiều hướng, đường đem đến cho người khơng biết sợ khó khăn khối cảm tuyệt vời

Tôi muốn chứng tỏ điều ví dụ định lýF ermat Euler

Ta đến đỉnh cao, phát minh vào kỷ XVII ba đường khác Một chúng tìm vào kỷ XVIII, đường khác - kỷ XIX đường thứ ba - kỷ XX

1 Cách chứng minh của Lagrange. Cách chứng minh (có thay đổi đơi chút) trình bày hầu hết sách lý thuyết số Nó dựa bổ để W ilson

nói nếup số nguyên tố số (p−)! + chia hết cho p

Để không sâu vào chứng minh kết phụ ta tường minh ý tưởng phép chứng minh ví dụ số 13 Với số nằm 2và 11 (kể số này) ta tìm số mà tích chúng chia cho 13 dư Ta có:

(13−1)! = 12! = (2.7).(3.9).(4.10).(5.8).(6.11).12.

Rõ ràng cặp hai số dấu ngoặc đơn có tích chia13 dư Từ suy 12! chia cho 13 có số dư 12, nghĩa 12! + chia hết cho 13 Trường hợp tổng quát chứng minh tương tự

Từ bổ đềW ilsonta rút hệ nếup= 4n+ số nguyên tố ((2n)!)2+ chia hết cho p Thật vậy, (bổ đề W ilson) (4n)! + chia hết cho p, phép biến đổi ta thu được:

(4n)! + = 1.2.3 (2n).(2n+ 1) (4n) +

= 1.2 (2n).(p−2n).(p−2n+ 1) (p−1) + = (2n)!.(−1)2n.(2n)!≡((2n)!)2+ mod p.

(13)

13

Do với hai cặp số (m1, s1)và(m2, s2)số dư phép chiam1+N s1 vàm2+N s2 cho p giống nhau, nghĩa số a+N b a = m1 −m2 b = s1 −s2 chia hết cho p Nhưng a2 −N2b2 = (a+N b)(aN b) chia hết cho p ý N2 ≡ −1 mod p ta thu a2 +b2 chia hết p, nghĩa là a2+b2 = rp với r nguyên dương Mặt khác

a2+b2 <2p suy ra r= 1 và thế a2+b2 =p Định lý chứng minh.

2 Chứng minh của D.T sagir. Phép chứng minh nhà toán học đương đại D.T sagir

làm tơi hồn tồn bất ngờ, điều kỳ diệu mà kết thu tưởng chừng khơng từ Sau cách chứng minh

Ta xét phép biến đổi mà ba số nguyên dương (x, y, z)được đặt tương ứng với ba số (x0, y0, z0) theo quy tắc:

    

x0=x+ 2z, y0=z, z0=yxz nếu x < yz

x0= 2yx, y0=y, z0=xy+z nếu yzx≤2y

x0=x−2y, y0=xy+z, z0=y trong trường hợp lại.

Ta ký hiệu phép biến đổi B : B(x, y, z) = (x0, y0, z0) Rất dễ dàng chứng minh rằng phép biến đổiB giữ nguyên dạng củax2+4yz Ta chứng minh điều này, chẳng hạn cho trường hợp thứ cách xác định Ta có:

x02+ 4y0z0= (x+ 2z)2+ 4z(yzx) =x2+ 4xz+ 4z2+ 4yz−4xz−4z2 =x2 + 4yz.

Trong trường hợp lại việc kiểm tra đơn giản Có nghĩa sốpnào ta có đẳng thứcx2+4yz=pthì đẳng thức giữ ngun sau phép biến đổiB.

Ta kiểm chứng phép biến đổiB xoắn, có nghĩa áp dụng B hai lần quay trở lại vị trí ban đầu Ta lại làm điều cho công thức thứ trên, trường hợp lại chứng minh tương tự

Với x < yz x0 = 2z +x, y0 = z, z0 = yzx từ đó x0 > 2y0 và nghĩa là phải tính B(x0, y0, z0)theo cơng thức thứ ba Nghĩa là:

    

x00=x0−2y0=x+ 2z−2z =x

y00 =x0−y0+z0=x+ 2zz+yxz =y

z00=y0=z.

Bây ta giả sử p số nguyên tố có dạng 4n + Khi đó, thứ phương trình

x2 + 4yz = p có hai nghiệm (x = 1, y = n, z = 1) (x = 1, y = 1, z = n) Và thứ hai phương trình có hữu hạn nghiệm (nguyên dương) Nếu giả sử nghiệm phương trình khơng có nghiệm mà y = z (nếu có nghiệm

(14)

Nếu nhìn vào cơng thức xác định B ta dễ dàng nhận thấy điểm bất động B điểm mà x = y Nhưng x = y > phương trình x2 + 4yz = p khơng có nghiệm (vì p khơng chia hết cho y) Nghĩa có điểm bất động (1,1, n) Từ tất lý luận ta suy số nghiệm phương trình x2+ 4yz=p số lẻ có điểm bất động(1,1, n)cịn tất nghiệm khác chia thành cặp

Nhưng, ta lại có phép biến đổi nữa, ký hiệu J, J thay đổi chỗ y z nghĩa J(x, y, z) = (x, z, y) Phép biến đổi tất nhiên giữ nguyên dạng x2+ 4yz và cũng xoắn Ta thử xem, ba số nghiệm phương trình x2+ 4yz =p

được J giữ nguyên tức mà J(x, y, z) = (x, y, z)

Ta giả sử từ trước y 6= z Nhưng khơng thể có điểm bất động Tất nghiệm chia thành cặp Như số nghiệm chẵn Nhưng ta vừa khẳng định số nghiệm lẻ Mâu thuẫn Vậy phải tồn nghiệm phương trìnhx2+ 4yz=p

y=z, p tổng hai bình phương Định lý chứng minh

3 Cách chứng minh thứ ba.Cách chứng minh củaM inkowsky sửa đổi đôi chút mà nói đến bây giờ, làm ngạc nhiên gấp bội Đáng tiếc cách chứng minh không sơ cấp lắm, cụ thể ta cần elippse công thức tính diện tích

Tất kết M inkowsky mà tưởng chừng khơng có liên hệ với định lý F ermat Euler mà quan tâm

Định lý.Cho a, b, c số nguyên, a >0và acb2 = Khi phương trình

ax2+ 2bxy+cy2 =

có nghiệm nguyên

Chứng minh. Ta xét hệ tọa độ Descartes vng góc cho tích vơ hướng cơng thức:

((x, y),(x0, y0)) =axx0+byy0+czz0.

Tích vơ hướng cho ta khoảng cách từ gốc tọa độ đến điểm(x, y) là:

d((0,0),(x, y)) =p((x, x),(y, y)) =pax2+ 2bxy+cy2.

Ta tìm khoảng cách ngắn từ gốc tọa độ đến điểm khác lưới nguyên (m, n) (m, nlà số nguyên) Gọi khoảng cách làd∗ và đạt điểm(m, n∗), thế:

am∗2 + 2bmn∗+cn∗2 =d∗2.

Tập hợp tất điểm(x, y)của mặt phẳng thỏa mãn bất đẳng thức:

(15)

15

ellipse Từ cách xây dựng ta suy vị tự ellipsenày theo tỷ số 1/2 đưa ellipse"co" đến tâm nằm điểm nguyên (tịnh tiến) tất cácellipse

thu có cắt cắt theo điểm biên

Dễ thấy diện tích phần giao ellipse với tam giác có đỉnh (0,0),(1,0),(1,1) nửa diện tích tồn ellipse Mà diện tích (chỗ khơng sơ cấp nhất):

πd∗2

4 ·(acb

) = πd ∗2

4 .

Như diện tích phần mà ellipse chiếm tam giác bẳng πd ∗2

8 nửa diện tích tam giác, nghĩa là:

πd∗2

4 < =⇒d

∗2

< π.

Bởi d∗2 số nguyên dương, d∗ = Định lýM inkowsky chứng minh

Nhưng kết tuyệt vời có liên quan đến định lý F ermat Euler? Liên quan trực tiếp đấy! Ta biết từ bổ đề W ilsonrằng số b2+ chia hết chop, khơng?!

Bây áp dụng định lý M inkowsky cho số a = p c = b 2+ 1

a Ta thu

tồn số nguyênmn cho:

1 =am2+ 2bmn+cn2

=⇒a=a2m2+ 2abmn+ (b2+ 1)n2 = (am+bn)2+n2.

Như (nhớ lại a = p) ta có p = (am+bn)2 +n2 nghĩa p tổng hai bình phương Một lần nữa, định lý lại chứng minh

namdung

www.diendantoanhoc.net

Trần Nam Dũng Giảng Viên Đại Học KHTN ĐHQG TP Hồ Chí Minh

Phụ lục.Chúng xin dẫn cách chứng minh sơ cấp định lýM inkowsky Giả sửb≥0và chứng minh quy nạp theob Vớib= 0mệnh đề Giả sử mệnh đề với 0,1, , b−1, ta chứng minh với b Sử dụng phép đổi biến (x=XY, y=Y)

=⇒ax2+ 2bxy+cy2 =aX2+ (2ba)XY + (c+a−2b)Y2 =AX2+ 2BXY +CY2.

Trong A=a,B =ba C=c+a−2b Suy B2=AC+ 1 vàA >0,0≤Bb−1. Sử dụng giả thiết quy nạp ta suy mệnh đề với bvà định lý chứng minh

K09

www.diendantoanhoc.net

(16)(17)

Chương 2

Các đề toán số học chọn lọc

Bài tốn 2.1. Tìm tất số ngun dương nguyên tố với phần tử của dãy:

an= 2n+ 3n+ 6n−1 n ≥1.

Bài toán 2.2. Giải phương trình nghiệm nguyên dương x2−(a2+b2)·y4 = 1.

Bài toán 2.3. Cho k số tự nhiên 1≤ a1 ≤ a2 ≤ akn thỏa mãn [ai, aj]> n với mọi

1≤ijk Chứng minh rằng:

(i)

k

X

i=1

ai

<

2 (ii)

k

X

i=1

ai

<

5.

Bài tốn 2.4. Hãy tìm tất số nguyên dương n sao cho tồn hoán vị {a1, a2, , an}

của {1,2, , n} thoả mãn tính chất hai tập hợp sau đây: (i) {a1, a1a2, , a1a2 an}

(ii) {a1, a1+a2, , a1+a2+ +an}

lập thành hệ thặng dư đầy đủ modun n.

Bài tốn 2.5. Tìm số nguyên dươngk lớn để tồn tại 2k số nguyên dương đôi phân biệt a1, a2, , ak, b1, b2, , bk k tổng a1+b1, a2+b2, , ak+bk đôi khác nhỏ

hơn 2005

Bài toán 2.6. Giả sử p là số nguyên tố Chứng minh trong 2p−1 số nguyên bất kì tồn tại p số có tổng bội số củap Kết luận toán thay đổi bỏ đi giả thiết p nguyên tố.

Bài toán 2.7. Chứng minh số hợp số thuộc hai dạng sau vô hạn:

(i) 22n + (ii) 62n + 1.

Bài toán 2.8. Giả sử a, b, c là số nguyên dương nguyên tố cho đẳng thức

an =b2+c2 đúng với số nguyên n >1 nào Chứng minh rằng a có thể viết thành tổng của hai số phương.

(18)

Bài toán 2.9. Một số tự nhiên bập bênh đem nhân với ta số đó nhưng viết theo thứ tự ngược lại chữ số Chẳng hạn số 1089 là số bập bênh có chữ số vì 1089.9 = 9801 Vấn đề tìm tất số bập bênh có n chữ số. Hơn tính số tất số bập bênh có n chữ số.

Bài toán 2.10. Chứng minh với số tự nhiên n bất kỳ tồn hai số nguyên x, y

thoả mãn n|x2−34y2+

Bài tốn 2.11. Tìm tất số tự nhiên k sao cho tồn số thực dương ck thoã mãn:

S(kn)

S(n) ≥cknN

Bài tốn 2.12. Tìm tập giá trị của N để phương trình sau có nghiệm ngun dương:

x21+x22+ +xn2 =N(x1x2 xn−1).

Bài toán 2.13. Dãy số p1.p2, , pn, là dãy tất số nguyên tố Chứng minh tồn

tại ba số hạng liên tiếp dãy thoả mãn tính chất số chúng lớn hơn bình phương số số đó.

Bài tốn 2.14. Chứng minh tồn số tự nhiên n để số 2n+ 3n có đúng 23 ước số

nguyên tố.

Bài toán 2.15. Cho dãy tăng số tự nhiên {an} có tính chất tồn số M sao cho

an+1−an < M với mọi nN Chứng minh tập ước số nguyên tố dãy vơ

hạn.

Bài tốn 2.16. Xét M =n(n−1) (nk+ 1) với n ≥2k Chứng minh rằng M có ước số nguyên tố lớn hơn k.

Bài tốn 2.17. Giả sử plà số ngun tố có dạng 4k+ Chứng minh p−1 số tự nhiên liên tiếp khơng thể chia làm hai nhóm có tích thừa số nhóm nhau. Bài tốn 2.18. Tìm số ngun dương n nhỏ sau cho n2 −n+ 11 là tích bốn số nguyên tố (khơng cần phân biệt).

Bài tốn 2.19. Tìm tất ba số nguyên dương (x, y, z) với z bé cho tồn số nguyên dương a, b, c, d có tính chất:

    

xy =zb =cd, x > a > c

z =ab=cd x+y=a+b.

Bài toán 2.20. Cho số nguyên a1, a2, , an b1, b2, , bn trong đó ai ≥ ∀i = 1, n.

Chứng minh tồn vô hạn số nguyên (c1, c2, , cn) sao cho ta có tính chất sau:

b1c1+b2c2 + bncn|c a1 +c

a2

2 + +c

an

n .

(19)

19

Bài tốn 2.22. Tìm tất số nguyên dương n sao cho n3−1 là số phương. Bài tốn 2.23. Chứng minh với hai số nguyên dương s, a (s) không chia hết cho 3) luôn tồn số tự nhiên n thoả mãn S(ns) =a với S(x) là tổng chữ số của x.

Bài toán 2.24. Cho số nguyên dương n >1 Tìm số ngun dương nhỏ khơng có dạng

nanb

ncnd với số nguyên dương a, b, c, d nào đó.

Bài tốn 2.25. Cho số ngun khơng âm a và số nguyên dương d Chứng minh trong 73 số a, a+d, , a+ 72d có số mà biểu diễn thập phân có chữ số Bài toán 2.26. Chứng minh với số thực δ ∈[0,1] và với mọi ε >0 bất đẳng thức:

ϕ(n)

nδ

< ε đúng với số tự nhiên n nào đó.

Bài tốn 2.27. Tìm giá trị lớn biểu thức:

S=

a1 +

a2

+ +

an

.

Với giá trị tự nhiên của a1, a2, , an biết rằng S <1

Bài toán 2.28. Cho số nguyên tố p= 4k+ Chứng minh tồn vô số số tự nhiên n

sao cho số [n.p] là số phương.

Bài tốn 2.29. Tìm tất số ngun dương m n sao cho với số dương a thoả mãn am, an là số nguyên suy ra a cũng số nguyên.

Bài toán 2.30. Cho trước số nguyên dương N Hãy tìm số nguyên dương k lớn cho với số nguyên a, b, c, d tuỳ ý mà N2 ≤a < bc < dN2+k thì ad6=bc.

Bài tốn 2.31. Tìm nghiệm nguyên dương phương trình:

t2 = 4zyzxy.

Bài toán 2.32. Giả sử A là tập hợp N thặng dư mod N2 Chứng minh tồn tập hợp

B gồm N thặng dư mod N2 thoả mãn tập hợp:

A+B ={a+b|aA, bB}

chứa nửa hệ thặng dư mod N2.

Bài toán 2.33. Cho số tự nhiên n >2 Chứng minh rằng:

1989|nnn

n

(20)

Bài toán 2.34. Sắp xếp dãy số nguyên tố theo thứ tự tăng dần p1, p2, Chứng minh rằng

pn!

pn(pn+ 1)(pn+ 2) (pn+1−1)

ZnN n >2.

Bài toán 2.35. Giả sử S là tập hợp tất số nguyên tố bé hơn 40 Tìm số k nhỏ nhất sao cho với tập con k phần tử của S đều tồn tại phần tử đôi phân biệt a, b, c sao cho a+b+c cũng số nguyên tố.

Bài toán 2.36. Số nguyên dươngn được gọi đáng ghét tồn số nguyên dương mmà trong tập hợp {1,2, ,28011980} có đúng n số x1 < x2 < < xn không đồng dư với nhau

theomod n Nếu điều khơng xảy thì n được gọi đáng yêu Xác định số nguyên dương đáng yêu bé nhất.

Bài toán 2.37. Cho số nguyên dương a, b Chứng minh tồn số nguyên dương (n1, n2, , nk) thoả mãn tính chấtni+ni+1|nini+1 ∀i= 0, k trong quy ước n0 =a, nk+1=b. Bài toán 2.38. Chứng minh số nguyên lớn hơn 17 đều biểu diễn thành tổng của số nguyên lớn hơn đôi ngun tố Chứng minh tính chất khơng đúng với 17

Bài tốn 2.39. Cho số nguyên tố p≥3 a1, a2, , ap−2 là số tự nhiên cho p không chia hết ak akk−1 với mọik Chứng minh chọn số để tích số có

số dư là khi chia cho p.

Bài toán 2.40. Với số nguyên dương n gọi S(n) là tổng chữ số của n Chứng minh rằng tồn tại k số tự nhiên a1, a2, , ak sao cho:

an+S(an) =am+S(am) ∀ 1≤n, mk

Bài toán 2.41. Chứng minh phương trình x3 +y3 +z3 −t3 = 42 có vơ hạn nghiệm ngun Số nghiệm ngun dương phương trình bao nhiêu, hữu hạn hay vơ hạn? Bài tốn 2.42. Giả sửn, a, b, c, dlà số tự nhiên (n≥2) thoả mãn a

b+ c

d <1vàa+c < n.

Cố định n, tìm giá trị lớn của a

b + c d.

Bài toán 2.43. Tập hợp S gồm k+m−1 số nguyên bất kỳ, mk ≥ 2,k|m Chứng minh rằng tồn tại m số số có tổng chia hết cho k.

Bài toán 2.44. Giả sử biểu diễn thập phân của

5 có dạng

5 = 2, a1a2 anbbb bbb| {z } msốb

an+1

Biết rằng b6=an, b =6 an+1 Chứng minh nm−2

Bài toán 2.45 (Open Question). Giả sửP là tập khác rỗng tập số nguyên tố cho với mọip1, p2, , pkP (khơng thiết phân biệt) ước số ngun tố của

số p1p2 pk + cũng thuộc vào P Hỏi tập hợp P có trùng với tập hợp tất số nguyên

(21)

21

Bài tốn 2.46. Tìm tất hàm số f :ZZ thoả mãn đẳng thức:

f(x3) +f(y3) +f(z3) = (f(x))3+ (f(y))3+ (f(z))3 ∀x, y, zZ.

Bài toán 2.47. Giả sử m là số nguyên dương lớn hơn cho trước Tìm số C lớn nhất cho:

X 1≤kn,(k,m)=1

1

kC

n

X

k=1

knN.

Bài toán 2.48. Chứng minh hai mệnh dề sau đây:

i) Nếu n > 49 thì tồn hai số nguyên a, b >1 sao cho a+b=n và:

ϕ(a)

a + ϕ(b)

b <1.

ii) Nếu n >4 thì tồn hai số nguyên a, b >1 sao cho a+b=n và:

ϕ(a)

a + ϕ(b)

b >1.

Bài toán 2.49. Với số tự nhiên n=at a2a1 xét hàm số:

T(n) = 10 X

ichẵn

ai+

X

ilẻ

ai.

Hãy tìm số nguyên dương A nhỏ cho tồn số tự nhiên n1, n2, , n148

m1, m2, , m149 thoả mãn hai điều kiện:

   

A=n1+n2+ +n148 =m1+m2+ m149

T(n1) =T(n2) = =T(n148)

T(m1) =T(m2) = =T(m149).

Bài toán 2.50. Ký hiệu ϕ(n) là số số nguyên dương nhỏ hơn n và nguyên tố nhau với n vàπ(n) là số số nguyên tố không vượt quá n Chứng minh với số tự nhiên

n >1 ta có:

ϕ(n)≥ π(n)

(22)(23)

Chương 3

Một số chủ đề số học chọn lọc

3.1 Số bập bênh

Bài toán 3.1.1 (Số bập bênh). Một số tự nhiên bập bênh đem nhân với9 ta được số viết theo thứ tự ngược lại chữ số Chẳng hạn số 1089 là một số bập bênh có chữ số vì 1089.9 = 9801 Vấn đề tìm tất số bập bênh có n chữ số Hơn tính số tất số bập bênh cón chữ số.

lời giải.Xét dãy sốF ibonaci{fn}xác định công thức truy hồi sauf0 = 0, f1= 1, fn+2 =

fn+1+fnnN Ngoài gọi số số bập bênh cón chữ số Sn Ta chứng minh

số có chữ số 1089 số bập bênh nhỏ với số tự nhiênn ≥4 ta có:

Sn =f[n/2]−1.

Thật vậy, kết luận thứ dễ dàng thu ta xét trực tiếp n = 1,2,3 Xét

n≥ Giả sử a1a2 anlà số bập bênh có n chữ số, điều có nghĩa là:

9.a1a2 an=an a2a1. (3.1)

Suy a1 = 1, an = Thay lại vào (3.1) thì80 + 9.a2a3 an0 = an−1 a20 =⇒a2 <2 Như a2 = a2 = 1, ta xét hai trường hợp này:

1 Nếu a2 = Từ (3.1) lấy theo mod 100 suy an−1 = 7, lại thay lại vào (3.1) suy ra:

9.(11.10n−2 + 79 +a3 an−200) = 97.10n−2 + +an−2 a300. Như vế trái lớn 99.10n−2 và rõ ràng lớn vế phải Loại.

2 Nếu a2 = Lấy (3.1) theo mod 100 suy raan−1 = Thay lại vào(3.1) ta có:

9.(10n−1 + 89 +a3 an−200) = 98.10n−2 + +an−2 a300 =⇒a3>7.

Như vậya3 nhận 1trong 2giá trị8 Gọi số nghiệm hai trường hợp làKn Tn tương ứng Khi rõ ràng ta có Sn=Kn+Tn

(24)

2.1 Bước Tính Kn. Dễ thấy K5 =K6 = Xétn ≥7 Ta có:

9.a4 an−3 = 8.10n−6 +an−3 a4. (3.2) Suy a4 ≥8 Xét trực tiếp dễ thấy a4 6= =⇒a4 = =⇒K7 = 0, K8 = (số 108981089) Xét n >9, đó9.a5 an−4 =an−4 a5 Đây cơng thức xác định số bập bênh có khơng qn−8chữ số (theo (3.1)) nên:

Kn =

[n/2] X

k=4

Sn−2k.

2.2 Bước Tính Tn.Dễ thấy T5 = (số 10989) Xét n ≥6, lấy (3.1) theo mod 1000 suy an−2 = Thay lại vào(3.1):

9.(109.10n−3 + 989 +a4 an−300) = 901 + 989.10n−3 +an−3 a400

=⇒9.a4 an−3 = 8.10n−6 −8 +an−3 a4. (3.3)

T6 = (số 109909) Xét n ≥ suy a4 ≥ 8, lại xét hai trường hợp Nếu a4 = 8, lấy (3.1) theo mod 1000 thu an−3 = 0, thay lại vào (3.1) ta có 9.a5 an−4 = 8.10n−8 +an−4 a5 Theo (3.2) số nghiệm phương trình Kn−2 Nếua4 = thay lại vào (3.1) suy

a5 an−4 = 8.10n−8 −8 +an−4 a5 Theo (3.3) số nghiệm phương trình Tn−2

Vậy ta có:

Tn=Kn−2 +Tn−2 =Sn−2.

Tóm lại chứng minh công thức sau dãy số Sn:

Sn=Kn+Tn=Sn−2+ [n/2] X

k=4

Sn−2k.

Đặc biệt với xác định nói dãy số F ibonaci thoả mãn:

fn=fn−2+ [n/2] X

k=4

fn−2k.

Việc chứng minh công thức không khó khăn dành cho bạn đọc tập nhỏ Bây để ý giá trị ban đầu hai dãy{Sn} và{f[n/2]−1}là hoàn toàn trùng Và ta kết luận Sn=f[n/2]−1 với số tự nhiênn≥4 Đây điều ta cần chứng minh

Xét dãy số bập bênh dạng đặc biệt sau đây: 

       

p1 = 1089

p2 = 10989

pn = 10 999| {z } 999 n−1số9

(25)

3.1 SỐ BẬP BÊNH 25 Khi với sơ đồ chứng minh dễ dàng thu dạng tổng quát sau tất số bập bênh:

pm1pm2 pmnpmn+1pmn pm2pm1.

Trong m1, m2, , , mnmn+1 số tự nhiên tuỳ ý

Lời Bình Đây tốn hay, vấn đề đặt khảo sát loại số đặc biệt điều

thú vị xuất xứ từ bập bênh Có thể thấy rõ qua cơng thức tổng qt xác định số bập bênh nêu trên, cấu trúc số có tính chất (3.1) nguồn gốc tên gọi thú vị Một điểm đặc biệt chứng minh dựa vào (3.2),(3.3) để thu công thức truy hồi đặc biệt dãy {Sn} Cần phải quan

sát cách tỉnh táo tránh suy luận thừa, khơng cần thiết lạc đề Để cho xác ta gọi số bập bênh kiểu số − bập bênh Một vấn đề có số bập bênh kiểu khác không, tức với số a tồn số

a−bập bênh, với giả thiết số a−bập bênh số mà đem nhân vớia ta số viết theo thứ tự chữ số ngược lại Đây tốn khơng q khó, chí cũ cần phải xét tất trường hợp a chữ số Kết a 1,9 Đặc biệt nếua = kết tương tự, khác biệt có cách xác định dãy {pn} Khi a= vai trò số 2178 thay vai trò số 1089 (chú ý

là2178.4 = 8712) Chính xác có kết sau đây:

Bài tốn 3.1.2. Chứng minh có số 4−bập bênh có nhiêu số 9−bập bênh.

Chứng minh trực tiếp kết thực việc khó (!!)

Thay cho lời kết Trong số học cịn vơ vàn loại số đáng ý khác chúng

ta sớm trở lại chủ đề với khảo sát chi tiết Dưới số toán loại số đặc biệt khác xuất kỳ thi học sinh giỏi gần trước kia: Bài toán 3.1.3 (Số đong đưa). Một số nguyên dương gọi đong đưa biểu diễn thập phân nó, hai chữ số đứng cạnh có số bằng và số khác 0, chữ số hàng đơn vị khác Tìm tất số ngun dương n sao cho n khơng có bội số nào là số đong đưa.

Bài toán 3.1.4 (Số luân phiên). Một số nguyên dương gọi luân phiên trong biểu diễn thập phân nó, hai chữ số đứng cạnh có số chẵn số lẻ. Tìm tất số ngun dương cho khơng có bội số ln phiên cả.

Bài toán 3.1.5 (Số bướng bỉnh). Cho số tự nhiên đôi nguyên tố nhau a, b, c. Một số tự nhiên gọi bướng bỉnh khơng biểu diễn dạng xab+ybc+zca với các số tự nhiên x, y, z Hỏi có số bướng bỉnh.

Bài tốn 3.1.6 (Số kim cương). Một số nguyên dương gọi kim cương 2005 nếu trong biểu diễn thập phân có 2005 chữ số đứng cạnh liên tiếp Dãy {an} tăng

ngặt số nguyên dương thoả mãn an< nC với số thực dương C nào Chứng minh

(26)

3.2 Định lý F ermat nhỏ ứng dụng đẹp

Bài toán 3.2.1 (Định lý F ermat nhỏ). Với số nguyên tốp cho trước số tự nhiên n tuỳ ý thì npn chia hết cho p.

Định lý quen thuộc với bạn, cách chứng minh truyền thống sử dụng phép ghép cặp hệ thặng dư Tuy nhiên muốn mời bạn thưởng thức lại cách chứng minh tuyệt đẹp toán (3.2.1) Cách chứng minh khác với cách chứng minh thông thưởng sử dụng hệ thặng dư, hai cách cách thứ hai mở rộng để chứng minh kết mạnh định lýF ermat (định lýEuler) dễ dàng

Chứng minh toán (3.2.1) Sử dụng phép đếm Ta chia vòng tròn thành p phần 1,2, , p tơ phần n màu cho trước Hai cách tô gọi đồng dạng qua phép quay chúng trở thành hình Đếm số lớp mà lớp bao gồm tất cách tô đồng dạng với

Ta có số cách tơ np, lớp cách tô đồng dạng với có cách tơ khác lớp có p phần tử (nghĩa p cách tơ sai khác phép quay) Chỉ có plớp mà lớp chứa n phần tử giống hệt (tương ứng vớin cách tơ tồn vịng trịn màu nhất) Vậy số lớp là:

npn

p .

Biểu thức dĩ nhiên phải số nguyên Điều chứng tỏ npn chia hết cho p.

Về tầm quan trọng định lý có lẽ nên dành hẳn chuyên khảo để viết nó, nhiên viết muốn giới thiệu tốn "nhỏ" thú vị, chứng minh trực tiếp đẹp mắt định lý nhờ kết hợp với đẳng thức thú vị:

1 =

1 +

1

6. (3.4)

Bài tốn 3.2.2 (IMO 2005). Tìm tất số nguyên dương nguyên tố với mọi phần tử dãy:

an= 2n+ 3n+ 6n−1 n ≥1.

Chứng minh toán (3.2.2).Ta chứng minh với số nguyên tố pbất kì tồn bội số p phần tử dãy {an} Thật vậy, rõ ràng 2|a1 = 10 3|a2 = 48, xét số nguyên tố p >3 Sử dụng định lýF ermatnhỏ ta có biến đổi sau:

ap−2 = 2p−2+ 3p−2+ 6p−2 −1 = 2p−1

2 + 3p−1

3 + 6p−1

6 −1

2 + +

1

6−1 mod p≡0 mod p.

(27)

3.3 MỘT SỐ TÍNH CHẤT CỦA HÀM TỔNG CÁC CHỮ SỐ 27

3.3 Một số tính chất hàm tổng chữ số

Đối với số tự nhiên n = a1a2 an ta xét S(n) = a1+a2 + +an gọi S hàm tổng

chữ số Trong mục chúng tơi trình bày số tính chất cùa hàm số Bài tốn 3.3.1. Chứng minh tính chất hàm S(n):

(1) S(a)≤a với số tự nhiên a

(2) S(a+b)≤S(a) +S(b) với số tự nhiên a,b

(3) S(ab)≤S(a)S(b) với số tự nhiên a,b.

Ba tính chất hiểu biết ban đầu hàm S(n), đơn giản quan trọng Chứng minh khơng q khó khăn bạn tự chứng minh chúng Bây mời bạn đến với2 kết đẹp đẽ sau đây:

Bài toán 3.3.2. Giả sử 10k −1|M với kN khi đó S(M)≥9k.

lời giải Đặt M =at a1a0 Với mọi i∈ {0,1, , k−1} ký hiệu:

Si =

X

ji modk

aj.

Theo giả thiết 10k −1|M =⇒10k −1|10lM với mọi l= 0,1, , k−1 Do ta có:

        

Sk−110k−1+Sk−210k−2+ +S1101 +S0100 chia hết cho10k−1

Sk−1100 +Sk−210k−1+ +S1102 +S0101 chia hết cho 10k −1

Sk−110k−2+Sk−210k−3+ +S1100 +S0101 chia hết cho10k−1.

Suy (Sk−1 + +S0)(10k−1 + + 10 + 1)≥k(10k −1) =⇒S(M) =

kP−1

i=0

Si ≥9k

Bài toán 3.3.3. S((10k −1)m) = 9k với mọi1≤m≤10k.

lời giải Đặt m =a1a2 as10t với a

s6= 0, sk Ta có:

S((10k −1)m) =S(a1a2 as−1 99| {z } 9

kssố9

(9−a1) (9−as−1)(10−as))

=

s−1 X

i=1

ai+ + +| {z + 9} kssố9

+

s−1 X

i=1

(9−ai) + (10−as)

= 9k Điều phải chứng minh

Lời Bình Hai kết tính chất đẹp hàmS(n), đẳng thức bài

(28)

Bài toán 3.3.4 (USAMO 2005). Đặt A = {1,2, , n} Gọi f(n) là giá trị nhỏ của số tự nhiên k có tính chất tồn tại n số nguyên dương phân biệt x1, x2, , xn thoả mãn:

S X

iI

xi

=k với mọi IA, I6=φ.

Chứng minh tồn tại 0< c1 < c2 thoả mãn c1log10nf(n)≤c2log10n với mọi n.

lời giải Trước hết ta chứng minh tồn của c1 Đặt m = [log10n] =⇒ 10m −1 < n.

Xét n số a1 = x1, a2 =x1+x2, ,an=x1+x2+ +xn Theo nguyên lý Dirichlet tồn

1≤i < jnsao cho aiaj mod(10m−1) Từ suy ai+1+ +aj ≡0 mod(10m−1)

Mặt khác áp dụng tốn (3.3.2) ta có f(n) =S(ai+1+ +aj)≥9m= 9[log10n]

Ta chứng minh tồn củac2 Chọnx1 = 10k−1, ,xn = (10k−1)n Vớik = [2 log10n+1.] Sử dụng toán (3.3.3) suy raS(P

iI

xi) = 9k với mọiIA,I 6=φ

=⇒f(n)≤9k ≤9[2 log10n+ 1].

Bài toán chứng minh

Một vấn đề khác khảo sát tính chất dãy {S(an)} với a số tự nhiên cố định Riêng vớia = lim

n→∞S(2

n) =∞ Đây tốn khó, bạn chứng minh rằng

S(2n)≥

2log2n từ dễ dàng suy kết tốn Chúng ta khơng bàn q sâu toán mà giải toán tương tự sau đây:

Bài tốn 3.3.5. Tìm tất cấc số tự nhiên k có tính chất tồn số thực dương ck sao cho:

S(kN)

S(N) ≥ck với số tự nhiên N bất kì.

lời giải Ta chứng minh số tự nhiên k thoả mãn điều kiện toán khi

trong phân tích tắc k khơng có thừa số nguyên tố khác Thật vậy,

k = 2p.5q thì ta có:

S(2p.5q.N)≥ S(10 p+q

.N)

S(5p.2q) =⇒

S(2p.5q.N)

S(N) ≥ck =S(5

p

.2q).

Ngược lại, k = 2p.5q.tvới số tự nhiên t >1 và nguyên tố với 10 Rõ ràng nếu

k thoả mãn tốn thìt Sử dụng định lý Euler ta có:

t|10ϕ(t)−1 =⇒ 10

ϕ(t)−1

(29)

3.3 MỘT SỐ TÍNH CHẤT CỦA HÀM TỔNG CÁC CHỮ SỐ 29 Trong cách viết chữ số ai Ta lại có:

Bm =

10(t)−1

t

= 10

ϕ(t)−

t

(10ϕ(t))m−1+ + 10ϕ(t)+ =B1B1 B1

| {z }

m

=⇒Bm+ =

10(t)+t−1

t =B|1B{z1 B}1

m

+1.

t >1nên a1a2(t)<99| {z } 99

ϕ(t)số9

Do ta có B1B1 B1+ =B1B1 (B1+ 1)

=⇒S

10(t)+t−1

t

m−1. (3.5)

Chọn m đủ lớn để 10(t)> t−1 ta có ngay:

S(10(t)+t−1) = +S(t−1). (3.6)

Tóm lại ta xét dãy số tự nhiên sau Am =

10(t)+t−1

t

Giả sử phản chứng, từ(3.5)và (3.6) ta suy ract

S(tAm)

S(Am)

≤ +S(t−1)

m−1 Chom → ∞suy ct= Mâu thuẫn Vậy ta có điều phải chứng minh Một câu hỏi mở đánh giá:

inf

S(kN)

S(N) |kN

.

Bài toán 3.3.6. Chứng minh tồn tại k số tự nhiên a1, a2, , ak sao cho:

an+S(an) =am+S(am) ∀1≤n, mk.

lời giải Ta chứng minh quy nạp theon rằng tồn tại n số tự nhiên khác đôi một

a1, a2, , an thoả mãn a1+S(a1) = a2+S(a2) = =an+S(an) Thật vậy, với n = chọn

a1 = 98, a2 = 107 98 +S(98) = 115 = 107 +S(107) Giả sử khẳng định tới n a1+S(a1) = a2+S(a2) = =an+S(an) = ls Rõ ràng tồn l ∈ {1,2, ,9}để

9|s+ 2l, đặt s+ 2l = 9m Dễ thấy m lớn số chữ số số ai, i= 1,2, , n Xét

số sau đây: (

a0

i = 10

m +a

i i= 1,2, , n

a0

n+1 = 10

ml.

Khi a0

i+S(a0i) = 10m + +s với i= 1,2, , n+ Vậy ta có điều phải chứng minh

(30)

Bài toán 3.3.7. Giả sử a, b, c là số tự nhiên thoả mãn S(a+b), S(b+c), S(c+a) đều nhỏ hơn Hãy tìm giá trị lớn S(a+b+c)

Bài toán 3.3.8. Với số tự nhiên n =at a2a1 xét hàm số

T(n) = 10 X

ichẵn

ai+

X

ilẻ

ai.

Hãy tìm số nguyên dương A nhỏ cho tồn số tự nhiên n1, n2, , n148

m1, m2, , m149 thoả mãn ba điều kiện:

   

A =n1 +n2+ +n148 =m1+m2+ m149

T(n1) =T(n2) = =T(n148)

T(m1) =T(m2) = =T(m149).

3.4 Hai ứng dụng phương trình P ell

Phương trình P ell cổ điễn phương trình có dạng x2 −Dy2 = 1 trong đó D là số nguyên dương khơng bình phương số tự nhiên Bằng cách sử dụng lý thuyết liên phân số, dùng bổ đề Dirichlet hay phương pháp hình học với đường hypecbol người ta chứng minh phương trình P ell ln có nghiệm

Từ việc có nghiệm ta thấy phương trình P ell có vơ hạn nghiệm Thật vậy, (x, y)là nghiệm thì(2x2−1,2xy)cũng nghiệm, bạn kiểm tra điều phép biến đổi tương đối đơn giản Vấn đề tìm cơng thức tổng quát tất nghiệm Để làm điều cần đến khái niệm nghiệm sở

Trong số nghiệm, ta lấy nghiệm (x0, y0) có tổng x0 +y0 nhỏ có thể, (x0, y0)gọi nghiệm sở Khi tất nghiệm phương trình P ell là:

      

xn=

(a+b

D)n+ (abD)n

2

D yn=

(a+b

D)n−(abD)n

2

D .

Việc chứng minh chi tiết định lý không thuộc phạm vi viết, bạn sử dụng phương pháp gen (sẽ dẫn ví dụ phần sau) để thực công việc

Mục đích chúng tơi trình bày với bạn hai ứng dụng đặc sắc hiểu biết phương trình P ell vào tốn số học

Bài tốn 3.4.1 (TST VMO 2005). Tìm tất hàm số f :ZZ thoả mãn đồng nhất thức sau với số nguyên x, y, z bất kỳ:

(31)

3.4 HAI ỨNG DỤNG CỦA PHƯƠNG TRÌNHP ELL 31

lời giải Ta tìm đẳng thức dạng a3+b3+c3 =d3+e3+f3 đúng vớin bất kỳ, trong

đó biến a, b, c, d, e, f phụ thuộc n Đây ý tưởng việc thực dẫn ta tới với phương trình P ell Thật vậy, ta có:

(x+l)3−(xl)3 = (y+h)3−(yh)3+ (zt)3−(z+t)3 ⇐⇒3lx2+l3 = 3hy2+h3−3tz2−t3.

Vì mục đích ta tìm cơng thức truy hồi có chứa n nên ta chọn x=n, l= đẳng thức trở thành:

3n2+ = 3hy2 +h3 −3tz2−t3. (3.7) Từ (3.7) suy ht ≡ mod 3, chọn h = 2, t = (3.7) tương đương với 3n2 + = 6y2+ 8−3z2−1⇐⇒3n2−6 = 6y2+ 8−3z2 −1⇐⇒n2−1 = 2y2−z2 Ta lại có:

(

2.12−n2 = 2−n2

2.52−72 =

=⇒2−n2 = (2.12 −n2)(2.52−72) = (

2.1−n)(

2.1 +n)(

2.5−7)(

2.5 + 7) = [(

2.1−n)(

2.5−7)][(

2.1 +n)(

2.5 + 7)] = [7n+ 10−

2(5n+ 7)][7n+ 10 +

2(5n+ 2)] = (7n+ 10)2−2(5n+ 7)2.

Và 2(5n+ 7)2−(7n+ 10)2 =n2−2, ta chọn y = 7n+ 10, z = 5n+ 7.Cuối ta có đẳng thức:

(n+m)3−(nm)3 = (5n+ 9)3−(5n+ 5m)3+ (7n+ 9m)3−(7n+ 11m)3.

Đến phép quy nạp đơn giản (các bạn tự thực hiện) ta có:

f(x) =     

0 với xZ f(1) =

x với xZ f(1) =

x với xZ f(1) =−1.

Bài toán 3.4.2 (TST VMO 2002). Tìm tất đa thức P(x) hệ số nguyên cho đa thức sau bình phương đa thức hệ số nguyên:

Q(x) = (x2+ 6x+ 10)(P(x))2−1.

lời giải Giả sử tồn đa thức hệ số nguyên R(x) ∈ Z[x] thoả mãn Q(x) = (R(x))2 Do

x2+ 6x+ 10 = (x+ 3)2 + 1>0 (R(x))2 ≥0 nên P(x)6= với xR Có thể giả sử

P(x)>0 với mọixR bậc P chẵn, đặt deg(P) = 2n

Từ giả thiết ta có (m2+ 1)(P(m−3))2−1là số phương với giá trị mnguyên, nên theo định lý phương trình P ell suy tồn f :NN cho với mN thì:

P(m−3) = 2√m2+ 1

(m+

m2+ 1)2f(m)+1−(m

m2+ 1)2f(m)+1

(32)

Gọi alà hệ số bậc cao củaP(x),a= 06 vìdeg(P) = 2n nên lim

x→∞

P(x)

x2n =a6= Từ

suy tồn 2n+ số nguyên dương phân biệt x1, x2, , x2n+1 cho 2f(xi) + = 2n+

với1≤i≤2n+ Xét đa thức:

A(x) =P(x−3)−

2

x2+ 1

(x+

x2+ 1)2n+1− (x

x2+ 1)2n+1

.

Ta có deg(A) ≤2nA(xi) = với i = 1,2n+ suy A(x) = (một đa thức bậc k

khơng có qk nghiệm)

=⇒P(x−3) =

x2+ 1

(x+

x2+ 1)2n+1 − (x

x2+ 1)2n+1

.

Đổi biến, ta thu được:

P(x) =

2√x2+ 6x+ 10

(x+ +

x2+ 6x+ 10)2n+1−

(x+ 3− √

x2+ 6x+ 10)2n+1

.

3.5 Định lý phần dư Trung Hoa

Bài tốn Hàn Tín điểm binh toán tiếng đến mức học sinh tiểu học nghe nói đến Đó xuất phát điểm cho định lý Trung Hoa phần dư mà phát biểu chứng minh

Bài toán 3.5.1. Xét hệ phương trình đồng dư bậc ẩn:

(H)         

xb1 mod m1

xb2 mod m2

xbn mod mn.

Chứng minh rằng (H) có nghiệm nếu (mi, mj) =dij|bibj với 1≤i < jn.

lời giải Ta chứng minh hai chiều định lý Chiều thuận hiển nhiên, ta có:

(

x=timi+bi

x=tjmj +bj

=⇒dij = (mi, mj)|tjmjtimi =bibj.

Ngược lại, ta xét trường hợp {mi} đôi nguyên tố (đây định lý Trung

Hoa) Ta biết với số nguyên xđiều kiện cần đủ để tồn số nguyên yxy≡1 mod n

là (x, n) = Thật vậy, (x, n) = theo định lý Bezout ta có tồn số nguyên

u, v cho ux+vn= suy ux≡1 mod n Chiều ngược lại điều hiển nhiên

Bây đặt M =

n

Q

i=1

mi Mi =

M mi

(33)

3.5 ĐỊNH LÝ PHẦN DƯ TRUNG HOA 33

tạici choMici ≡1 mod mi Lấy x= n

P

i=1

Mibici, rõ ràng nghiệm hệ (H)

Bây xét trường hợp (mi, mj) =dij|bibj với 1≤i < jn Đặtmc = k

Q

i=1

ci

i

Khi hệ (H)có thể viết dạng:

Với mọi1≤cn

        

xbc mod p βc1

xbc mod p βc2

xbc mod p βck

k

Sắp xếp lạin hệk phương trình thành k hệ n phương trình sau:

Với mọi1≤ik

        

xb1 mod p

β1i

i

xb2 mod p

β2i

i

xbn mod p βni

i .

Các phương trình hệ k hệ xét theo modun số nguyên tố, βciβdi xét hệ:

(

xbc mod p βci

i

xbd mod p βdi

i .

Hệ tương đương với phương trình xbc mod p βci

i (suy từ điều kiện

dij|bibj) Nghĩa ta đưa trường hợp tổng quát trường hợp riêng định lý Trung

Hoa vừa giải Định lý chứng minh hoàn toàn

Đây định lý thực lý thú bổ ích, ứng dụng tương đối đơn giản ý nghĩa:

Bài toán 3.5.2. Chứng minh với số tự nhiên n bất kỳ tồn hai số nguyên x, y

thoả mãn n|x2−34y2+

lời giải Có thể đặt n = 3αm với (m,3) = 1và α >0.

Ta có 52−34.12 + 32 = tồn tạij để3j ≡1 mod m =⇒m|(5j)2−34j2+

Ta lại có 32 −34.12 + 52 = 0, theo định lýEuler thì 5ϕ(3α)

≡1 mod 3α

=⇒3α

3·5ϕ(3α)−1

−34

5ϕ(3α)−1

+ 1.

Theo toán (3.5.1) tồn tạix, y thoả mãn hai hệ đồng dư: (

x≡5j mod m x≡3·5ϕ(3α)−1 mod 3α

(

(34)

Khi rõ ràng n|x2−34y2+ Đây điều phải chứng minh

Cuối cùng, mời bạn sử dụng định lý phần dư Trung Hoa để giải số toán sau đây:

Bài tốn 3.5.3. Tìm tất số tự nhiênn có tính chất tồn số ngun m 2n−1 ước số của m2+ 9.

Bài toán 3.5.4. Cho đa thức hệ số nguyên P(x)và tập hợp hữu hạn số nguyên p1, p2, , pr

khác thoả mãn với số nguyên n thì P(n) là bội sốpi nào Chứng minh khi

đó tồn số i pi|P(n)với mọi nZ.

Bài toán 3.5.5. Giả sử a1, a2, , an n số nguyên đôi khác và b1, b2, , bn n số

nguyên tuỳ ý Chứng minh với số tự nhiên t cho trước tồn đa thức với các hệ số là ±0,±1, , ,±(t−1) sao cho f(ai)≡bi mod t với mọi i= 1,2, , n.

Lời Bình Hãy ý đến dạng tốn(3.5.5) trong trường số thực, có nhất

một đa thức hệ số thựcP(x)thoả mãnP(ai) =bi vớii= 1,2, , n Chứng minh điều

bạn sử dụng công thức nội suy Lagrangequen thuộc

3.6 Biểu diễn số

Các toán thuộc dạng toán biểu diễn số đa dạng phong phú Có nhiều số chúng thuộc loại kinh điển tốn biểu diễn số thành tổng bình phương, lập phương, Đó tốn thú vị viết khơng có ý định tổng kết lại kết kinh điển mà xét đến tốn biểu diễn số hữu tỉ Vấn đề thứ chúng tơi đặt tìm hiểu việc phân tích số hữu tỉ dương dạng tổng lập phương k số hữu tỉ dương Rõ ràng định lý F ermat nói lên k khơng thể Vậy k nhỏ Câu trả lời 3, xác ta có tốn sau đây:

Bài tốn 3.6.1. Mọi số hữu tỉ dương biểu diễn vô hạn cách thành tổng lập phương ba số hữu tỉ dương.

lời giải Xét số hữu tỉ dương r Luôn tồn số hữu tỉv sao cho:

3 r

3r

2 < v <

3r.

Chọn u= 3rv

3r+v3, s=v(1 +u), z =sut =

s

3(1−u2) Và chọn x=st, y=tz Từ cách chọn v ta có < u <

3 =⇒ 3(1 −u

) > 1, mà 3u(1 −u2) < =⇒ x, y > Vậyx, y, z số hữu tỉ dương

Lại có: (

(35)

3.6 BIỂU DIỄN SỐ 35

=⇒x3+y3+z3 = 3s(1−u)t2 = s

(1−u) 3(1−u2)2 =

s3

3(1 +u)(1−u2) =

v3(1 +u)2 3(1−u2) =

v3(1 +u) 3(1−u) =r. Do r tổng lập phương của3 số hữu tỉ dương Để chứng minh số cách biểu diễn vơ hạn ta chọn v đủ gần √3

3r Khi đóz =su đủ nhỏ Do ta tạo vơ hạn cách biểu diễn theo cách Từ ta có hệ sau:

Hệ 3.6.1. Mọi số hữu tỉ tổng lập phương của số hữu tỉ.

Hệ 3.6.2. Với số nguyên dương n bất kỳ, phương trình x3 +y3+z3 = nt3 có vơ hạn nghiệm ngun dương ngun tố nhau.

Hệ 3.6.3. Với số nguyên dương k≥3, số hữu tỉ dương biểu diễn vô hạn cách dạng tổng lập phương của k số hữu tỉ dương.

Từ hệ 3.6.3 vấn đề đặt giải trọn vẹn Ngồi lời giải cịn cho ta phương pháp phân tích số hữu tỷ thành tổng lập phương số hữu tỉ Chẳng hạn vớir= 3/5 thì:

3

5 =r = 86 105 + 73 735 + 18 49 .

Bây ta cải tiến cách chứng minh định lý cách chọn v đủ nhỏ v > √3 3r cho u2 <

3 Khi khơng khó khăn để chứng minh −1 < u < 0, z < 0, t > 0, y > 0,

x >0 Kết dẫn ta tới với định lý sau:

Bài toán 3.6.2. Bất kỳ số hữu tỉ dương biểu diễn vơ hạn cách dạng

x3+y3−z3 trong đó x, y, z là số hữu tỉ dương.

Hệ toán 3.6.2 thu cách áp dụng cho số r+t3 vớit là số thực dương: Hệ 3.6.4. Bất kỳ số hữu tỉ dương biểu diễn dạng x3+y3−z3−t3 trong đó x, y, z, tlà số hữu tỉ dương.

Chúng ta xét đến vấn đề thứ hai, biểu diễn số hữu tỉ r dương dạng:

r = a

1+ +a

n

b3

1+ +b3n

vớia1, , anb1, , bn số nguyên dương Câu hỏi đặt vớin

có thể biểu diễn

Trước hết hiển nhiên thấy n >1 Ta xét trường hợp đơn giản n

Trường hợp n = 2.Xét a1, a2, b1, b2 số nguyên dương cho a1 =b1 Khi đó:

a3 1+a32

b3 1+b

3

= (a1+a2) (b1+b2)

(a2

1−a1a2+a22) (b2

1−b1b2+b22)

(36)

Ta chọn số cho a21−a1a2+a22 =b

1−b1b2+b22, chẳng hạn chọn a1 =a2+b2

=⇒ a

3 1+a

3

b3 1+b32

= 2a2+b2

a2+ 2b2

.

Nếu 1/2< r <2, viếtr =a/btrong a, blà số nguyên dương thỏa mãn b <2a, a <2b Ta chọna2= 2abb2 = 2ba Khi r =

a3 1+a

3

b3 1+b32

thỏa mãn

Nếur không thuộc khoảng(1/2,2) Luôn tồn số hữu tỉp/q vớip, q nguyên dương cho

2 < r

p3

q3 <2 Theo trường hợp r

p3

q3 biểu diễn dạng:

rp

3

q3 =

a3 1+a

3

b3 1+b32

=⇒r = (qa1)

3+ (qa2)3 (pb1)3+ (pb2)3. Do trường hợp n = 2, biểu diễn thực tồn

Trường hợp n = Sử dụng trường hợp n = toán 3.6.2 suy tồn số nguyên dương a, b, c, d, e, f, g, hsao cho:

r= a +b3

c3+d3 =

e3+f3−g3

h3 =

(ag)3+ (bg)3 (cg)3+ (dg)3 =

(be)3+ (bf)3 −(bg)3 (bh)3 . Theo tính chất tỉ lệ thức:

r= (ag)

3+ (be)3+ (bf)3 (cg)3+ (dg)3 + (bh)3.

Ta có vớin = 3biểu diễn xác lập Mặt khác từ trường hợp n= vàn = sử dụng tính chất tỉ lệ thức ta có tốn sau:

Bài toán 3.6.3. Cho số nguyên dương n >1 Khi số hữu tỉ dương biểu diễn dưới dạng:

a3

1+ +a3n

b3

1+ +b3n

trong đó a1, , an b1, , bn là số nguyên dương.

Như vấn đề thứ hai giải trọn vẹn Bây tương tự hóa dạng biểu diễn vấn đề thứ hai ta có tốn sau:

Bài toán 3.6.4. Chứng minh với số hữu tỷ dương r đều tồn số nguyên dương a, b, c, m, n, pthỏa mãn:

r= a

+b3+c5

m7+n11+p13. (3.8)

lời giải Giả sửp là số nguyên tố, với số tự nhiênn ký hiệuvp(n)là số mũ của p trong

phân tích tắc n Ta chứng minh nhận xét:

Nhận xét.Với số tự nhiên t, s, m, n (m, n) = 1, tồn số tự nhiên

(37)

3.7 MỘT DẠNG PHƯƠNG TRÌNHDIOP HAN T E ĐẶC BIỆT 37

Chứng minh. Giả sử p số nguyên tố, sử dụng định lý Bezout ta tìm số nguyên không âm αp, βp thỏa mãn αpmβpn=vp(s) =vp(t) Dễ dàng chứng minh rằng:

a =Yp, b =Yp

thỏa mãn điều kiện Nhận xét chứng minh

Với số thực dươngrchúng ta chọn số tự nhiêna, b, c, m, n, pthỏa mãna2 =rm7,b3 =rn11 c5=rp13 Khi rõ ràng(3.8) thỏa mãn Điều phải chứng minh

Bài toán 3.6.4 tốn dễ, có chút thay đổi nhỏ, chẳng hạn thay đổi vai trò dấu cộng dấu trừ, ta có tốn khó Mà ví dụ: Bài tốn 3.6.5. Cho số ngun dương n >1 Tìm số ngun dương nhỏ khơng có dạng

nanb

ncnd với số nguyên dương a, b, c, d nào đó.

Các bạn tự giải toán xem tập

3.7 Một dạng phương trình Diophante đặc biệt

Trong mục muốn ghi chép lại lời giải ba phương trình Diophante nghiệm tự nhiên sau đây:

4xyx−1 =z2 (3.9)

4xyxy=z2 (3.10) 4xyzxy=t2. (3.11) Trong (3.9) (3.10) phương trình Euler,Golbach cho (3.9) lời giải đẹp đẽ Euler mơ cách chứng minh để giải quyết(3.10) Công việc cuối sử dụng ký hiệuJ acobi (mở rộng ký hiệuLegendre) luật thuận nghịch bậc hai để giải trọn vẹn (3.11)

Bài tốn 3.7.1. Chứng minh phương trình (3.9) khơng có nghiệm tự nhiên.

lời giải Giả sử phản chứng, gọi a là số nhỏ có tính chất tồn tạim, n tự nhiên mà:

4mnm−1 = a2. (3.12) Cộng 4m2−4mavào hai vế (3.12) ta được:

4m(na+m)−m−1 = (a−2m)2. (3.13) Dễ dàng chứng minh được:

a < m. (3.14)

(38)

na+m < n vế trái (3.13) nhỏ vế trái (3.12), tức là(a−2m)2 < a2, điều trái với giả thiết xác định số a giá trị tự nhiên nhỏ z thỏa mãn (3.9)

Ta chứng minh tiếp rằng:

4n−1>2a. (3.15) Thật vậy, cộng 4(n−1)2 −2a(4n−1) vào hai vế của (3.13) ta được:

(4n−1)(m−2a+ 4n−1)−1 = (a−(4n−1))2

⇐⇒4n(m−2a+ 4n−1)−(m−2a+ 4n−1) + = (a−(4n−1))2.

Do đóz =|a−(4n−1)|thỏa mãn (3.9), theo cách xác định sốata dễ dàng suy ra(3.15)

Từ (3.12),(3.14),(3.15) ta suy a2 + = (4n−1)m > 2a.a = 2a2 =⇒ a2 < 1 Điều vơ lý chứng minh kết luận tốn

Bài tốn 3.7.2. Chứng minh phương trình(3.10) khơng có nghiệm tự nhiên.

lời giải Giả sử phản chứng, gọia là giá trị tự nhiên nhỏ của z thỏa mãn (3.10), nghĩa

là tồn m, n tự nhiên cho:

4mnmn =a2. (3.16) Nhân hai vế (3.16) với4 biến đổi ta có:

(4m−1)(4n−1)−1 = 4a2. (3.17) Cộng 4(4n−1)2−8a(4n−1) vào hai vế của(3.17) dẫn tới:

(4n−1−8a+ 4(4n−1))(4n−1)−1 = 4(a−4n+ 1)2. (3.18) Hai đẳng thức (3.17),(3.18) có dạng giống nhau, cho thấy (3.10) có nghiệm z =

|a−4n+ 1|, theo cách xác định a ta suy bất đẳng thức:

a ≤ |a−4n+ 1|=⇒a2 <(a−4n+ 1)2.

Do từ hai đẳng thức (3.17),(3.18) ta có:

(4n−1−8a+ 4(4n−1))(4n−1)>(4n−1)(4m−1) =⇒4n−1>2a.

Vì (3.16) đối xứng vớim, nnên với lý luận tương tự ta có4m−1>1 Đặt: (

4m−1 = 2a+p

4n−1 = 2a+q.

Trong đóp, q số tự nhiên Như (4n−1)(4m−1) = 4a2+ 2a(p+q) +pq, từ (3.17) suy 2a(p+q) +pq= với số tự nhiên a, p, q Điều vơ lý Bài tốn chứng minh xong

(39)

3.7 MỘT DẠNG PHƯƠNG TRÌNHDIOP HAN T E ĐẶC BIỆT 39

lời giải Ta sử dụng ký hiệuJ acobi trong chứng minh, bạn đọc chưa rõ lý thuyết này

có thể đọc phần thích sau

Giả sử phản chứng (x, y, z, t) nghiệm tự nhiên (3.11) Thế thì: 4zt2+ 1

4yz−1 = 4zx−1 số nguyên, tức là:

(2zt)2 ≡ −z mod (4yz−1). (3.19) Giả sử z = 2αz0 với a số nguyên không âm, z0 số tự nhiên lẻ Khi sử dụng tính chất ký hiệuJ acobi ta có:

z

4yz−1

=

−1 4yz−1

2α 4yz−1

z0

4yz−1

.

Thừa số thứ bằng−1, thừa số thứ ba

4yz−1

z0

(−1)

z0−1 =

−1

z0

(−1)

z0−1 = Nếu a = 2k + với số nguyên không âm k z = 2z00 thừa số thứ hai

2 4yz−1

=

8yz00−1

= Nếu a = 2k với số nguyên không âm k

2 4yz−1

= =⇒

z

4yz−1

= −1 Điều mâu thuẫn với đồng dư thức (3.19) Bài toán chứng minh xong

Luật thuận nghịch bậc (The law of reciprocity) Đối với số nguyên tố p số nguyêna nguyên tố vớip ta xét ký hiệu Legendre sau:

a p = (

1 tồn xZx2≡ a mod p

−1 không tồn xZx2 ≡a mod p. Chứng minh tiêu chuẩn Euler:

a p

a p−1

2 mod p.

Tiếp theo chứng minh bổ đềGauss: gọi slà số phần tử tập hợp

ia

1≤ip

−1

mà chia cho p số dư nhận lớn p/2 Với a >1,(a, p) = 1, đó:

a p

= (−1)s.

Chứng minh tiếp đẳng thức Cayley: với hai số nguyên tố lẻp, q khác ta có:

p−1 X i=1 iq p +

q−1 X j=1 jq q

= p−1 ·

(40)

Suy luật thuận nghịch bậc hai: với hai số nguyên tố lẻp, q ta có: p q · q p

= (−1)

p−1 ·

q−1 .

Ký hiệuJ acobi định nghĩa sau: với số tự nhiên lẻn =Qpti

i số nguyênanguyên

tố với n, đó:

a n

=Y a

pi

ti

.

Trong vế phải ký hiệu Legendre Khi có luật thuận nghịch bậc hai cho ký hiệu J acobi: với hai số tự nhiên lẻ nguyên tố m n ta có:

m n · n m

= (−1)

m−1 ·

n−1 .

3.8 Số nguyên phức

Giới thiệu. Các số nguyên phức Gauss lần sử dụng Gauss nghiên cứu ông tương hỗ bậc hai, ông coi người sử dụng số phức số học cách rõ ràng mạch lạc Lớp số có tầm quan trọng định số học sơ cấp Trong mục đề cập đến khái niệm tính chất số nguyên phức Gauss (mà sau ta gọi số nguyên phức) đồng thời đưa số ví dụ để minh họa cho tính chất

3.8.1 Các khái niệm mở đầu

Số nguyên phức số có dạng a+bitrong ilà đơn vị ảo, a, blà số nguyên Khi hiển nhiên thấy với phép tốn cộng nhân trường số phức số phức

Gaussta thu số phức Gauss

Với số phức z = a+ bi (a, bR) số z0 = abi gọi số phức liên hợp z Như số số nguyên phức số phức liên hợp số nguyên phức

Ta dễ dàng chứng minh tính chất sau số phức liên hợp:         

(z0)0 =z

Nếuz =u+v thìz0=u0+v0

Nếuz =uv z0=u0−v0

Nếuz =uv thìz0=u0v0

Ta nói số nguyên phức z chia hết cho số nguyên phức t tồn số nguyên phứcusao choz =tu, viết làt|z Rõ ràng nếua+bi, c+dilà số ngun phức c+di khác 0thì ta có thương số:

a+bi c+di =

(a+bi)(cdi)

c2+d2 =

ac+bd c2+d2 +

(41)

3.8 SỐ NGUYÊN PHỨC 41 Như vậyc+di|a+bi nếuc2+d2|ac+bdc2+d2|bcad Hơn ta có tính chất sau:

(

Nếu u|t t|z thìu|z

Nếu t|zi vớii = 1,2, , n thìt|z1u1+z2u2+ +znun với mọiu1, u2, , un

Bây ta làm quen với khái niệm chuẩn số phức.Chuẩn số phức tích số phức liên hợp Nếu kí hiệu chuẩn số phức z N(z) thìN(z) =zz0. Hơn z =a+bitrong a, b số thực N(z) = a2 +b2 Các bạn đọc dễ dàng chứng minh tính chất sau:

    

N(z) =N(z0)

N(uv) =N(u)N(v)

z|t=⇒N(t)|N(z).

Hai số nguyên phức khác thỏa mãn số chia hết cho số gọi hai số nguyên phức liên kết Như vậy z, t hai số nguyên phức liên kết z|t t|z Từ

N(z)|N(t)và N(t)|N(z) N(t) =N(z)suy hai số nguyên phức liên kết với chuẩn chúng Ta cịn có thêm số tính chất bản:

(

Nếu z liên kết với t thìz0 cũng liên kết với t0

Nếu z chia hết cho t số liên kết vớiz chia hết cho số liên kết vớit

Bây ta tìm số nguyên phứculiên kết với số nguyên phứczcho trước Theo định nghĩa z =tu N(z) =N(t)N(u) Theo tính chất ta có N(z) = N(t)6= suy N(u) =

Viết u = a + bi N(u) = a2 +b2 = Điều xảy trường hợp (a, b) ∈ {(0,1),(0,−1),(1,0),(−1,0)} Kiểm tra đơn giản chiều ngược lại ta có số phức liên kết z z,z, iz,iz Ta có định lý sau:

Định lý 3.8.1. Bất kì số nguyên phức z khác nào có đúng số nguyên phức liên kết với là z,z, iz,iz.

3.8.2 Thuật toán Euclid và ước chung lớn hai số nguyên phức

Định lý 3.8.2. Nếu z t là số nguyên phức khác thì tồn số nguyên phức c r

sao cho z =ct+r N(r)≤

2N(t)

Chứng minh Ta viết z

t = x+yi x, y số hữu tỉ Gọi α β

số nguyên gần x, y Đặt x1 = xα y1 = yβ Khi x1, y1 số hữu tỉ

|x1|,|y1| ≤

(42)

Từ định lý ta có thuật tốn Euclid tìm dãy số nguyên phức r0, r1, cho số ti số nguyên phức thỏa mãn:

    

r0=t, r1 =r

ri =tiri+1+ri+2 với i= 0,1,2,

N(ri+1)< N(ri)

Dãy tiếp tục chừng ri khác0

Nhưng ý dãy chuẩn N(ri) dãy số tự nhiên giảm dần nên dãy ri không

thể kéo dài vơ hạn Do bắt buộc phải tồn số nguyênn chorn−2 =tn−2rn−1 hay qui ước rằngrn = Dễ dàng nhận thấy từ kết nàyrn−1 số chia hết z t chia hết cho ước số chung hai số Như ta có hệ (định nghĩa): Hệ 3.8.1. Với hai số ngun phức khác 0cho trước có ước số chung chia hết cho ước số chung khác hai số Các ước số chung gọi ước số chung lớn hai số này.

Từ ta có khái niệmuớc chung lớn cho nhiều sốcác số nguyên phức nguyên tố nhau Các số nguyên phức gọi nguyên tố chúng khơng có ước chung khác ±1,±i

Một định lý quan trọng nghiên cứu vấn đề ước số chung lớn nhất, mở rộng định lýBezoutcho số nguyên

Định lý 3.8.3. Hai số nguyên phức nguyên tố tồn số nguyên phức x, y sao cho ax+by=

Chứng minh Chiều ngược lại dễ dàng Ta chứng minh chiều thuận Giả sử(a, b) = 1.

Ta gọi S tập hợp số có dạng az+bt z, t số nguyên phức không đồng thời 0.N(S)được kí hiệu tập hợp chuẩn phần tử S Rõ ràng tồn phần tử nhỏ N(S) Giả sử phần tử bằngn Khi tồn tạiαS

sao choN(α) =n

Do αS nên tồn z1, t1 số nguyên phức không đồng thời cho

α = az1 + bt1 Ta chứng minh phần tử sS chia hết cho α Thật vậy, với

s=az+btS ta viếts =+r N(r)< N(α)

Mặt khác r = a(zcz1) +b(tct1) nên r 6= rS Điều mâu thuẫn với cách chọn α N(r)< N(α)

Vậyr= hay ta có nếusS thìs chia hết cho α

a, bS a, b chia hết cho s Vì (a, b) = nên suy N(s) = Khơng giảm tổng qt ta giả sử s= Khi tồn x=z1;y=z2 số nguyên phức không đồng thời cho ax+by = Định lý chứng minh

(43)

3.8 SỐ NGUYÊN PHỨC 43 Hệ 3.8.2. Với số nguyên phức a, b, c sao cho (a, b) = b|ac thì b|c. Hệ 3.8.3. Nếu (a, b) = (a, c) = thì (a, bc) =

3.8.3 Số phức nguyên tố vấn đề phân tích số nguyên phức

Chúng ta biết số ngun phứcz khơng liên kết với1cũng có 8ước số là1,−1, i,i, z,z, iz,iz Từ người ta định nghĩa số nguyên phức làsố nguyên phức nguyên tốnếu có ước số phân biệt

Điều tương đương với khẳng định số nguyên phức số nguyên phức nguyên tố có chuẩn lớn 1và khơng biểu diễn thành tích hai số phức có chuẩn lớn Như số phức liên kết liên hợp với số phức nguyên tố số phức nguyên tố

Như số nguyên phức nguyên tố mở rộng khái niệm số nguyên tố Z Vấn đề đặt (tương tự Z) phân tích số nguyên phức thành tích số nguyên phức nguyên tố Dễ thấy số nguyên phức nguyên tố có cách phân tích (là nó) Vậy với số nguyên phức trường hợp tổng qt Ta có định lý sau:

Định lý 3.8.4. Bất kỳ số nguyên phức có chuẩn lớn hơn 1cũng tích hữu hạn các số nguyên phức nguyên tố.

Chứng minh Ta dùng phản chứng để chứng minh định lý này.

Gọi M tập hợp số có chuẩn lớn hơn1nhưng khơng phân tích thành tích hữu hạn số nguyên phức nguyên tố N tập hợp chuẩn số phức M Nếu

M khác rỗng, suy N khác rỗng Do N tập số nguyên dương nên tồn phần tử nhỏ Ta gọi phần tử làm Khi tồn số phức zM cho N(z) =m

Hiển nhiên z không số nguyên tố N(z) = m >1 Do theo định nghĩa số nguyên tố ta suy tồn số nguyên phức u, v cho u, v 6∈ {1,−1, i,i, z,z, iz,iz} thỏa mãn z = uv Khi N(u)N(v) = N(z) = m Do u v khác tám giá trị nêu nên

N(u), N(v)>1 =⇒1< N(u), N(v)< m

Theo cách chọn m ta có u, v 6∈ Mu, v có chuẩn lớn nên chúng phân tích thành tích hữu hạn số nguyên phức nguyên tố Từ z phân tích thành tích hữu hạn số phức nguyên tố Mâu thuẫn với cách chọn z Vậy giả sử ta sai hay M

phải tập rỗng Do ta có điều phải chứng minh

(44)

Định lý 3.8.4 ý quan trọng Nó đem lại cho số nguyên phức tính chất gần gũi với số nguyên dạng phân tích thành tích số phức nguyên tố Có thể khám phá nhiều tính chất tương tự với tính chất quen thuộc Z Chẳng hạn:

Định lý 3.8.5. Cho số nguyên phứcz1, z2, , znđôi nguyên tố có tích

là lũy thừa cấp n của số nguyên phức Khi số zi cũng lũy thừa số

nguyên phức.

Lý thuyết số nguyên phức nhiều điều thú vị, nghiên cứu chi tiết sâu sắc quay trở lại viết khác Bây giờ, với hiểu biết ban đầu đây, thử vận dụng để giải số toán tương đối quen thuộc

3.8.4 Sử dụng số nguyên phức để giải số toán

Bài toán 3.8.1. Cho a, b, c là số nguyên dương cho (b, c) = Biết tồn n

nguyên dương lớn hơn sao cho an =b2+c2.

i) Chứng minh rằng a là tổng hai số phương. ii) Biết rằng n = p+

2 trong đó p là số nguyên tố lẻ có dạng 4k+ (trong đó k ngun dương) Chứng minh tích bc chia hết cho p.

lời giải Ta có phân tíchan= (b+ci)(bci) Gọid = (b+ic, bic)suy ra:

d|(b+ic)−(bic) = 2ic=⇒N(d)|N(2ic) = 4c2. (3.20)

Tương tự d|(b+ic) + (bic) = 2b=⇒N(d)|4b2. (3.21) Lại có N(d)|N(b+ic) = b2 +c2 Chú ý rằng b, c nguyên tố đó b2+c2 lẻ suy N(d) lẻ Vậy N(d)|b2 N(d)|c2 Cũng (b, c) = N(d) nguyên khơng âm nên ta có

N(d) = Vậy b+ic, bicnguyên tố

Từ tính chất suy tồn số nguyên x, y cho b+ic= t1(x+iy)n bic=

t2(xiy)ntrong đót1, t2 ∈ {1,−1, i,i}vàt1t2 = Hệ công thức làa =x2+y2 tổng hai số phương Đây nội dung cần chứng minh (i)

Với n = p+

2 p số nguyên tố dạng 4k + (k > 1) Khi có hai trường hợp sau xảy ra:

Trường hợp 1: (b+ic)2= (x+iy)p+1,(bic)2 = (xiy)p+1 Trường hợp 2: (b+ic)2=−(x+iy)p+1,(bic)2 =−(xiy)p+1

Trong hai trường hợp sau đem hai biểu thức trừ cho đồng phần thực phần ảo ta có|2bc|= (p+ 1)(xpyypx) Theo định lý nhỏ Fermat ta cóbc chia hết cho

p Đây yêu cầu của(ii) Bài toán chứng minh hồn tồn

(45)

3.9 PHƯƠNG TRÌNH CARM ICHAEL 45

lời giải.Giả sửn3−1 =k2 Lý luận tốn3.8.1, từ phân tíchn3 = (ki)(k+i)ta có

kik+inguyên tố Do tồn số nguyêna, bsao choki= (abi)3 vàk+i= (a+bi)3 Đồng phần thực phần ảo ta suy ra 3a2bb3=−1 Dễ dàng thấy

a= 0, b = Do giá trị n thỏa mãn n=

Một ứng dụng đặc sắc số nguyên phức Gauss chứng minh định lý tổng hai bình phương giới thiệu phần trước tài liệu, bạn tìm lại chứng minh Bài tốn 3.8.3. Ký hiệu d1(n) d3(n) lần lượt số thừa số nguyên tố đồng dư mod và đồng dư mod trong phân tích tắc số tự nhiên n, gọi số cách biểu diễn n

dưới dạng tổng hai bình phương, đó:

f(n) = (

0 nếu d1(n)≤d3(n) 4(d1(n)−d3(n)) nếu d1(n)> d3(n)

Ngoài loại số nguyên phức cịn có G− số, số có dạng xJ+yO x, y

là số nguyên vàJ, O bậc3của đơn vị âm J = +i

3 ,I =

1−i

3

2 Sử dụng loại số ta chứng minh định lýF ermattrong trường hợpn = 3, xác chứng minh định lý tập G− số Chứng minh chi tiết việc khảo sát tính chất G− số hẹn bạn dịp khác thích hợp

3.9 Phương trình Carmichael

Phương trìnhCarmichaellà phương trình nghiệm ngun có dạng x2+y2+z2 =t2 Phương trình có vơ hạn nghiệm ngun xác định đẳng thức:

d2(m2 −n2−p2 +q2)2+d2(2mn−2pq)2+d2(2mn+ 2pq)2 =d2(m2+n2+p2+q2)2.

Bài toán sau cho ta hiểu biết cấu trúc nghiệm phương trình này:

Bài tốn 3.9.1 (Định lý Carmichael). Bởi khơng có số phương chia nên trong số x, y, z phải có nhất số chia chết cho 2, giả sử y z Chứng minh rằng tất nghiệm phương trình Carmichael có dạng:

        

y= 2l, z = 2m x= l

2

+m2−n2 n t= l

2+m2+n2

n .

Trong đó n|l2+m2 0< n <l2+m2.

lời giải Thật vậy, giả sử y= 2l, z = 2m, lấy t > x, đặt u=tx >0 suy ra: =⇒x2 + 4l2+ 4m2 = (x+u)2 =⇒u2 = 4l2+ 4m2−2xu

=⇒u= 2n) =⇒n2 =l2+m2−nx=⇒x = l

+m2−n2 n

=⇒t = l

2+m2+n2

(46)

Chiều ngược lại dễ dàng kiểm tra

Vấn đề đặt tình hình x, y, z, t khơng cịn q thoải mái, xác hơn, cho biến thêm ràng buộc kết thay đổi nào, chẳng hạn toán sau:

Bài tốn 3.9.2. Giải phương trình nghiệm ngun dương x2−(a2+b2)·y4=

Theo biết tốn chưa có câu trả lời trọn vẹn, khảo sát trường hợp đặc biệt, khia=b= tốn có lời giải Nhưng trước hết cần điểm lại số hiểu biết phương trình dạng P ythagore

Bài tốn 3.9.3 (Phương trình P ythagore). Tất nghiệm nguyên thủy phương trình nghiệm nguyên dương x2 +y2 =z2 đều cho công thức (x =m2 −n2, y = 2mn, z =

m2 +n2) (khơng tính hốn vị), đó (m, n) = 1 m, n khác tính chẵn lẻ, m > n. Bài tốn 3.9.4. Chứng minh tập số nguyên không âm, có đẳng thức

x4+y4 =z2 thì hai số x, y có số bằng

Chứng minh toán 3.9.3đã quen thuộc với bạn, dựa vào kết toán sử dụng phương pháp xuống thang chứng minh tốn3.9.4khơng q khó khăn Một dạng tương tự toán 3.9.4 là:

Bài toán 3.9.5. Chứng minh tập số ngun khơng âm, có đẳng thức

x4−y4 =z2 thì y=

Lời giải tốn khơng có liên hệ cụ thể với toán trên, mà hệ trực tiếp định lý đẹp đẽ sau F ermat:

Bài toán 3.9.6. Chứng minh rằng y= nếu x, y, z, tlà số nguyên không âm thỏa mãn: (

x2+y2 =z2

x2−y2 =t2.

lời giải Nếuy >0, giả sử (x, y) = 1và z là nhỏ Ta có 2x2 =z2+t2 suy

ra z+t

2 ∈Z =⇒x =

z+t

2

+

zt

2

z+t

2 ,

z+t

2

= Từ toán 3.9.3 suy tồn số tự nhiên m, n nguyên tố cho:

  

zt

2 =m 2−

n2 z+t

2 = 2mn

hoặc

  

z+t

2 =m

n2 zt

2 = 2mn =⇒2y2 =z2 −t2 = 2.4mn(m2−n2) =⇒y2 = 4(m2−n2)mn, y = 2k

=⇒k2 = (m2−n2)mn.

(47)

3.10 MỘT SỐ BÀI TOÁN KHÁC 47 Mẫu thuẫn chứng tỏ y=

Các tốn phương trình dạng P ythagore đa dạng phong phú, trường hợp kết nho nhỏ lại có ích việc giải phương trìnhCarmichael

ở trường hợp đặc biệt sau đây:

Bài toán 3.9.7. Chứng minh phương trìnhx2−8y4 = 1chỉ có hai nghiệm ngun khơng âm là (x= 1, y = 0) (x= 3, y = 1)

lời giải Viết phương trình dạng x2 = (2y2)2 + (2y2)2 + Nếu x = 1 thì y = 0, xét

x 6= Theo định lý Carmichael (bài toán 3.9.1) suy l = m = y2 2l2 =n(n+ 1) Ta xét hai trường họp sau

Nếu n = 2α4 n+ = β4 suy β4 −2α4 = =⇒ β4 + (α2)4 = (α4 + 1)2 Theo toán 3.9.4 suy α = suy n= Loại

Nếu n = α4 n + = 2β4 suy α4 −2β4 = −1 =⇒ (β2)4 −α4 = (β4 −1)2 Theo toán 3.9.5 suy α=β = n= Vậy x= 3, y=

3.10 Một số toán khác

Bài toán 3.10.1. Giả sử p là số nguyên tố Chứng minh trong 2p−1 số ngun bất kì tồn tại p số có tổng bội số của p.

lời giải Ký hiệu S(X) = P

xX

x |X| số phần tử tập hợp X Giả sử phản

chứng tồn tập hợpS gồm 2p−1số ngun a1, a2, , a2p−1 có tính chất tổng p phần tử S bội số p Như ∀AS,|A| = p

S(A)6≡0 mod p từ ta suy ra(S(A))p−1≡ mod p:

=⇒S = X |A|=p

(S(A))p−1 ≡

2p−1

p

6≡ mod p.

Để mâu thuẫn ta chứng minh S≡0 mod p Thật vậy, biến đổi sau:

(S(A))p−1 = (ai1 +ai2 + +aip)

p−1

=X

α

(p−1)!

α1!α2! αp!

.aα1

i1a

α2

i2 a

αp

ip.

Các tổng lấy tất α= (α1, α2, , αp), αi ≥0, p

P

i=1

αi =p−1

=⇒S = X (ai1,ai2, ,aip)

X

α

(p−1)!

α1!α2! αp!

.aα1

i1a

α2

i2 a

αp

ip =

X

α

X (ai1,ai2, ,aip)

(p−1)!

α1!α2! αp!

.aα1

i1a

α2

i2 a

αp

ip.

Ta chứng minh với α= (α1, α2, , αp) cố định tổng sau bội số củap:

T = X

(ai1,ai2, ,aip)

1

i1 a

α2

i2 a

αp

ip =

X

v

1

v(1)a

α2

v(2) a

αp

(48)

Tổng lấy tất đơn ánh v:{1,2, , p} → {1,2, ,2p−1} Gọiαj1, αj2, , αjm

với mp−1 phần tử có giá trị dương số α1, α2, , αp Rõ ràng ta cần

quan tâm tới giá trị v(j1) = v1, v(j2) = v2, , v(jm) =vm Vì có

2p−1−m pm

đơn ánh nên có:

T =

2p−1−m pm

X

v

j1

v1 a

αj2

v2 a

αjm vm .

Mặt khác ta có:

2p−1−m pm

= (2p−1−m)! (pm)!.(p−1)!.

Biểu thức trước hết số nguyên, vì1≤mp−1 nên tử số nhân tử bội củap (chính p) cịn mẫu khơng biểu thức bội số p Vậy ta có T bội số p Đây điều ta cần chứng minh

Nếu thay số nguyên tốp số tự nhiên n kết tốn đúng, bạn chứng minh điều theo hai cách Trong cách sử dụng kết vừa chứng minh trên, cách thứ hai, bạn thực phép quy nạp trực ước số nguyên tố lớn số Tuy nhiên cách đơn giản đẹp đẽ có lẽ cách chứng minh dựa bổ đề nhỏ sau:

Bổ đề.Trong n số nguyên tồn số số có tổng chia hết chon

Chứng minh bổ đề đơn giản, bạn tự thực (bằng cách sử dụng nguyên lý Dirichlet) Công việc áp dụng bổ đề vào toán sau:

Bài toán 3.10.2. Chứng minh trong 2n−1 số nguyên tồn tại n số có tổng chia hết cho n.

lời giải Gọi m là số lớn thoả mãn tính chất cóm số đồng dư với theo modun n.

Nếumn cần chọn n số đồng dư với modun n, số có tổng chia hết cho

n Nếum ≤2thì tồn n số phân biệt theo modun n Với n lẻ ta chọn n số phân biệt theo modun n, với n chẵn ta chọnn−1số phân biệt modun n không chia hết cho n

Trong trường hợp cịn lại ta giả sử m số chia hết cho n, không ta đem tất 2n−1số trừ số dư chung modun n m số Xét2n−1−m số cịn lại, 2n−1−mnnên tồn k số (kn)có tổng chia hết chon (theo bổ đề) Nếu

k+mn hiển nhiên cần chọn k số nk số chia hết cho n Nếu k+m < n

thì2n−1−kmn, ta tiếp tục lập luận trình rõ ràng phải kết thúc ta chọn n số có tổng chia hết chon

Bài toán chứng minh xong

Bài toán 3.10.3. Chứng minh với số thực δ ∈ [0,1] và với mọi ε > ta có bất

đẳng thức:

ϕ(n)

nδ

(49)

3.10 MỘT SỐ BÀI TOÁN KHÁC 49

lời giải Ký hiệu pk là số nguyên tố thứ k Đặtak = 1−1/pk Do pk > k nên:

lim

k→+∞ak = 1.

Giả sử p1, p2, , pk tất số nguyên tố không vượt n đó: k

Y

i=1

pi

pi−1

=

k

Y

i=1

1 +

pi + p2 i +

> ln(n)

=⇒0< a1a2 ak <

1

ln(n)

=⇒

+∞ Y

k=1

ak = 0.

(3.22)

Với số thực δ∈(0,1) lấy k0 >

1−δ ta có: ak0 = 1−

1

pk0

>1−

k0

>1−(1−δ) =δ. (3.23)

Từ (3.22),(3.23) suy tồn số nguyên không âm n cho:

λ=ak0ak0+1 ak0+n > δ > ak0ak0+1 ak0+n+1 =⇒0< λδ < ak0ak0+1 ak0+nak0ak0+1 ak0+n+1

=ak0ak0+1 ak0+n+1

1

ak0+n+1

−1

< δ

ak0+n+1

−1

< ε.

Bất đẳng thức cuối k0 >1 +δ/ε Cuối để ý rằng:

λ= ϕ(pk0.pk0+1 pk0+n)

pk0.pk0+1 pk0+n

.

Bài toán chứng minh xong

Bài tốn tính chất kì lạ phi hàm Euler, hàm số này, toán sau đẹp đẽ khơng kém:

Bài tốn 3.10.4. Ký hiệu ϕ(n) lần lượt số số nguyên dương nhỏ hơn n và nguyên tố cùng với n π(n) là số số nguyên tố không vượt quá n Chứng minh với mọi số tự nhiên n >1 ta có:

ϕ(n)≥ π(n)

2 .

Bài toán 3.10.5. Cho số nguyên N và tập hợp A gồm hữu hạn số nguyên dương Chứng minh tồn tập số nguyên dương B chứa A sao cho:

Y

xB

x−X xB

(50)

lời giải Ta chứng minh kết hai cách.

Cách Đặt M = QxAx P = PxAx2 Ta tìm tập hợp C các số nguyên dương phân biệt cho:

M Y

xC

x=N +P +X

xC

x2.

Trước hết để thử lý luận chọn C gồm m số 1và k số Khi ta cóM.2k =N +P + 4k.

Chú ý lim

k→∞(M.2

k

4k) = +∞ nên tồn k0 cho M.2k −4k > N +P với

kk0 Với kk0, ta chọn số m = M.2k −4kNP Ta ln chọn nghiệm có dạng (1,1,1, ,1,2,2, 2) Thực quy tắc sinh nghiệm sau, (x1, x2, , xn)

là nghiệm (x1, x2, , xi−1, M x1x2 xi−1xi+1, , xnxi, xi+1, , xn) nghiệm Sử

dụng quy tắc sinh nghiệm ta có:

(1,1,1, ,1,2,2, ,2)−→(1,1,1, ,1,2,2, ,2, M.2k−1−2)

−→(1,1,1, ,1,2,2, ,2,2k−2(M.2k−1 −2)−2, M.2k−1−2) −→

Từ ta tạo nghiệm (x1, x2, , xn) mà thành phân đôi phân biệt

Bây ta chọn C = (x1, x2, , xn) B =AC Bài toán chứng minh

Cách Với tậpS ta ký hiệud(S) = Q

xS

x− P xS

x2

Chọn S0 ={1,2, , m}sao choSX Ta chọn m đủ lớn cho:

m!−1−2− m > N +P.

Xét dãy S0 = m!−1, xn+1 = m!x1x2 xn−1 Sk = {1,2, , m, x1, x2, , xk} Dễ thấy

d(Sk+1) =d(Sk)−1 từ tồn tạik cho d(Sk) =N +P Bài tốn lại chứng minh

Lời Bình Lời giải 1 của toán minh họa cho phương pháp gen trong phép giải

phương trình nghiệm nguyên Phương pháp gen quy tắc xây dựng nghiệm phát sinh từ nghiệm ban đầu nhờ phép gen hóa Từ việc tạo nghiệm ta nghiên cứu tính chất họ nghiệm dựa vào nghiệm sở Hai ứng dụng phương phápgenlà phương trìnhP ellvà phương trình M arkov(mà toán 3.10.5 thể biến thể phương trình dạng M arkov) Nhờ tốn ta có cách giải tốn sau vốn đề chọn đội tuyển Việt Nam dự thi toán quốc tế năm 2002, lời giải hệ sử dụng phương pháp trực tiếp cách

Bài toán 3.10.6. Chứng minh tồn số nguyênm0 sao cho với số nguyên mm0 luôn tồn tại m số nguyên dương phân biệt a1, a2, , am sao cho N là số phương với:

N =

m

Y

i=1

ai−4

X

i=1

a2i.

Bài toán 3.10.7. Cho k số tự nhiêna1 ≤ a2 ≤ akn thỏa mãn [ai, aj] > n với

mọi 1≤ijk Chứng minh rằng:

(i)

k

X

i=1

ai

<

2 (ii)

k

X

i=1

ai

<

(51)

3.10 MỘT SỐ BÀI TOÁN KHÁC 51

lời giải Rõ ràng câu(i) là hệ câu (ii), nên ta chứng minh câu (ii), nhiên

nếu đứng độc lập câu(i) thú vị

Có thể giả sử rằngak =nvì nếuak < nthì thaynchoak ta có kết sau

Đặt Sn = {1,2, , n} Với i = 1,2,3 đặt ri số thuộc vào tập S[n/i] không bội số aj Với m = 2,3, , n+ ta xét tập hợp sau:

Bm =

ai n

m < ain m−1

.

Ký hiệu bm số phần tử tập Bm Với số thuộc

n m,

n m−1

m−1 bội số thuộc vào Sn suy Bm có (m−1)bm bội số thuộc vào Sn

=⇒b2+ 2b3+ + (n−1)bn =nr1. (3.24) Tương tự ta có đẳng thức

b3+b4 + 2b5 + 2b6 + 3b7 + =

n

2

r2. (3.25)

b4+b5+b6+ 2b7+ 2b8+ 2b9+ =

n

3

r3. (3.26)

Từ đẳng thức(3.24,3.25,3.26) ta suy

S =X

ai

n +

2(b2−1)

n +

3b3

n +

≤ −1

n +

2b2+ 3b3+

n

=−1

n +

2(nr1)

n

b3+ 2b4+ 3b5+, , ,

n

≤ 2−

n

2r1

n

[n/2]−r2+ [n/3]−r3

n

= 2−

n +

r2+r3−2r1

n

[n/2] + [n/3]

n

≤ 2−

n

5 6+

1/2 + 1/3

n =

7 +

1 6n

6 5. Đánh giá cuối với n≥5 đủ để hoàn thành chứng minh

(52)(53)

Chương 4

Tổng nghịch đảo

Trần Mạnh Tuấn & Trần Quốc Hoàn

Giới thiệu. Với số nguyên dương n tồn số tự nhiên an bn nguyên tố cùng

nhau thoả mãn đẳng thức:

S(n) = +

2+ +

n = an

bn

.

Vấn đề đặt khảo sát tính chất số {an} {bn} Một cách cụ thể mối quan

hệ số với số nguyên tố p cho trước Đây câu hỏi hay rất khó! Một trường hợp riêng (quan trọng), với n như thì an chia hết cho p Theo

chúng tơi biết tốn chưa có lời giải Bài viết nhỏ thực hiện vài khảo sát số trường hợp đặc biệt.

Để cho thống nhất, từ khơng có đặc biệt quy ước p dùng để số nguyên tố với hai phân số tối giản a/b, c/dac mod p bd 6= mod pthì ta viết a

bc

d mod p Nếu achia hết cho số ngun xkhác0thì ta nói a/bchia hết cho x Các ký hiệu đồng dư sử dụng theo nghĩa

Chúng ta bắt đầu với vài kết nhỏ quen thuộc

Mệnh đề Chứng minh ta có hai tính chất sau với số nguyên tố p≥5:

(i) ap−1 chia hết cho p2.

(ii) ap2−p ap2−1 chia hết cho p.

lời giải Sử dụng phép ghép cặp để chứng minh(i) và sử dụng (i) để chứng minh(ii).

(i)

Ta có S(p−1) =

p−1 X

k=1

k = p−1

2 X

j=1

1

j +

1

pj

=p·

p−1 X

j=1

1

(54)

Nhận xét với mỗij ∈ {1,2, ,p−1

2 }đều tồn xj

1,2, ,p−1

2

sao cho(jxj)2 ≡

1 mod p với1≤jp−1, ta có:

{x1, x2, , x p−1

} ≡ {1,2, ,p−1

2 }.

Sử dụng nhận xét ta được:

p−1 X

j=1

j(pj) ≡

p−1 X

j=1

(−x2j) = 1−p

2 ·

p+ ·p

6 ≡0 mod p. (4.2)

Đồng dư cuối vớip≥5 Kết hợp(4.1),(4.2)suy p2|ap−1 Điều phải chứng minh

(ii)

S(p2 −p) =

p−2 X

k=0

1

kp+ +

kp+ + +

kp+p−1

+1

p ·S(p−1).

Một hệ câu (i) làp|ap−1 ta có:     

kp+ +

kp+ + +

kp+p−1 ≡ 1+

1

2+ +

p−1 ≡0 mod p

p2|S(p−1) =⇒

p ·S(p−1) ≡0 mod p.

VậyS(p2−p)≡0 mod p Tiếp tục biến đổi:

S(p2−1)−S(p2−p) =

p2−p+ 1 +

p2−p+ 2 + +

1

p2−p+p−1

1 +

2 + +

p−1 ≡0 mod p.

Mệnh đề Giả sử k là số nguyên không âm Chứng minh rằng an không chia hết cho số

nguyên tố lẻ p với số tự nhiên n∈[pk,2pk −1].

lời giải Trước hết ta phát biểu chứng minh nhận xét:

Nhận xét. Với p là số nguyên tố lẻ và n là số nguyên dương, nếu an không chia hết

cho p thì anp, anp+1, , anp+p−1 đều không chia hết cho p.

Chứng minh. Thật vậy, cách ghép cặp tương tự sử dụng mệnh đề1 ta có với mọim nguyên khơng âm số ngun tố lẻp thì:

1

mp+ +

mp+ + +

1

(55)

55

chia hết cho p Do với k = 0,1,2, , p−1 ta viết:

anp+k

bnp+k

= pa

b + an

pbn

+ c

d

trong a/bc/d phân số tối giản b, ckhông chia hết chop Từ đẳng thức ta dễ dàng suy an khơng chia hết cho p với k= 0,1,2, , p−1 ta có anp+k

khơng chia hết cho p Nhận xét chứng minh

Bây ta chứng minh mệnh đề Do a1 không chia hết cho p, theo nhận xét suy

ap, ap+1, , a2p−1 không chia hết cho p Tương tự ta có ap2, , a2p2−1 khơng chia hết cho p Tiếp tục trình cách sử dụng nhận xét ta thu apk, , a2pk−1

không chia hết cho p Đây điều phải chứng minh

Sử dụng mệnh đề2 với bổ đề Dirichlet ta có kết thú vị sau:

Mệnh đề Cho trước số nguyên dương N T Chứng minh tồn tại m nguyên dương cho với k =m, m+ 1, , m+T ta có ak N nguyên tố nhau.

Với số nguyên tố p tuỳ ý, kết thu hạn chế Việc xét số trường hợp riêng p dễ dàng Thật vậy, với p= 2, p= vàp= toán giải trọn vẹn định lý

Định lý. Tử số an chia hết cho nếu nếu n = hoặc n = Và bn không chia

hết cho 5khi khi n nhận giá trị:

1,2,3,4,20,21,22,23,24,100,101,102,103,104,121,122,123,124.

lời giải Ta chứng minh lần lượt 2nội dung nhắc đến định lý.

(i) Ta cần sử dụng nhận xét sau đây:

Nhận xét. Nếu an chia hết cho thì a[n/3] cũng chia hết cho

Chứng minh.Thật vậy, đặt k = [n/3], ta có đẳng thức:

a3k

b3k

=

k

X

i=1

1 3i+ +

1 3i+

+

k

X

i=1 3i =

ak

3bk

+

k

X

i=1

6i+ (3i+ 1)(3i+ 2).

Lập đẳng thức tương tự cho số 3k+ 1,3k+ suy nhận xét chứng minh

Bây ý hệ nhận xét a[n/3] không chia hết cho an

(56)

Vớin= 21hoặcn= 23thìankhơng chia hết cho3, ta có

an

bn

= a7 3b7

+3·c

d =

121 140+3·

c d

trong c

d phân số tối giản có mẫu số d không chia hết cho

Với n = 22, ta có a22

b22

= 121 140 + ·

c d +

1 22 =

1401 1540 + ·

c

d nên a22

cũng không chia hết cho3

Vậyan chia hết cho3 n= n=

(ii)

S0(n) =

n

X

i=1

k

[n/5] X

k=i

1 5i.

Ta quy ước A dạng phân số tối giản có mẫu số khơng chia hết cho B dạng phân số tối giản có mẫu số chia hết cho 5, C dạng phân số tối giản có mẫu số tử số không chia hết cho 5và D dạng phân số tối giản có tử số khơng chia hết cho Các đẳng thức mang tính chất tượng trưng A+A= 25A có nghĩa tổng hai phân số dạng A 25 lần phân số dạng A

Ta có S(1) = 1, S(2) =

2, S(3) = 11

6 S(4) = 25

12 Chú ý rằng:

5k+ 1+ 5k+ 2+

1 5k+ 3+

1

5k+ = (10k+5)

1

(5k+ 1)(5k+ 4)+

1

(5k+ 2)(5k + 3)

= 25A.

Do S0(n) = 25A Với số tự nhiênn chia hết cho5hoặc chia 5 dư4 Ta xét trường hợp sau đây:

Vớin = 5,6, ,19 vìS([n/5])∈ {S(1), S(2), S(3)} nên:

S(n) =S0(n) + 5·S

n

5

=A+1

5C =B.

Vớin = 20,21,22,23,24 ta cóS

n

5

=S(4) = 25 12

=⇒S(n) =S0(n) +

5S(4) =S

(n) + 12 =A.

Vớin = 25,26, ,99 ta có S

n

5

=C

=⇒S(n) =S0(n) + 5S n + 25S n 25

=A+1 5A+

1

25C =B. Vớin = 100,101,102,103,104 thì:

S(n) =S0(n) + 5S

0

(20) + 25S

0

(57)

57

Với n= 105,106, ,119 ta có:

S(n) =S0(n) + 5S n + 12. Mà 5.S n

∈ {S0(21), S0(22), S0(23)} và S0(21) = S0(20) +

21 = 25A+

21 = C, tương tựS0(22), S0(23) đều dạng C Vậy ta có S(n) =A+

5C+ 12 =B Với n= 120,121,122,123,124 ta có S(n) =S0(n) +

5S

0(24) + 12 =A Với n≥125 Gọik số nguyên dương nhỏ cho 5kn Khi đó:

S(n) = X 5k−2|x

1

x +

X 5k−2không chia hếtx

1

x =

1

5k−2S(u) + 5k−3A.

Trong 25 ≤ u ≤ 124 Kiểm tra với u = 100,101,102,103,104,120,121,122,123,124 ta thấy phân số:

1 5k+ +

1 12,

1 5k+ +

1 5k+ +

1 12.

1 5k+ +

1 5k+ +

1 5k+ +

1 12

đều có tử sô không chia hết cho Vậy với giá trị uta cóS0(u) = 5A Suy ra:

S(u) =S0(u) + 5S u +

12 = 5A+ 5A+D =D =⇒S(n) =

D

5k−2 +

A

5k−3 =B. Trong phân số dạng D có tử số khơng chia hết cho

Vậy giá trị cần tìm n là1,2,3,4,20,21,22,23,24,100,101,102,103,104,121,122,123 124 Định lý chứng minh hoàn toàn

Chứng minh chứng minh trọn vẹn, có lẽ chưa thể cốt lõi vấn đề Bằng chứng bạn cho chứng minh sử dụng nhiều kỹ thuật Theo chúng tôi, định lý sau giúp ích bạn độc giả nhìn nhận kết cách sâu sắc

Định lý.Giả sử p số nguyên tố lẻ thoả mãn hai điều kiện:

(i) p−1 số nguyên dương nhỏ số r thoả mãn p|ar

(ii)

p2S(p−1) +S(r) có tử số khơng chia hết cho p với1≤rp−1 Khi bn khơng chia hết cho p nếu:

n

(

(58)

Rõ ràng p|an bn khơng chia hết cho p Do n phải số nêu

trên, hệ an không chia hết cho p np3 Nhận xét số nguyên tố

3,5,7,13,17,19,23,29 có tính chất trên, câu hỏi với trường hợp p = 11 sao, có điều khác biệt Câu hỏi xin dành cho bạn đọc suy nghĩ thêm

Để kết thúc chuyên đề mời bạn thưởng thức toán tuyệt đẹp sau đây, toán hay theo chúng tơi khó Tuy nhiên cách sử dụng số kết trình bày trên, chuyện vô sáng vơ rõ ràng

Bài tốn. Chứng minh rằng an không phải luỹ thừa số nguyên tố với vô hạn giá trị

tự nhiên của n.

lời giải Bổ đề an> n+ 1 với mọi n= 4,5,6,

Chứng minh Gọik số tự nhiên thoả mãn2kn <2k+1 Bằng phép quy đồng mẫu số tổng +1

2 + +

n suy

k|

bn an số lẻ (điều nói lên mệnh

đề2 trường hợp p= 2) Suy rabn≥2k >

n

2 Ta có với n≥4 thì:

an =bn n

X

i=1

in+

2 X

i=1

i >2 n+

2 =n+ 1.

Ta chứng minh với số nguyên tố lẻp tuỳ ý dãy sốn =p, p2−p, p2−1, p3−

p, p3−1, , pp−1có số có tính chất an khơng phải luỹ thừa số nguyên

tố Giả sử phản chứng Theo mệnh dề1ta cóS(p−1) chia hết chop Ta dùng quy nạp theo

np để chứng minh S(pnp) S(pn−1) chia hết cho p Thật vậy, theo bổ đề

apn−1 > pn apn−1 chia hết chop (mệnh đề 1), mà apn−1 luỹ thừa số nguyên tố

=⇒ap2−1 chia hết cho p2 (4.3) Ta có đẳng thức sau:

S(pn+1 −1) =pn+1· X

1≤k≤pn+1−1

pkhông chia hếtk

1

k(pn+1−k)+

pS(p

n

1). (4.4)

S(pn+1 −p) = X (k,p)=1

1

k +

1

pS(p

n

−1). (4.5)

Kết hợp kết (4.3),(4.4),(4.5) ta suy kết luận quy nạp Sử dụng vào tốn ta có: 

 

pn|S(pn−1) vớin= 1,2, pn|apn−p vớin= 2,3,

S(pn−1)−S(pnp) +S(p−1) =pn

pP−1

i=1

p(pni) chia hết chop n

Từ suy raS(p−1) chia hết chopn với mọinS(p−1) = pP−1

i=1

i < pbp−1 ≤(p−1)!

(59)

Phần II

Một số chủ đề Tổ Hợp

(60)(61)

Chương 5

Bổ đề Sperner

Nguyễn Quốc Khánh

Giới thiệu. Trong Toán Học thường xuất kết có cách phát biểu giản dị, lại tỏ quan trọng nhiều lĩnh vực sâu sắc Trong viết này, muốn ghi chép lại số hiểu biết kết thế, tổ họp, người ta gọi kết bổ để Sperner, ứng dụng quan trọng tham gia vào chứng minh tổ hợp - giải tích định lý điểm bất động Brower.

Bổ đề Sperner. Các đỉnh tam giác đánh số chữ số 0,1,2 Tam giác chia thành số tam giác cho khơng có đỉnh tam giác nằm cạnh tam giác khác Các đỉnh tam giác ban đầu giữ nguyên số cũ đỉnh đánh số 0,1,2 cho đỉnh nằm cạnh tam giác ban đầu đánh số số dùng để đánh số đỉnh cạnh Chứng minh tồn tam giác nhỏ đánh số 0,1,2

Chứng minh.Xét đoạn thẳng phân từ cạnh 01, gọia số đoạn thẳng dạng00,

b số đoạn thẳng dạng 01 Đối với đoạn thẳng xét số số đứng đầu nó, cộng tất số ta 2a+b Mặt khác, tất số nằm tính hai lần, thêm số0đứng đỉnh tam giác ban đầu Do số2a+blẻ, suy rablà số lẻ

(62)

Bây ta xét đến phép chia tam giác Giả sử a1 tổng số tam giác dạng 001 011, cònb1 số tam giác dạng 012 Đối với tam giác xét số cạnh có dạng 01 cộng tất số lại Tổng cộng ta 2a1 +b1 Mặc khác tất cạnh nằm tính hai lần tổng đó, cịn tất cạnh biên nằm cạnh 01 tam giác ban đầu số lẻ theo lập luận Do 2a1+b1 số lẻ, suy b1 số lẻ điều phải chứng minh

Bổ đề Sperner rõ ràng đơn giản, chứng minh dễ dàng Vậy đâu ý nghĩa sâu sắc Để chuẩn bị cho việc trình bày chứng minh tổ hợp - giải tích định lý điểm bất động Brower dựa bổ đề Sperner làm quen vói vài khái niệm tơpơ đại cương Đó khái niệm ánh xạ liên tục ánh xạ đồng phơi Hãy tưởng tượng có bóng, làm bẹp Khi dó nói dùng ánh xạ liên tục tác động vào bóng Hãy ý qua ánh xạ đoạn nối hai điểm M, N bề mặt bóng khơng bị cắt đứt, "liên tục" lúc Nhưng bạn lại lấy kim để chọc thủng bóng sao, ánh xạ, liệu có liên tục hay khơng Câu trả lời thực không (nhưng để chứng minh điều khơng đơn giản) Vậy có ánh xạ khơng liên tục, lấy ví dụ khác rõ ràng Các bạn xét sợi dây, bạn thắt nút vịng cách tuỳ ý rõ ràng ánh xạ liên tục, nhiên bạn lấy kéo để cắt đôi sợi dây rõ ràng ánh xạ không liên tục Với hai vật thể A B khơng gian, có ánh xạ liên tục f biến A

thành B ta nói A B đồng phơi với Chẳng hạn bánh qui hình trịn bánh qui hình vng đương nhiên đồng phơi với Tất nhiên, nói ngược lại hai vật A, B không đồng phôi với dùng ánh xạ liên tục để biến từ vật thành vật Những vật thể nhiều, chẳng hạn hình cầu hình cầu khác sau đục thủng lỗ không đồng phôi với nhau:

(63)

5.1 BAO LỒI 63

Định lý điểm bất động Brower. Mọi ánh xạ liên tục từ hình cầu đặc vào ln có điểm bất động

Chúng ta tìm cách chứng minh định lý dựa bổ đề Sperner, lưu ý có nhiều cách chứng minh dành cho định lý điểm bất động nhiều định lý điểm bất động khác

5.1 Bao lồi

Ta nói hình S khơng gian lồi với A, BS với

C ∈[AB] ta có CS Một điểm B mà hình cầu tâm B bán kính nhỏ tuỳ ý chứa điểm thuộcS điểm không thuộc S gọi điểm biên S.S gọi tập đóng chứa điểm biên

Xét tập hợp điểm U không gian U = {u0, u1, , un} Nếu S hình lồi nhỏ

nhất chứa U S gọi bao lồi U viết S =co(uo, u1, , un) ("co" viết tắt cho

chữ "convex" tiếng Anh có nghĩa lồi)

Nếu S tứ diện UoU1U2U3 U0, U1, U2, U3 dược gọi đỉnh tứ diện bao lồi k+ đỉnh tứ diện gọi làk diện tứ diện

Phép phân chia tứ diện S thành tứ diện Si với i = 1,2, , n cho hợp chúng

bằng S hai tứ diện giao phần giao phải diện chung chúng gọi phép tứ diện phân Để cho đơn giản ta hiểu phép tứ diện phân thực theo cách chia tứ diện thành 8tứ diện tích nhau:

Bây đỉnh V tứ diện gọi co{ua, ub, ,}là diện nhỏ chứa V

thìvđược gán số a, b, , phép gán số gọi phép gán số Sperner (dễ thấy đỉnh Ui tứ diện gán sối) Ta gọi tứ diện tốt

(64)

Bổ đề Sperner không gian. Tồn tứ diện tốt phép tứ diện phân

5.2 Bổ đề KKM

Bổ đề KKM (Knaster - Kuratowski - Mazurkiewics - 1929). Cho trước tứ diện

S = UoU1U2U3 tập đóng Fo, F1, F2, F3 S thoả mãn điều kiện KKM: với tập I tập 4số nguyên không âm {0,1,2,3} ta cóco{Ui|iI} ⊂ ∩iIFi, đó:

\

iI

Fi 6=φ.

Trước chứng minh bổ đề nhắc lại vài kiến thức giải tích Trước tiên, nhờ nguyên lý Cantor cho đoạn lồng ta chứng minh từ dãy số thực bị chặn ln trích dãy hội tụ nhờ tính chất ta chứng minh họ điểm vơ hạn tứ diện ln trích họ điểm hội tụ Bây chứng minh bổ đềKKM Thực phép tứ diện phân S thực phép gán số sau: lấy đỉnh V tứ diện con, gọi diện nhỏ S

chứa Vco{Ui|iI}, theo điều kiện KKM suy V ∈ ∩iIFi tồn m

VFm, ta gánm choV viếtVm Vậy sau gán xong thìUi phải gán sối Cách

gán thoả mãn điều kiệnSpecnerdo tồn tứ diện tốt ∆1(V1 0, , V

1 3)vàV

1

iFi,

i= 0,3 Coi∆1 đóng vai trị như S lại tiếp tục q trình phân chia, q trình thực vơ hạn lần Và ta nhận vô hạn tứ diện tốt lồng ∆miFi, i= 0,1,2,3

Ta trích từ dãy điểm (Vm

0 ) dãy (V

mi

0 ) hội tụ đến V0 ∈ F0, sau lại trích từ dãy (Vmi

0 ) dãy (V

mi,k

0 ) hội tụ đến V1 ∈ F1 Sau bốn lần trích dãy ta nhận dãy (Vmj

4 )hội tụ đến V4 ∈F4 Và ta nhận dãy đơn hình tốt(∆mj) dãy(∆m) mà dãy đỉnh tương ứng hội tụ tới điểm Mặt khác, biết thể tích cảu dãy tứ diện tiến tới0và tất 4điểm hội tụ trùng Tức giao tập đóng Fi khác rỗng Bổ đề chứng minh

5.3 Chứng minh định lý điểm bất động Brower

Giả sử ϕ:TT0là ánh xạ đồng phôi, ánh xạ liên tụcf :T0→T0có điểm bất động ϕ−1 ◦fϕ :TT cũng có điểm bất động Do thay xét hình cầu ta có thể chứng minh định lý cho tứ diện đặc Khi với điểmX nằm tứ diệnS tồn số xi không âm mà tổng chúng 1, nữaOX~ =P3i=0xiOX~ i, ta viết

X = (x0, x1, x2, x3) Bây ta xét ánh xạ liên tụcf :SS, xét XS,X = (x0, x1, x2, x3) f(X) = Y = (y0, y1, y2, y3) Ký hiệu Fi tập hợp điểm X thuộc tứ diện S

xiyi Dễ thấy họFi thoả mãn điều kiện KKM tồn X ∈ ∩Fi, tức xiyi với

mọi i= 0,1,2,3 Nhưng

3 X

i=0

xi = =

3 X

i=0

yi

(65)

Chương 6

Các đề toán tổ hợp chọn lọc

Bài toán 6.1. Có cách từ điểm (0,0) đến điểm (p, q) trên mặt phẳng toạ độ nguyên, bước từ điểm (x, y)đến điểm (x+ 1, y) hoặc (x, y+ 1) sao cho con đường khơng cắt đường thẳng x=y.

Bài tốn 6.2. Cho n là số nguyên dương Tính số cách phân hoạch tập hợp n số nguyên dương đầu tiên Sn ={1,2, , n} thành tập hợp con A, B, C thoả mãn tính chất |AB|>

0,|BC|>0,|CA|>0 |ABC|=

Bài toán 6.3. Cho số nguyên dương n > Trong không gian cho hệ trục toạ độ Y Z. Ký hiệu T là tập hợp gồm tất điểm P(z, y, z) x, y, z là số nguyên thoả mãnx, y, zn Ta tô màu điểm thuộc T sao cho nếu A(x0, y0, z0 được tơ tất các điểm B(x1, y1, z1) x1 ≤ x0, y1 ≤ y0, z1 ≤ z0 (các đẳng thức không đồng thời xảy ra) đều không tô màu Hỏi ta tơ màu tối đa điểm.

Bài tốn 6.4. Cho tập hợp X 56 phần tử Tìm giá trị nhỏ của n sao cho với mỗi 15 tập của X, số phần tử hợp mỗi7 tập chúng không nhỏ hơn n thì tồn tại trong chúng có giao khác rỗng.

Bài tốn 6.5. Cho bảng vuông n.n, ô vuông bảng điền số thực dương sao cho với hàng cột có tổng số nằm là Chứng minh tồn tại n ô điền số mà không nằm hàng hay cột.

Hệ Cho bảng vng2006.2006, có số ô vuông bảng đáng dấu cho mỗi

hàng cột có đúng ơ đánh dấu Chứng minh ta tơ ô đánh dấu bằng màu cho không có ơ màu nằm hàng hay cột. Bài toán 6.6. Cho số nguyên dương n X là tập hợp có n2 + 1 số nguyên dương cho trong tập con n + phần tử của X đều có hai phần tử phân biệt x, y thoả mãn x|y. Chứng minh có tồn tập con {x1, x2, , xn+1} của X có tính chấtxi|xi+1 với mọi 1≤in.

Bài tốn 6.7. Cho 4n điểm đường tơ màu xanh đỏ xen kẽ Biết khơng có đoạn thẳng nối điểm trong4n điểm đồng quy Nối 2n điểm xanh thành n đoạn thẳng, nối 2n điểm đỏ thành n đoạn thẳng Ta đánh dấu điểm giao điểm đường thẳng có hai đầu xanh đường thẳng có hai đầu đỏ Hỏi ta đánh dấu bao nhiêu điểm.

(66)

Bài tốn 6.8. Có ếch đỉnh của 2n giác đều (n >1) Tại thời điểm tất cả ếch nhảy đến đỉnh kề lúc (có thể có nhiều ếch nhảy đến cùng đỉnh), gọi cách nhảy Biết tồn cách nhảy cho đường thẳng chứa cặp đỉnh phân biệt có ếch sau nhảy, không qua tâm của đa giác Tìm tất giá trị của n.

Bài toán 6.9. Cho tập hợp S ={1,2, , n} P = {P1, P2, , Pn} là tập hợp tập

con có phần tử của S thoả mãn điều kiện |PiPj| = nếu nếu (i, j) ∈ P Chứng

minh phần tử của S thuộc đúng phần tử của P.

Bài toán 6.10. Cho n viên sỏi và người A, B chơi trò chơi sau Đầu tiên A lấy

k viên sỏi với 1≤kn−1 sau đó B lấy t viên cho 1≤tk Cứ hết, người lấy viên sỏi cuối người chiến thắng Hỏi A có chiến lược ln thắng hay khơng.

Bài tốn 6.11. Một tứ giác cạnh bị phủ kín bởi 6đường trịn bán kính R Chứng minh rằng R

3/10

Bài toán 6.12. Trong lớp học bạn nam quen với bạn nữ Chứng minh rằng chọn nhóm gồm nhiều nửa số thành viên lớp mà bạn nam quen với số lẻ bạn nữ nhóm.

Bài tốn 6.13. Trong câu lạc có 42 thành viên Biết cứ 31 thành viên bất kỳ thì có đơi nam nữ quen Chứng minh từ thành viên câu lạc có thể chọn ra 12 đôi nam nữ quen nhau.

Bài toán 6.14. Cho Sn ={1,2, , n} giả sử A1, A2, , An n tập khác rỗng của Sn.

Đặt k= [n/2] + Chứng minh tồn k tập Ai1, Ai2, , Aik trong tậpAi mà tồn tại

hai tập hợp con X của Sn để |Aij∩X| là số lẻ với mọi 1≤jk.

Bài tốn 6.15. Cho bảng vng n.n gồm số nguyên không âm aij Biết với mọi

i, j thì tổng tất số hàng i và cột j đều không nhỏ hơn n Chứng minh rằng: X

i,j

aij

n2 2.

Bài toán 6.16. Cho đồ thị lưỡng phân với tập đỉnh A1, A2, , An B1, B2, , Bn Biết

Ai nối với Bi với mọi 1≤in Ai nối với Bj nếu nếu Aj nối vớiBi Chứng minh

rằng tồn tập điểm:

S ={Ai1, Ai2, , Aik}

sao cho với số Bi số cạnh nối Bi với đỉnh thuộc S là lẻ.

Bài tốn 6.17. Có tồn hay khơng n >2 điểm mặt phẳng cho khơng có ba điểm nào thẳng hàng tâm đường tròn ngoại tiếp tam giác có đỉnh điểm đó trong n điểm đó.

(67)

67

Bài toán 6.19. Trên bảng ban đầu có n số (n ≥2) Cứ sau lần ta lấy số tuỳ ý a, b

và thay chúng số a+b

4 Sau n−1 lần bảng cịn lại số Tìm giá trị lớn nhất, nhỏ số đó.

Bài tốn 6.20. Cho trước số ngun dương N Hai người A, B chơi trò chơi sau. Bắt đầu từ người A viết sốN lên bảng, sau người viết sốm thì người sau viết số m−1 hoặc [m/2] Ai viết số trước thắng Hỏi người thắng cuộc, sao?

Bài toán 6.21. Trong thi hoa hậu, giám khảo đề nghị 10 thí sinh vào vịng chung khảo Một nhóm thí sinh gọi chấp nhận với giám khảo A nếu trong nhóm có thí sinh do A đề nghị Biết cứ giám khảo có thí sinh là nhóm chấp nhận với cả Chứng minh chọn 10 thí sinh nhóm chấp nhận với tất thành viên ban giám giảo.

Bài toán 6.22. 101 thành phố Biết hai thành phố có đường bay một chiều khơng có đường bay cả.

i) Biết thành phố có 50 đường bay đến, 50 đường bay Chứng minh với thành phố bất kì A B ta bay từ A đến B mà phải dừng nhiều nhất 1thành phố C.

ii) Biết thành phố có 40 đường bay đến, 40 đường bay Chứng minh với thành phố bất kỳ A B ta bay từ A đến B mà phải dừng nhiều nhất thành phố C1, C2

Bài toán 6.23. Cho trước số nguyên dương n ≥ 12 và tập hợp X n phần tử. F một họ gồm tập con phần tử của X sao cho giao cặp tập phân biệt trong

F có nhiều nhất phần tử Chứng minh có tập con S của X chứa nhất √3 6n−6 phần tử cho khơng có tập con phần tử của S nằm họ F.

Bài toán 6.24. Cho số nguyên dương chẵn n A1, A2, , An n tập tập S = {1,2, , n}sao cho iAiiS iAj nếu nếu jAi với i6=j Chứng minh rằng

tồn tại i6=j |AiAj| là số chẵn.

Bài toán 6.25. Trong phịng có 2005 cái hộp, hộp chứa vài loại trái cây táo chuối nho Dĩ nhiên số trái nguyên Chứng minh tìm được 669 hộp cho tồn chúng chứa nhất 1/3 của tất số táo nhất 1/3 tất số chuối Liệu có phải ln ln tìm được 669 hộp cho tồn chúng chứa nhất 1/3 tất cả số táo, nhất 1/3 tất số chuối nhất 1/3 tất số nho.

Bài toán 6.26. Ta gọi số x= (x1.x2, , xn) là một vecto trong không gian n chiều Với

hai vecto trong không gian n chiều là x = (x1, x2, , xn) y = (y1, y2, , yn) ta ký hiệu

xy =

n

P

i=1

xiyi là tích vơ hướng của vecto x, y Giả sử rằng f(n) là số lớn mà tồn tại

f(n) vecto khác mà tích vơ hướng của vecto bất kì khơng dương Chứng minh rằng

(68)

Bài toán 6.27. Cho tập hợp A n phần tử và n tập nhiều hơn phần tử là

A1, A2, , An Giả sử với tập con 2p phần tử A0 ∈ A có tập con Ai

của A0 Chứng minh với 2 tập trong n tập ban đầu có chung phần tử.

Bài toán 6.28. Cho n là số nguyên dương tập hợp số sau:

Sn ={(a1, a2, , an)|ai ∈[0,1],i= 1, n}.

Với hai phần tửa= (a1, a2, , a2n), b = (b1, b2, , b2n)∈Sn Định nghĩa khoảng cáchd(a, b) =

2n

P

i=1

|aibi| Chúng ta gọi tập hợp con ASn là tốt nếu d(a, b)≥2n−1 thoả mãn với cặp

phần tử phân biệta, b của A Hỏi tập tốt của Sn có thể có nhiều phần

tử.

Bài toán 6.29. Giả sử tập hợp A ∈ {(a1, a2, , an)|aiR,i= 1, n} Định nghĩa hàm

khoảng cách sau: với mỗi a= (a1, a2, , a2n), b = (b1, b2, , b2n)∈A đặt:

γ(a, b) = (|a1−b1|,|a2−b2|, ,|anbn|).

Xét tập hợp D(A) ={γ(a, b)|a, bA} Chứng minh rằng |D(A)| ≥ |A|.

Bài toán 6.30. Cho tập hợp gồm k dãy nhị phân đôi khác có độ dài lần lượt n1, n2, , nk Giả sử không tồn dãy nhị phân 0,1 nào mà ta biểu diễn bằng

cách đặt liên tiếp số n1, n2, , nk (không thiết khác nhau) hai cách khác nhau.

Chứng minh rằng:

1 2n1 +

1

2n2 + + 2nk

≤1.

Bài tốn 6.31. Cho bảng vng n.n với n > Hãy tìm tất cách tơ màu hai màu đen trắng cho khơng có hai ô đen kề ô trắng kề với ít nhất hai đen.

Bài tốn 6.32. Chứng minh khơng thể có nhiều hơn 4096 cây nhị phân độ dài 24 sao cho cây chúng có nhất vị trí khác nhau.

Bài tốn 6.33. Trong buổi tiệc có 2n người gồm nam nữ Họ ngồi cái bàn trịn Hãy tìm tất cả n sao cho với cách ngồi ta chia họ thành n cặp nam nữ mà hai người cặp không ngồi cạnh nhau.

Bài tốn 6.34. Cho tập hợp hữu hạn M gồm hai số thực dương khác Biết rằng với bất kỳaM tồn sốb, cM (a, b, c không thiết phân biệt) cho a= +b

c.

Chứng minh tồn hai số x, yM (x6=y) sao cho x+y >4

Bài tốn 6.35. Cho số ngun dương n >2 Hãy tìm số số nguyên a thoả mãn điều kiện tồn song ánh f :{1,2, , n} → {1,2, , n} |f(i)−i|=ai= 1.n.

(69)

69

Bài toán 6.37. Cho số nguyên dương n, k thoả mãn n = 2k −1 k ≥ Giả sử

T ={x = (x1, x2, , xn)|xi ∈ [0,1],i= 1, n} Định nghĩa hàm khoảng cách d(x, y) là số các

chỉ số j sao choxj 6=yi Giả sử tồn tập con S của T với 2k phần tử thoả mãn với mỗi

phần tử xT tồn phần tử yS sao cho d(x, y)≤3 Chứng minh n= 23 Bài toán 6.38. Cho số nguyên dương n, k Trong mặt phẳng n đường trịn bố trí sao cho hai đường tròn tuỳ ý cắt tại điểm phân biệt khơng có ba đường trịn nào cùng qua điểm Các giao điểm tô bởi trong n màu, màu dùng nhất một lần, đường trịn có đúng k màu Tínhk, n để việc tơ màu thực được.

Bài toán 6.39. Với cặp số khác nhau (x, y) của tập hợp hữu hạn phần tử X ta gán cho nó số là f(x, y) bằng hoặc sao cho f(x, y) 6= f(y, x) ∀x 6= y Chứng minh có đúng tính chất sau đạt được:

i) X là hợp của tập rời khác rỗng U, V sao cho f(u, v) = ∀uU, vV.

ii) Các phần tử của X có thể gán cho x1, x2, , xn thoả mãn f(x1, x2) = f(x2, x3) = =

f(xn, x1)

Bài toán 6.40. Giả sử đường trịn cón ơ (n≥3) Mỗi viết hai ký hiệu 0,1 Một phép toán thực theo luật sau Chọn C nào có ký hiệu 1, biến đổi thỳanh và biến đổi ký hiệu x, y trong hai ô kề với ô C thành 1−x,1−y. Trạng thái ban đầu có mang ký hiệu cịn khác mang ký hiệu tìm giá trị

n msao cho sau số hữu hạn bước thực phép tốn ta đưa ký hiệu trên các tồn là

Bài tốn 6.41. Từ định nghĩa số có 10 chữ số gồm số 0,1 Một phép biến đổi từ chọn số chữ số liên tiếp từ cho tổng chúng số chẵn dảo ngược số Hai từ gọi đồng nghĩa sau số dùng phép biến đổi từ biến thành từ Tìm số lớn từ đơi khơng đồng nghĩa. Bài tốn 6.42. Cho bảng bng n.n Trên vng bảng có ghi số nguyên dương cho hiệu hai số ghi hai ô kề là 1hoặc −1 Chứng minh ta có thể tìm số nguyên dương k mà tất cột chứa k hoặc tất hàng chứa

k.

Bài tốn 6.43. Cho bảng vng 2n.2n với n là số nguyên, n ≥ Ta điền 2n2 số tự nhiên từ 1tới 2n2 vào bảng, số lặp lại2 lần Chứng minh tồn cách chọn 2n2 số tự nhiên từ tới 2n2, số lần cho hàng cột ln có nhất một số chọn.

Bài toán 6.44. Viết n số tự nhiên đường trịn Tìm n sao cho với dãy gồm n

số tự nhiên ta ln tìm hai số cạnh cho sau xoá chúng số cịn lại có thể chia thành hai tập hợp có tổng phần tử nhau.

(70)

Bài tốn 6.46. Cho bảng vng 1983.1984 tơ màu bàn cờ vua Trên ô trắng ta ghi số 1hoặc −1 Tên ô đen ta ghi tích trắng kề với Biết tất các ô đen đều ghi số Chứng minh tất số bảng dều số

Bài toán 6.47. Người ta điền số vào 441ô vuông bảng vuông 21.21 sao cho hàng và cột có khơng q giá trị khác điền vào Chứng minh có số xuất hiện nhất hàng nhất cột bảng vuông này.

Bài tốn 6.48. Cho họ đường trịn mặt phẳng có phần rời cặp. Mỗi đường trịn tiếp xúc với nhất đường trịn khác họ Chứng minh họ có vơ hạn phần tử.

Bài toán 6.49. Giả sử N là số nguyên dương, hai người chơi A, B lần lượt viết số 1,2, , N lên bảng. A viết số ở lượt Từ lượt thứ nếu người viết số n thì người viết sốn+ hoặc số 2n Ai viết sốN là người thắng Ta nói N là kiểu A hay kiểu B tuỳ theo A hay B có chiến thuật thắng Xác định kiểu của 2004 và tìm N nhỏ nhất lớn hơn 2004 khác kiểu với 2004

Bài toán 6.50. Cho tập hợpnphần tử phân biệtA Giả sử tập hợp conU1, U2, , Um

của A thoả mãn tồn số nguyên k sao cho với cặp (i, j) ta có |UiUj| = k Chứng

minh rằng mn.

(71)

Chương 7

Một số chủ đề tổ hợp chọn lọc

7.1 Bài toán Rubik lục lăng

Giới thiệu.Tốn học giống "trị chơi", truyền cho người làm tốn khát vọng chinh phục chiến thắng Ngược lại để chơi giỏi trị chơi đó, cần phải có suy luận toán học logic hợp lý dẫn đường Rubik trị chơi "trí tuệ" Hẳn bạn lần trăn trở khối màu rubik lục lăng (6 mặt), hẳn bạn trầm trồ "biểu diễn" với đồ chơi nho nhỏ Làm họ xoay, xoay xoay để cuối tất khối màu hỗn độn trở vị trí Thực để thực điều bạn cần nắm khoảng 7kỹ thuật

Nhiều người lại cho việc quay rubik quy thủ thuật đơn giản đó, nên khơng cịn thú vị Sự thực người chưa hiểu nhiều rubik Vấn đề rubik phong phú Chẳng hạn làm để tìm thủ thuật có ích, lại tồn thủ thuật Đây câu hỏi khó, mối liên hệ mặt rubik "vướng víu" khơng dễ để tìm mơ hình tốn học tốt cho tốn Tuy nhiên, vấn đề khó thú vị khơng phải mục đích

Trong viết nhỏ muốn đề cập đến kết tưởng chừng hiển nhiên, trăn trở nhiều rubiker sành sỏi Vấn đề có tình cờ lắp ngược khối rubik liệu xoay trạng thái ban đầu hay không Chúng ta cần phân biệt hai trường hợp tương ứng với hai loại vị trí khối,

(72)

cạnh đỉnh Do cần phải giải hai tốn tương đối khác

Bài toán 7.1.1. Chứng minh lật ngược khối cạnh rubik mà các khối khác vị trí ban đầu (vị trí tương đối so với khối khác).

lời giải Để giải toán ta cần tìm bất biến phép quay hợp lệ.

Chúng ta tiến hành đánh dấu mặt khối cạnh mũi tên sau:

Nghĩa thời điểm vẽ mũi tên theo ba vịng "bện" xung quanh rubik Khi khối cạnh hai mũi tên hai mặt khối tạo thành hai hướng Sau xoay, hướng khối vị trí giống với huớng khối vị trí lúc ta nói khối đúng, gọi khối sai trường hợp ngược lại

Rõ ràng sau lần xoay hiệu số số sai giữ nguyên tính chẵn lẻ, điều suy từ kiện khối sai sau xoay thành khối Rõ ràng thời điểm có khối sai hiệu số số lẻ, lúc (chưa xoay) hiệu chẵn Vậy xoay lại cho tất cả6mặt

Một kết tương tự mà quan tâm thay khối cạnh khối đỉnh chuyện Một kết đạt khối đỉnh có "xoay" khơng thể q thoải mái, hình vẽ minh hoạ nói lên điều đó:

Tuy nhiên tốn sau thực khó có lẽ chưa có lời giải (!?)

(73)

7.2 NGUYÊN LÝ BẤT BIẾN VÀ NỬA BẤT BIẾN 73

7.2 Nguyên lý bất biến nửa bất biến

Giới thiệu. Bất biến nửa bất biến công cụ quan trọng có hiệu cao nhiều toán tổ hợp Xét lớp toán sau: cho A, Blà hai tập hợp f phép toán Bắt đầu từA tác độngf lên phần tử củaAta thu đượcf(f(A)) =f2(A) Tiếp tục ta thu f3(A), f4(A), Vậy tốn đặt gì? Đó có tồn hay không số nguyên dương kfk(A) = B tồn liệu có hay

khơng? Rõ ràng câu hỏi hay khó Có nhiều toán cách phát biểu khác xa lại thuộc lớp tốn

7.2.1 Bất biến

Trong mục xét đến lớp tốn khơng tồn tạik tức không ta nhận tập B từ tậpA thơng qua f phương pháp xây dựng đặc trưngψ(X)mà khơng thay đổi q trình thực phép tốnfψ(A)6=ψ(B) Đặc trưng ψ(X) gọi bất biến Chúng ta xét ví dụ sau đây:

Ví dụ 7.2.1. Trên bảng số cho 10 số 11 số −1, lần cho phép lấy số thay vào số nếu số lấy và −1nếu khác Hỏi sau 20 lần số cịn lại là số hay khơng?

lời giải Trong tốn dễ thấy tập A là tập gồm 10 số 1 và 11 số −1, B tập gồm số phép toán cho phép lấy phần tử thay tích chúng Do f thay phần tử tích chúng nên ta dễ dàng thấy bất biến ψ(X) = Q

xX

x Mà ta có

ψ(A) = −16= =ψ(B)do dẫn tới kết luận tốn

Ví dụ 7.2.2. Cho dãy số x0 = 1, x1 = 0, x2 = 1, x3 = 0, x4 = 1, x5 = Với nthì xn+6 là chữ số tận số xn+xn+1+xn+2+xn+3+xn+4+xn+5 Hỏi có tồn k hay không

xk = 0, xk+1 = 1, xk+2 = 0, xk+3 = 1, xk+4 = 0, xk+5 =

lời giải Dễ thấy là A là số (x0, x1, x2, , x5); B = (0,1,0,1,0,1) và:

fk(A) = (xk, xk+1, xk+2, xk+3, xk+4, xk+5).

Với phép toán f xác định:

xk+6 ≡xk +xk+1+xk+2+xk+3+xk+4+xk+5 (mod 10).

Bây ta tìm đặc trưng ψ Với X = (c1, c2, , c6) ta xét từ dạng đơn giản hàm tuyến tính:

ψ(X) =t1c1+t2c2+t3c3+t4c4+t5c5+t6c6 (mod 10). Ta có:

ψ(fk+1(A))≡t1xk+1+t2xk+2+t3xk+3+t4xk+4+t5xk+5+t6xk+6 (mod 10)

(74)

Như ta cần cót6 ≡t1 (mod 10), t6+t1≡t2 (mod 10), , t6+t5 ≡t6 (mod 10) Nên dễ dàng nhận rat1 = 2, t2 = 4, , t6 = 12 thỏa mãn Hay:

ψ(X) = 2c1+ 4c2+ 6c3+ 8c4+ 10c5+ 12c6 (mod 10).

Vậy ln có ψ(fk(A)) =ψ(A) = Nhưng ψ(B) = 6=ψ(A) ta khơng thể có B nghĩa

là khơng tồn k Bài toán chứng minh

Qua ví dụ ta thấy phần tử số ta thường tìm hàm tuyến tính, đa thức, .

Ví dụ 7.2.3. Trên bảng ô vuông vô hạn, ô (1,1) có viên bi Cho phép bỏ bi theo quy tắc sau: Mỗi lần lấy chọn ô (i, j) mà ô (i+ 1, j) (i, j + 1) đều khơng có bi lấy bi ở ơ (i, j) ra khỏi bảng đặt vào ô (i+ 1, j) (i, j+ 1) mỗi ô viên Hỏi cách đó ta làm cho ơ (1,1),(1,2),(2,1),(1,3),(2,2),(3,1) đều khơng có bi hay khơng?

lời giải Đối với toán ta quan tâm tới chứa bi Ta có trạng thái k của các

ô chứa bị bảng cho ta tập hợp Xk ={(i, j)|ơ (i, j) có bi}vớiX0 ={(1,1)} Ta xét hàm ψ(X) = P

(i,j)∈X

s(i, j)

Ta tìm s cho ψ bất biến hay ta cần ψ(Xk+1) = ψ(Xk) với k Rõ ràng ta có

trạng tháik+ nhận từ trạng tháik qua việc bỏ bi ơ(i, j)và thay vào hai (i+ 1, j) và(i, j+ 1)nên ta có ψ(Xk+1)−ψ(Xk) =s(i+ 1, j) +s(i, j+ 1)−s(i, j) Do quan

hệ bình đẳng s(i+ 1, j) s(i, j + 1) nên ta chọn s cho s(i+ 1, j) = s(i, j + 1) đồng thời s(i, j) = 2s(i+ 1, j) ta nghĩ đếns(i, j) = 2i1+j

Như ta ln có ψ(X) = P (i,j)∈X

1

2i+j với k thìψ(Xk) =ψ(X0) = 14

Bây giả sử ta làm cho (1,1),(1,2),(2,1),(1,3),(2,2),(3,1) khơng có bi ta thu tập B gồm ô khơng có bi Mặt khác với cách bỏ bi dễ thấy thời điểm hàng {(1,4),(1,5), } có có bi Tương tự cho cột

{(4,1),(5,1), } có có bi Do ta có thời điểm có hữu hạn có bi nên:

ψ(B)<

21+4 + 24+1 +

X

i,j≥2|i+j≥5 2i+j =

1

4 =ψ(X0).

Nên ta làm Bài toán chứng minh

Những đại lượng bất biến cung cấp cho phương hướng quan trọng để chứng minh biến đổi từ đối tượng thành đối tượng khác Việc phát đại lượng bất biến dường khơng có quy tắc tốt hết ta nên quan sát hết giả thiết, phân tích nó, dựa đối tượng mà hướng đến cố gắng từ biểu thức đơn giản đến phức tạp dần

Bằng cách sử dụng đại lượng bất biến bạn hoàn thành tập sau:

(75)

7.2 NGUYÊN LÝ BẤT BIẾN VÀ NỬA BẤT BIẾN 75 Bài toán 7.2.2. Cho a1, a2, , an là hoán vị của (1,2, , n) Cho phép thực quy

tắc: lần lấy hai số hạng nằm liên tiếp đổi chỗ chúng cho Hỏi sau một số lẻ lần đổi chỗ ta có thu hốn vị ban đầu hay khơng?

Bài tốn 7.2.3. Cho bảng 11×12 Tơ màu bảng theo quy tắc sau: lần tô hoặc ô chưa có màu nằm liên tiếp hàng cơt Hỏi cách đó tô màu hết tất ô vuông sau 19 lần tô hay không ? Câu trả lời với 20 lần tơ.

Bài tốn 7.2.4. Cho lục giác Chia cạnh lục giác thành 1000 phần bằng nhau Nối điểm chia với đoạn thẳng song song với cạnh lục giác Mỗi giao điểm đoạn nói gọi nút Tô màu nút theo quy tắc sau: Mỗi lần tơ nút chưa có màu là 3đỉnh tam giác Hỏi sau số lần tơ ta mà cịn lại một nút khơng có màu nút đỉnh lục giác hay khơng?

Một số tốn lát phủ sau thuộc dạng này:

Bài toán 7.2.5. Một nhà hình chữ nhật lát hình 2×2 1×4 Nhưng lát lại nhà vỡ viên 2×2, người ta thay viên 1×4, Hỏi họ lát nhà lại hay khơng?

Bài tốn 7.2.6. Chom, n≥3là số ngun dương Cho bảng kích thước m×n Mỗi hình nhận từ hình vng 2×2 bị góc gọi thước thợ Ta bỏ bi sau: mỗi lần ta bỏ bi vào ơ mà ơ tạo thành hình thước thợ viên bi Hỏi với cách đó ta bỏ bi cho sau số hữu hạn lần có số bi hay không?

7.2.2 Nửa bất biến

Trong mục xét lớp toán tồn k hay ta nhận B từ A

thông qua f Một hướng để giải toán thuộc dạng xây dựng hàm ψcó đặc điểm :ψ thay đổi đơn điệu qua lần thực phép toánf,ψchỉ nhận số hữu hạn giá trị khác Để thấy rõ xét vài ví dụ:

Ví dụ 7.2.4. Trên một n ≥4 giác đều, đỉnh điền số thực Cho phép thực hiện thực phép toán sau: lần lấy ra4 số a, b, c, d theo thứ tự nằm liên tiếp mà (ad)(bc)<0 và đổi chỗ bvà c cho Chứng minh sau số lần phép toán sẽ phải dừng lại.

lời giải Giả sử a1, a2, , ann số viết đỉnh lúc ban đầu Dễ dàng nhận

thấy lần lấy số i mà (aiai+3)(ai+1−ai+2)<0và đổi chỗ ai+1 ai+2 cho Chú ý là: (aiai+3)(ai+1−ai+2) =aiai+1+ai+2ai+3−aiai+2−ai+1ai+3 Do ta có

thể thấy hàm ψ lấy sau:

ψ(a1, a2, , an) =a1a2+a2a3+ .+ana1.

Rõ ràng tập số đỉnh n-giác không thay đổi sau phép toán nên ψ nhận hữu hạn giá trị Mặt khác sau lần thực phép tốn ta có:

(76)

= (aiai+3)(ai+1−ai+2)<0.

Nghĩa sau lần thực phép tốn giá trị ψtăng lên Mà số giá trị màψ nhận hữu hạn nên thực hữu hạn lần mà thơi

Ví dụ 7.2.5. Tại đỉnh ngũ giác đều, ta đặt tương ứng số nguyên cho tổng cả5số số dương Nếu có 3đỉnh liên tiếp đặt tương ứng 3số x, y, z

y < thì ta thực phép tốn sau: thay số x, y, z đó tương ứng thành số

x+y,y, z+y Phép toán trên thực lặp lại trong số đỉnh cịn có số âm Hãy xác định xem việc thực phép tốn có dừng sau số hữu hạn bước không?

lời giải Gọi x1, x2, , x5 là 5số tương ứng đặt trên5 đỉnh ngũ giác cho:

S =x1 +x2+x3+x4 +x5 > Đặt χ = (x1, x2, x3, x4, x5) f(χ) = f(x1, x2, x3, x4, x5) = (x1−x3)2 + (x2−x4)2 + (x3−x5)2+ (x4−x1)2+ (x5−x2)2 Rõ ràng ta có f(χ)≥

f(χ) khơng thay đổi xi thay đổi vị trí

Khi x3 <0 tốn tử xác định nói tác động lênχ biếnχ thành:

Y = (x1, x2−x3,x3, x4−x3, x5). Từ tính tốn đơn giản ta thu được:

f(Y)−f(χ) = 2x3(x1+x2+x3 +x4+x5) = 2x3S ≤ −2.

Nói cách khác lần thực phép tốn hàm f(χ) sễ giảm đi2 đơn vị Vì vậy, q trình dừng lại sau số hữu hạn bước

Ví dụ 7.2.6. n tập truyện tranh xếp theo thứ tự thành hàng ngang trên giá sách Mỗi lần bạn lấy tập k (1≤kn) nhỏ (khơng nằm vị trí thứ k) xếp nó vào chỗ thứ k từ trái qua phải Hỏi sau số hữu hạn lần làm bạn có thể xếp liên tiếp n tập truyện theo thứ tự khơng?

lời giải Với mỗi0≤in, ta định nghĩadi = 0nếu tập truyệni nằm vị trí thứi

di = ngược lại Đặt ψ=d1+d2.2 + .+dn.2n−1

Lúc ta có ψ ≤ + + . + 2n−1 = 2nψ = tập truyện i

nằm vị trí thứ i Mặt khác lần thực phép thay đổi vị trí tập truyện có

dkdk chuyển từ 1thành dk+1, dk+2, , dn khơng thay đổi Do dù trường

hợp xấu d1, d2, , dk−1 chuyển từ thành ta có ψ giảm đơn vị Do sau khơng q 2n−1 lần phép tốn phải dừng lại Hay bạn ln xếp Chú ý toán này2n−1 lần số lần chuyển sách tối thiểu.

Ví dụ 7.2.7. Giả sử G là đồ thị có bậc đỉnh khơng lớn hơn 11 Chứng minh có thể tô màu đỉnh của G bởi4 màu cho có khơng q n cạnh có hai đỉnh màu.

lời giải Trước hết ta tô màu cách Bây ta chứng minh số phép

(77)

7.2 NGUYÊN LÝ BẤT BIẾN VÀ NỬA BẤT BIẾN 77 Với đồ thị ta gọi ψ số cạnh có hai đỉnh màu Cứ lần mà có đỉnh A(giả sử A có màu xanh) chẳng hạn kề với là3 đỉnh màu với

B, C, D số đỉnh kề với A 11 mà 11 đỉnh tô màu nên có màu tơ màu cho không hai đỉnh, gọi màu đỏ chẳng hạn Thì ta đổi màu đỉnh A

từ xanh sang đỏ Rõ ràng ta có số cạnh có hai đỉnh màu giảm hay

ψ giảm Nhưng rõ ràng ta có ψ ln nhận giá trị ngun khơng âm Do ta có phép đổi màu thực hữu hạn lần hay từ lúc đỉnh kề với khơng q hai đỉnh màu với hay ta có tốn chứng minh

Chú ý với n = 12k n số tốt nhất, lấy đồ thị gồm hợp k đồ thị

K12 thi dễ dàng kiểm tra cách tô màu 4màu có số cạnh có hai đỉnh khơng nhỏ n

Bài tốn thuộc toán trạng thái đặc biệt số toán thuộc dạng này, bạn giải để hiểu rõ phương pháp

Bài tốn 7.2.7. Trong đồ thị có n đỉnh, chứng minh ta tơ màu đỉnh bởi hai màu cho với đỉnh đỉnh kề với nó, số đỉnh màu với nó khơng số đỉnh khác màu với nó.

Bài tốn 7.2.8. Trong nhà có số chẵn bóng đèn lắp đặt căn phịng cho phịng có nhất 3bóng Mỗi bóng đèn có cơng tắc chung với đúng một bóng đèn khác (khơng thiết phải khác phịng) Mỗi lần bật tắt cơng tắc làm thay đổi trạng thái hai bóng mà tác động Chứng minh với trạng thái ban đầu bất kì bóng, có dãy hữu hạn thao tác tắt bật cơng tắc mà ta làm cho mỗi căn phịng có bóng sáng bóng tắt.

Cuối cùng, mời bạn giải ba toán sau đây, xem tập:

Bài tốn 7.2.9. Có bài 52 quân, rút quân Trước rút cho phép đoán chất quân Nếu đốn thắng Chứng minh có thuật tốn để có thể thắng nhất 13 lần.

Bài toán 7.2.10. Giả sử hai người chơi ván cờ caro sau: bảng vô hạn các ô vng, người chơi đến lượt điền ký hiệu X O vào các ô vuông (mỗi người điền loại dấu) Ai điền hàng ngang, cột dọc một đường chéo độ dài 11 gồm tồn dấu chiến thắng Hỏi có chiến thuật để khơng người chơi thua hay không.

Câu hỏi tương tự thay 11 bởi

Bài toán 7.2.11. Cho hàm số f :N ×ZN thoả mãn đồng thời điều kiện sau: i) f(0,0) = 52003, f(0, n) = 0 với số nguyên n 6= 0.

ii) F(m, n) =f(m−1, n)−2.[f(m−1, n)/2] + [f(m−1, n−1)/2] + [f(m−1, n+ 1)/2] với số tự nhiên m >0 và với số nguyên n.

Chứng minh tồn số tự nhiên M sao cho:

f(M, n) = (

(78)

7.3 Phương pháp phân nhóm

Giới thiệu. Có lẽ mục mục có tính chất tổ hợp tồn tài liệu Tuy nhiên, có ích cho bạn đọc muốn nhìn thấy chút liên hệ tổ hợp phân môn khác Thật vậy, bạn theo dõi để thấy lời giải số toán đại số mang màu sắc tổ hợp rõ nét

Ví dụ 7.3.1. Cho số thực x1, x2, , xn thỏa mãn: n

P

i=1

|xi| = 1, n

P

i=1

xi = Chứng minh

rằng: n X i=1 xi i ≤ − 2n.

lời giải Đặt A={i|xi ≥0}, B ={i|xi<0} Khi điều kiện tốn trở thành:

  

P

iA

xi+ P iB

xi =

P

iA

xi

P

iB

xi = 1.

Do P

iA

xi = 12

P

iB

xi =−12 Bây ta có:

X

iA

xi i ≤ X

iA

xi =

1 2và X

iB

xi i =− X

iB

xi

i ≤ −

X

iB

xi

2n =−

1 2n.

Suy iP=1n xi

i

= P

iA

xi

i

P

iB

xi

i

1 −

1

2n. Đây điều phải chứng minh

Ví dụ 7.3.2. Giả sửx1, x2, , xn n số thực cho n

P

i=1

|xi|= Chứng minh tồn

S ⊂ {1,2, , s} thỏa mãn:   

1≤ |S∩ {i, i+ 1, i+ 2}| ≤2 ∀ i= 1,2, , n−2

|P iS

xi| ≥

1 6.

lời giải Với mỗi i= 0,1,2,đặt:

si =

X

xj≥0,ji (mod 3)

xj, ti =

X

xj<0,ji (mod 3)

xj.

(79)

7.3 PHƯƠNG PHÁP PHÂN NHĨM 79 Ví dụ 7.3.3. Với số nguyên dương n kí hiệu d(n) là số chữ số trong cách viết của n hệ số Chứng minh rằng:

+∞ X

n=1 10d(n)

n3 <+∞.

lời giải Với số nguyên dương k kí hiệusk =P10

d(n)

n3 tổng lấy theo tất số

nk chữ số số Ta có:

sk = k−1 X

t=0

10t( X

d(n)=t

1

n3).

Mặt khác ta có có 2kt số có k chữ số số 3mà chứa t số nên:

sk < k−1 X t=0 k−1 t 2kt

33(k−1) <

k−1 X

t=0 27k−110

t

k−1

t

2kt = 27k−112

k−1

= 2.(12 27)

k−1

.

Do ta có +∞P

n=1 10d(n)

n3 = +∞P

k=1

sk <

+∞P

k=1 2.(12

27)

k−1

<+∞ Điều cần chứng minh

Ví dụ 7.3.4. Giả sử x1, x2, , xn, dãy tăng gồm số ngun dương khơng có chữ số

9 nào biểu diễn thập phân Chứng minh rằng: +∞ X i=1 xi < 80.

lời giải Đặtsk =P1

n tổng lấy tất n mà cónk chữ số khơng chứa chữ

số biểu diễn thập phân Ta có đúng8.9k−1 số có k chữ số mà khơng chứa chữ biểu diễn thập phân Do sk <8.9k−1/10k−1 Nên:

+∞ X i=1 xi = +∞ X k=1

sk <

+∞ X

k=1

8.9k−1 10k−1 = 8.

1 1−

10 = 80.

Chú ý: toán tương tự sau có cách chứng minh hồn tồn tương tự ví dụ7.3.4

Bài tốn 7.3.1. Cho trước số nguyên dương n Chứng minh nếu s là tổng nghịch đảo của số không chứa chữ số n−1 trong biểu diễn số n thì s <+∞.

Với tốn ta giải toán sau:

Bài toán 7.3.2. Giả sử {an} là dãy tăng gồm số nguyên dương thỏa mãn {an/n} là dãy

bị chặn Chứng minh có vơ số số hạng thuộc dãy {an} có chứa 2005 chứ số liên tiếp

(80)

lời giải Xét sốm = 102005 bây ta có dãy{an/n} bị chặn nên ta chứng minh

rằng tồn vô sốnsao choanchứa chữ sốm−1 Thật ta có dãyankhơng chứa số

có chữ sốm−1 Khi theo tốn7.3.1ta có +∞P

i=1

ai

<+∞ Nhưng ta lại có dãy số {an/n}

bị chặn nên ta có tồn tạiC choan< nC với mọin Suy

+∞P

i=1

ai

>

+∞P

n=1

nC = +∞ (mâu

thuẫn) Vậy tồn tạin0 mà an0 có chứa chữ số m−1 Bây xét dãy an0+1, an0+2 , ak,

Dãy có tính chất đề nên tồn vơ số nan chứa chữ số m −1

số m Hay có vơ số nan chứa2005 chữ số biểu diễn thập phân

Như qua tốn ta có nhận xét là: dãy số nguyên dương a1 < a2 < thỏa mãn khơng có nan chứak chữ số 9liên tiếp

P

n≥1

xi

<+∞ Bây kí hiệu

Sj tổng nghịch đảo số nnj chữ số vàn khơng chứa chữ số k chữ số

liên tiếp biểu diễn thập phân

Giả sử M tổ hợp gồm k chữ số hệ thập phân (với chũ số khác 0) Khi số chữ số nnj chữ số không chứa số M số số n

nj chữ số khơng chứa k chữ số9 liên tiếp Do đặt tj tổng các sốnn

j chữ số hệ thập phân vàn không chứa M Khi tj ≤10sj Như với t tổng

nghịch đảo số nguyên dương mà không chứa M biểu diễn thập phân ta có

t=t1+t2+ .≤10(s1 +s2+ .)<+∞ Vậy ta giải toán

Bài toán 7.3.3. Giả sử M là tổ hợp các chữ số (với chữ số đầu khác 0) Khi đó nêu t tổng nghịch đảo các số không chứa M trong biểu diễn thập phân thì t <+∞.

Với tốn7.3.3thì ta thu hệ tốn 7.3.4 sau (là toán tổng quát đề thi VN TST 2005 trên)

Bài toán 7.3.4. Giả sửM là tổ hợp chữ số (với chữ số khác 0) Giả sử dãy số {an} là dãy số nguyên dương tăng dãy {an/n} là dãy bị chặn có vơ số số hạng

của dãy có chứa M trong biểu diễn thập phân.

Với phương pháp ta dễ dàng giải số toán sau:

Bài toán 7.3.5. Chứng minh với dãy {x1, x2, xn} có tính chất khơng có số nào

bắt đầu số khác trong n số thì:

n

X

i=1

xi

≤1 +1 2+

1

3 + .+ 9.

Bài toán 7.3.6. Với n là số nguyên dương kí hiệu f(n) là số chữ số trong biểu diễn thập phân của n Chứng minh rằng:

+∞ X

n=1

af(n)

n2 <+∞ ⇐⇒a <91.

(81)

7.4 VAI TRÒ CỦA CÁC BỘ SỐ ĐẶC BIỆT 81

7.4 Vai trò số đặc biệt

Giới thiệu. Một tính chất đặc trưng tốn tổ hợp mang tính chất tối ưu tính hai chiều chứng minh Chẳng hạn muốn chứng minh đáp số toán k =α, nhiệm vụ ta phải chứng minh kα kα, cịn phải cách bố trí cho cách bố trí thực k =α Bước thứ nhiều trường hợp khơng q khó khăn, khó khăn thực lại nằm bước thứ hai Trong mục xin dẫn số ví dụ

Bài tốn 7.4.1. Tại cửa hàng ăn thực đơn có 20 món Có 10 khách hàng vào ăn, mỗi người gọi đúng món Hai người gọi giống giả sử a món nhiều người gọi Gọi k là số người gọi món a Hãy tìm giá trị nhỏ của

k.

lời giải Gọi ăn là {1,2, , n} danh sách ăn người chọn tập ba phần tử tập hợp Ta chứng minh câu trả lời toán Để chứng minh k ≥ ta xét 10 tập hợp phần tử A số 10 phần tử Do tập A có chung phần tử với tập hợp àno tập lại nên theo nguyên lý Dirichlet suy có phần tử A xuất khơng ba lần tập Vậy k ≥ Nếu k = tức phần tử A xuất ba lần tập lại Điều thực với phần tử thuộc vào ăn gọi tổng số ăn (tính lặp) là10.3 = 30khơng chia hết cho4 Vậyk 6= 4suy rak ≥5

Bây ta cách gọi mà k = Thật vậy, xét 10 tập phần tử sau {1,2,3},{1,4,5},{1,6,7},{2,4,6},{2,5,7},{3,4,7},{3,5,6},{2,4,6},{3,5,7},{3,5,7} Trong tập xuất lần, số 2,4,6 xuất lần 3,5,7 xuất lần Vậy k = Do k nhỏ là5

Bài tốn 7.4.2. Tìm số tự nhiênknhỏ cho trongkphần tử tuỳ ý tập{1,2, ,50}

luôn chọn ba số độ dài cạnh tam giác vuông.

lời giải Trước hết ta liệt kê toàn tập ba phần tử tập {1,2, ,50} mà ba phần tử độ dài ba cạnh tam giác vng Có 20 tập vậy:

(82)

Một tập hợp có khơng 42 phần tử chứa tập hợp Vậy giá trị nhỏ củak là42 Bài toán giải xong

Bài toán 7.4.3 (IMO 1991) Xét tập hợp S ={1,2, ,280} Tìm số nguyên dương n nhỏ nhất thoả mãn n số lấy từ S số nguyển tố đôi một.

lời giải Trước hết ta ra 216 phần tử của S mà trong 5 phần tử ln có 2 phần

tử có ước chung lớn Thật vậy, tập hợp sau có tính chất (chứng minh khơng q khó):

A={kS |k chia hết cho 2,3,5,7 11}.

Xét tập hợp TS gồm 217 phần tử Ta chứng minh T chứa phần tử đôi nguyên tố Đặt:

B1 =A\ {2,3,5,7}

B2 ={112,11.13,11.17,11.19,11.23,132,13.17,13.19}

P =S\(B1∪B2).

Rõ ràng |P|=|S| − |B1| − |B2|= 60 P hợp tất số nguyên tố S Néu|TP| ≥5 ta có điều phải chứng minh

Nếu|TP| ≤4 Ta có|T∩(S\P)| ≥217−4 = 213 Suy tập hợp số khơng ngun tố S có nhiều 280−213−60 = 7số không thuộc T Xét tập hợp :

M1={2.23,3.19,5.17,7.13,11.17}

M2={2.29,3.23,5.19,7.17,11.13}

M3={2.31,3.29,5.23,7.19,11.17}

M4={2.37,3.31,5.29,7.23,11.19}

M5={2.41,3.37,5.31,7.29,11.23}

M6={2.43,3.41,5.37,7.31,13.17}

M7={2.47,3.43,5.41,7.37,13.19}

M8={22.32,52,72,132}.

Rõ ràng tồn tạii0 để TMi0, suy điều phải chứng minh

Thực chưa có phương án cụ thể để giải chiều ngược lại loại tốn Tuy nhiên chúng tơi muốn sử dụng ví dụ nhỏ coi bước đầu cơng phá vấn đề Bài tốn 7.4.4. Tìm số tự nhiênk nhỏ cho trongkphần tử tuỳ ý tập{1,2, ,50}

luôn chọn hai số độ dài hai cạnh góc vng tam giác vng.

(83)

7.4 VAI TRỊ CỦA CÁC BỘ SỐ ĐẶC BIỆT 83 Xét B = {3,33,12,24,8,36,40,48,16,20,28} Ta thấy cặp P ythagore có phần tử thuộcB Do C=S\B tập hợp không chứa cặp số P ythagorenào Suy k >|C|= 39

Xét11 cặp sóP ythagorerời (3,4), (33,44), (5,12), (7,24), (6,8), (27,36), (40,42), (48,14), (16,30), (45,28), (21,20) Vì tập gồm40 phần tử củaS phải chứa cặp P ythagore , cho nênk ≤40

Kết hợp hai đánh giá suy k = 40

Vấn đề làm để tìm tập hợp B 11 cặp P ythagore rời Suy nghĩ chút, có lẽ bạn nhận thấy đồ thị trợ giúp tuyệt vời

Biểu diễn phần tử tậpS điểm đồ nối tất cặpP ythagore

(a, b) Ta xây dựng tập B gồm phần tử cho tập C = S\B khơng có

P ythagore Muốn vậy, đồ thị, B phải đỉnh cho ta xoá đỉnh tất cạnh có nối với đồ thị khơng cịn cạnh

Từ ý đồ đó, ta tiến hành chọn điểm B sau Vì (3,4), (33,44) hai đoạn riêng lẻ ta chọn {3,33} ∈ B (hoặc {3,44}, {4,33}, {4,44} được) Xố đoạn Các đoạn thẳng có đầu đỉnh đơn đỉnh (khơng đơn) phải thuộc B Vậy

{12,24,8,36,40,48} ∈B Xố chúng cạnh nối với chúng

Cặp (16,30) riêng lẻ nên ta chọn 16 (hoặc 30) Cuối {20,28} ∈ B Lúc này, đồ thị bị xoá hết cạnh Vậy B ={3,33,12,24,8,36,40,48,16,20,28}

(84)

Xoá tất cạnh có đầu nằm số diểm (cịn lại hình) Dễ dàng chọn tiếp (16,30) Và chọn hai bốn cặp (45,28), (28,21), (21,20) (20,15).Chẳng hạn ta chọn (45,28), (21,20) Đúng 11 cặp Pitago rời xác định Bài tốn giải xong hồn tồn

Cuối mời bạn giải hai toán sau xem tập nhỏ:

Bài tốn 7.4.5. 10 em bé đứng gần nhau, em bé càm bóng Khi có hiệu lệnh em bé đồng thời chuyền bóng cho bạn đứng gần Hỏi cuối có ít bao em bé có bóng (giả thiết khoảng cách cặp em bé khác nhau). Bài toán 7.4.6 (VMO 2005). Trong mặt phẳng cho bát giác lồi A1A2A3A4A5A6A7A8 khơng có ba đường chéo cắt điểm Ta gọi giao điểm hai đường chéo bát giác nút Xét tứ giác lồi mà tức giác có bốn đỉnh là đỉnh bát giác cho Ta gọi tức giác tức giác Hãy tìm số ngun dương n nhỏ có tính chất tơ màu n nút cho với mọii, k ∈ {1,2,3,4,5,6,7,8}

i6=k, ký hiệu s(i, k)là số tứ giác nhận Ai, Ak làm đỉnh đồng thời có giao điểm

hai đường chéo nút tơ màu tất giá trị s(i, k) đều nhau.

7.5 Hai tốn phủ hình vng

Giới thiệu. Dùng hình đồng dạng để phủ hình ban đầu vấn đề thú vị hình học tổ hợp Trong viết xin dẫn hai kết tương đối quen thuộc việc dùng hình vng bé để phủ hình vng lớn ban đầu, trình bày lời giải cho toán Từ lời giải bạn thấy lời giải tốn cịn lại Bài tốn 7.5.1. Chứng minh khơng thể dùng hai hình vng có cạnh nhỏ hơn 1để phủ một hình vng cạnh

Bài tốn 7.5.2. Chứng minh khơng thể dùng năm hình vng có cạnh 1/2 để phủ một hình vng cạnh

lời giải Xét hình vng ABCD tâm O cạnh 1, gọi cạnh của5 hình vng dùng để phủ

(85)

7.5 HAI BÀI TOÁN VỀ PHỦ CÁC HÌNH VNG 85

Xét hình vng VA, rõ ràng tổng độ dài hai đoạn mà phủ hai cạnhAB, AD phải nhỏ

hơn 1, x+y ≤p2(x2+y2)≤√2.√2.a < 1 Do tổng đoạn mà V

A, VB, VC, VD phủ

trên cạnh nhỏ 4, VO phải có giao với cạnh Giả sử cạnh

BC

Xét hình vng cạnh a phủ lên đoạn hai đường thẳng song song cách 1/2 hình vẽ Gọi l độ dài mà hình vng phủ lên hai đường Ta có:

l = (h1 +h2)(tanϕ+ cotϕ) =

h1+h2 sinϕ.cosϕ =

2 cos (ϕ−450)−1/2 sinϕ.cosϕ

=⇒l <

2 cos (ϕ−450)−a

sinϕ.cosϕ =

a(cosϕ+ sinϕ)−a

sinϕ.cosϕ =a

a(sinϕ−1)(cosϕ−1) cosϕ+ sinϕa.

Vậy tổng độ dài hai đoạn phủ ≤a

Bây xét l1, l2, l3 ba đường thẳng cách 1/2 hình vẽ đây:

Trong hai cặp (VA, VB) (VC, VD) cặp phải có hình phủ hai đoạn

cách nhau1/2, cịn hình phủ lên đoạn ≤ √

2a Vậy tổng độ dài mà 5hình vng phủ lên3 đoạn thẳng l1, l2, l3 da+a+a+

2a+

2a= (3 +

(86)

7.6 Câu hỏi mở tính chất chùm đường tròn

Giới thiệu. Việc khảo sát mối quan hệ nhiều đối tượng đặc trưng tổ hợp Trong mục khảo sát số tính chất chùm đường trịn Tất nhiên số lượng tính chất kiểu đa dạng, liệt kê viết nhỏ Vì chúng tơi tập trung vào đối tượng đặc biệt, đường tròn đơn vị Hạn chế quan tâm tới đường tròn đơn vị giả thiết thêm cặp đường tròn số chúng cắt hai điểm phân biệt Đối với 3đường trịn có hai cách bố trí sau:

Cách bố trí bên phải cho ta tính chất đẹp, tam giác ABC có bán kinh đường trịn ngoại tiếp bé đơn vị, cịn bên trái, điều khơng phải lúc Các bạn tự chứng minh tham khảo thêm phần hình học tài liệu

Câu hỏi tạo nên khác biệt? Thực có tính chất có mặt tam giác cong "hổng" mà ta gọi "lỗ" cách bố trí đường tròn Một câu hỏi tự nhiên đặt phải nếun hình trịn xếp tuỳ ý liệu tạo tứ giác cong, ngũ giác cong hay không Câu trả lời có, xác ta có: Định lý.Lỗ cong n đường trịn tạo có nhiều n cạnh

Chứng minh.

(87)

7.6 CÂU HỎI MỞ VỀ MỘT TÍNH CHẤT CỦA CHÙM CÁC ĐƯỜNG TRÒN 87

Bắt đầu từ (O1),(O2),(O3) ba đỉnh lỗ A1, A2, A03 Quy ước hai điểm A, B (AB) đường trịn bán kính có tâm khơng thuộc nửa mặt phẳng bờ AB hứa Q (là giao điểm thứ hai của(O1),(O2)) Lấy A3, A40 sau, điểmA3 thuộc cungA2A03 điểmA

0 thuộc cungA0

3A1 mà A3 đủ gần A2,A04 đủ gần A

3 cho(O4) = (A3A04) cắt (O2) Điều thực C4 = 4, tứ giác cong trường hợp A1A2A3A04

Bây cóCn=n vớin≥4theo cách xây dựng trên, lấy A0n+1 thuộc cung

A0

nA1 đủ gần A0n điểm An thuộc cung An−1A0n đủ gần An−1 cho (On+1) = (AnA0n+1) cắt (O2), điều thực ta có Cn+1 = n + 1, n + - giác cong lúc

A1A2 AnA0n+1 Định lý chứng minh hoàn toàn

Chúng ta khai thác theo cách khác Chúng ta biết theo định lý Helly cho họ hình lồi với n hình trịn mà ba có giao điểm chung điều với hệ n hình trịn, tức khơng thể tồn lỗ Vậy, xếp tuỳ ý từ n hình trịn tạo nhiều lỗ Đây toán thực khó kết chúng tơi đạt khảo sát nhỏ Gọi Ln số lỗ

nhiều tạo Thế ta có:

Định lý.Số lỗ nhiều tạo khơng nhỏ n−2 Lnn−2

Chứng minh. Rõ ràng L3 ≥ 1, xét n ≥ Công việc cách bố trí mà cách bố trí có n−2 lỗ tạo Quy ước viết (O) tức đường trịn tâmObán kính1 Xét điểmO1, vẽ đường tròn(O1,2) LấyO2, Ontrên đường

tròn cho O2dO1On ≤600 Trên cung nhỏ O2On lấy điểmO3, , On−1 cho

i= 3, , n−1thì giá trị gócO2dO1Oi tăng bé O2dO1On Với cách

xác định (O1) tiếp xúc với(Oi) với i= 2,3, , n, với mọi2≤j < kn

thì(Oj) (Ok)cắt Lúc ta có n−2 lỗ tạo thành

Xét tia O1x cắt cung nhỏ O2On với ≤ j < kn tồn số j,k mà với

mọi phép tịnh tiến (O1) thành (O1,j,k) thoả mãn O1,j,kO1x O < O1O1,j,k < j,k Khi

lỗ ba đường trịn (O1),(Oj),(Ok) bảo tồn tính tồn Và đổi tâm O1 O vớiOO1x 0< O1O <min{j,k | 2≤j < kn} Định lý chứng minh

Một cách tự nhiên đặt câu hỏi, n − có phải câu trả lời tốt cho tốn hay khơng Đây câu hỏi mở

Bài toán 7.6.1 (Open Question). Chứng minh số lỗ tạo từn đường tròn đơn vị nhiều là n−2

THAY Cho LờI KếT Vấn đề chùm đường tròn nhiều, chẳng hạn vấn đề

(88)

7.7 Định lí Konig-Hall

Trước hết ta cần nhắc lại vài hiểu biết khái niệm cặp ghép lý thuyết đồ thị Cho đồ thị lưỡng phân G= (X, Y, U), tập cạnh E (EU) gọi cặp ghép đồ thị

G tập cạnh độc lập đồ thị G hai cạnh thuộc E khơng có đỉnh chung Nếu cặp ghép lập quan hệ tương ứng - tập AX tập BY ta nói ghép cặp tậpA tập B hay ghép cặp A vàoY

Định lý Hall-Konig.Trong đồ thị hai mảng vơ hướngG= (X, Y, U) ta ghép cặp X

vào Y tập AX có |D(A)| ≥ |A| (Với D(A) tập hợp đỉnh Y kề với đỉnh A)

Chúng ta phát biểu định lí theo cách khác sau: Cho tậpA={1,2, , m}

và giả sửA1, A2, , Anlà tập củaA Gọi dãyx1, x2, , xnlà tốt nhưxiAi

với i= 1,2, , n, đồng thời xi 6=xj với i 6=j ∈ {1,2, , , n} Khi điều kiện

cần đủ để tồn dãy tốt với I ⊂ {1,2, , n} thì|S iI

Ai| ≥ |I|

Chứng minh.Ta chứng minh phương pháp quy nạp theok=|X|+|Y| Vớik = 2,3thì hiển nhiên ta có|X|= khẳng định Giả sử toán với mọi1≤kn Ta chứng minh khẳng định chok =n+ 1≥4 Thật xét trường hợp: Nếu với AX ta có |D(A)| ≥ |A|+ Khi ta lấy đỉnh x0 ∈ X lấy đỉnh y0 ∈Y kề với x0 Xét đồ thị G0 nhận từGkhi bỏ đỉnh x0,y0 cạnh x0y0 dễ thấy G0 với AX \ {x0} ta có |DG0(A)| ≥ |A| Khi theo giả thiết quy nạp ta ghép cặp từX\ {x0}vàoY \ {y0} Kết hợp thêm cạnh x0y0 ta ghép cặp từ X vàoY

Nếu tồn tạiAX mà|D(A)|=|A| Khi đặtX0 ⊂X\Avà xóa hết cạnh xuất phát từ đỉnh trongAđồng thời xóa đỉnh thuộcD(A) Ta thu đồ thịG0= (X0, Y0, U0) Ta chứng minh với mọiBX0 thì|D

G0(B)| ≥B Thật xảy ra|DG0(B)|<|B| cách xét tập AB G ta có:

|DG(AB)|=|DG(A)|+|DG0(B)|<|A|+|B|=|AB|.

Vơ lí Do theo giả thiết quy nạp ta ghép cặp từ X0 vào Y0, từ A vào D(A) suy ra ghép cặp từX vào Y Như theo nguyên lí quy nạp tốn với k Định lí chứng minh

Định lí kết hay lí thuyết đồ thị có nhiều tốn khác giải gọn gàng nhờ áp dụng cách khéo léo toán Chúng ta xem xét số vài tốn

Bài tốn 7.7.1. 1000 hộp bi, hộp chứa đúng 10 viên bi Biết khơng có 11 viên bi màu Chứng minh lấy từ hộp hịn bi cho tất cả 1000viên được lấy có màu khác nhau.

(89)

7.7 ĐỊNH LÍ KONIG-HALL 89 k hộp có tất 10×k viên bi Mặt khác lại có khơng có 11 viên bi màu ta có k hộp bi có khơng q (k−1)×10 viên bi Đây điều vơ lí Nhận xét chứng minh

Vậy ta đưa toán điều kiện thỏa mãn định lí nêu Theo đinh lí ta chọn được từ 1000 hộp bi cho1000 viên bi có màu khác nhau, hộp1 viên Bài tốn 7.7.2. Cho bảng vng n×n Trong vng bảng có điền số thực không âm cho tổng số nằm hàng, cột bảng là Chứng minh ta chọn n vng khơng có hai hàng hay cột mà trong mỗi điền số thực dương.

lời giải Đối với toán ta cần ý tới quan hệ hàng cột, ô vuông

là giao hàng cột nên ta xét quan hệ kề sau Hàng i cột j kề ô giao hàng i cộtj điền số thực dương Chú ý tới điều kiện tồn n mà khơng có hai không nằm hàng hay cột ô điền số thực dương Ta thấy điều tương đương với tìm n cạnh mà đơi khơng có đỉnh chung Hay ghép cặp từ tập hàng vào tập cột (theo quan hệ trên) Sử dụng định lí ta cần k hàng kề với k cột Thật ta giả sử có k hàng mà kề vớimk−1cột ta xét bảng chữ nhật

k×m tạo k hàng vàm cột Khi ta có tổng số nằm hàng Do tổng số viết bảng k Tuy nhiên tổng số nằm cột ≤1 tổng số viết bảng khơng vượt q mk−1(Vơ lí)

Vậy k hàng kề với k cột nên theo định lí suy điều cần chứng minh

Bài toán 7.7.3. Cho 2n đại biểu đến từ n quốc gia, quốc gia có đúng đại biểu Biết rằng họ ngồi tròn Chứng minh ta chia họ thành nhóm, nhóm gồm n đại biểu đến từ n nước mà khơng có người ngồi liên tiếp.

lời giải Ta đánh số 2n người theo thứ tự theo chiều quay kim đồng hồ Và chia họ

thành n cặp (1,2), ,(2n−1,2n) Bây ta chứng minh lấy từ cặp người cho n người lấy đến từ n nước khác Thật ta có cặp gồm người nước có 2đại biểu theo tốn7.7.1ta chọn từ cặp người cho n người đến từ n nước khác Rõ ràng n người cịn lại đến từ n nước khác dễ thấy khơng có ba người thuộc nhóm ngồi cạnh liên tiếp Đây điều phải chứng minh

Bài toán 7.7.4. Trong hội nghị có 100 đại biểu đến từ 25 quốc gia khác nhau, quốc gia có đúng đại biểu Biết họ ngồi bàn trịn Chứng minh chia họ thành nhóm phân biệt, nhóm gồm 25 đại biểu của 25 nước mà khơng có hai đại biểu nào nhóm ngồi cạnh nhau.

lời giải Theo tốn 7.7.3 ta chia 100 người thành 2 nhóm cho nhóm

(90)

cùng nhóm ngồi cạnh Tương tự cho nhóm lớn cịn lại Ta có 100 người chia thành nhóm cho nhóm có 25 đại biểu đến từ 25 nước khác hai người nhóm ngồi cạnh Điều phải chứng minh

Cuối bạn sử dụng định líHallKonig để giải số tốn sau đây:

Bài toán 7.7.5. Trong khiêu vũ có n ơng và n bà Biết ông quen đúng k

bà bà quen đúng k ông (k là số nguyên dương cho trước) Chứng minh ta có thể chia họ thành n cặp nhảy cho người nhảy với người mà quen.

Bài tốn 7.7.6. Cho bảng vng n ×n Giả sử vng đơn vị bảng con

n×k(k < n) của bảng tô bởin màu (mỗi màu sử tơ cho khơng q n ơ) thỏa mãn khơng có hai ô màu nằm hàng hay cột Chứng minh rằng ta tơ màu tiếp cịn lại bảng bởi n màu cho màu tơ cho đúng

n ơ khơng có hai màu nằm hàng hay cột.

Bài tốn 7.7.7. Cho bảng n×n biết vng bảng điền số tự nhiên sao cho hai hàng khác Chứng minh ta xóa cột mà bảng

n×(n−1) thu có tính chất trên.

7.8 Định lý Erdos - Skerezes

Định lý Erdos - Skerezes.Số Ramsey R(m, n) vớim, nN số nguyên dương nhỏ thoả mãn tính chất trongR(m, n)người tồn nhấtmngười đơi quen tồn n người đơi khơng quen Khi với mọim, nZ, m, n≥2ta có bất đẳng thức sau R(m, n)≤R(m−1, n) +R(m, n−1)

Hệ Với m, nlà số nguyên lớn hơn 2 ta có:

R(m, n)≤

m+n−2

m−1

.

Chứng minh. Đặt a = R(m−1, n) +R(m, n−1) Xét a người bất kỳ, ký hiệu người tương ứng với điểm mặt phẳng, hai người quen nối với màu xanh, ngược lại khơng quen hai điểm tương ứng với hai người nối màu đỏ Xét ngườiAbất kỳ trongangười Giả sử số cạnh màu xanh (tương ứng đỏ) xuất phát từ đỉnh làu(tương ứngv) Như ta cóu+v=a−1 =R(m−1, n)+R(m, n−1)−1

=⇒[vR(m, n−1)] + [uR(m, n−1)] =−1.

Do hai số hạng ngoặc vuông số nguyên suy tồn hai số khơng âm Xét hai trường hợp:

Trường hợp 1: vR(m, n−1) ≥ suy vR(m, n−1) Do v người tương ứng với v đỉnh có m người quen n−1 người không quen Mà A

(91)

7.8 ĐỊNH LÝERDOS - SKEREZES 91 Trường hợp 2: uR(m−1, n)≥0suy uR(m−1, n) Do tậpu người có m−1 người quen hoặcn người không quen MàAquen với tất u người suy a người m người quen n người không quen

Kết hợp hai trường hợp ta suy R(m, n)≤R(m−1, n) +R(m, n−1)

Từ khẳng định ta dễ dàng suy 2nn ln có n người đôi quen đôi không quen Bên cạnh thấy việc đánh giá số R(m, n) lỏng, tất nhiên trường hợp riêng đánh giá chặt chẽ cần phải có kỹ thuật tinh tế hơn, ví dụ với 2cách chứng minh

Bài tốn. Chứng minh 18 người tuỳ ý tồn người đôi quen đôi không quen

Rõ ràng đánh giá chặt chẽ cho số R(4,4), 4+4−24−1 = 63= 20>18

lời giải 1 Ký hiệu người điểm mặt phẳng tô màu đoạn thẳng

nối hai điểm số điểm hai màu xanh, đỏ Xét điểm A1 nối với 17 điểm lại hai màu, suy tồn chín điểm nối vớiA1 màu, giả sử A1A2, , A1A10 tô màu xanh Nếu tồn tam giác ó ba cạnh màu xanh AiAjAk với ≤ i < j < k ≤ 10 điểm A1, Ai, Aj, Ak nối

màu xanh Ngược lại bốn điểm A, Ai, Aj, Ak tồn cạnh đỏ nối chúng

thì ta chứng minh tồn bốn điểm nối với màu đỏ Xét9 điểm A2, A3, , A10, A2 nối với điểm lại hai màu Xét hai trường hợp

Trường hợp 1: tồn không điểm nối với A2 màu xanh, giả sử

A3, A4, A5, A6 tam giác dạng A2AmAn với ≤ m < n ≤ có cạnh A2Am, A2An

xanh suy raAmAn đỏ, bốn điểm A3, A4, A5, A6 nối toàn cạnh đỏ

Trường hợp2: đỉnhAi vớii= 2,10 có khơng 5cạnh đỏ, tổng

số cạnh đỏ nối điểm số chẵn (tính lặp nên cạnh đỏ tính lần), 9.5 = 45 lại số lẻ suy tồn điểm nối với cạnh đỏ Giả sử

A2A3, , A2A8 Xét điểm A3 nối vớiA4, A5, A6, A7, A8 có nhất3 điểm nối vớiA3 màu, chẳng hạn A3A4, A3A5, A3, A6 màu Nếu tồn tạiA4A5, A4A6 hoặcA5A6 màu với A3A4 tồn tam giác màu, ngược lại tam giácA4A5A6 màu Như từ điểm A3, A4, A5, A6, A7, A8 ln có tam giác màu, theo tính chất tồn cạnh đỏ xác định tam giác màu phải màu đỏ, giả sử ∆A3A4A5 Khi bốn điểm A2, A3, A4, A5 đôi nối với màu đỏ

Bài toán chứng minh xong

Lời giải suy luận thông thường dựa nguyên lý Dirichlet Chúng muốn giới thiệu với bạn lời giải nữa, lời giải có sử dụng kết chứng minh viết mở rộng toán6người trình bày cuối tài liệu

lời giải 2 Sử dụng cách tô màu cách 1, áp dụng định lý số tam giác cùng

(92)

nhau tồn đoạnAB cạnh chung nhất4tam giác màu Gọi4đỉnh cịn lại bốn tam giác màu làC1, C2, C3, C4

Nếu tồn tạiCi, Cj màu vớiAB A, B, Ci, Cj nối màu

Nếu đoạn CiCj khác màu với AB thìC1, C2, C3, C4 tơ màu

Vậy hai trường hợp ta bốn người đơi quen đôi không quen nhau, tốn chứng minh hồn tồn

7.9 Một số toán khác

Bài toán 7.9.1 (Bài toán đồng hồ). Đối với đồng hồ treo tường, quan sát hai kim phút Tất vị trí mà cặp hai kim tạo thành nửa ngày được gọi hợp lý Đếm số vị trí hợp lý mà đổi chỗ vị trí hai kim ta hợp lý.

b

lời giải Chúng ta sử dụng trục toạ độ để biểu thị vị trí hai kim:

Giả sử kim chỉx giờ, kim phút y phút Vì kim phút chạy nhanh gấp 12 lần kim nên điều kiện để vị trí(x, y)hợp lý lày= 12{x} Và để thay đổi vị trí hai kim phải cóx= 12{y} Giải hệ hai phương trình sau: (chú ý(0,0)≡(12,12))

x

144 =

{y}

12 =

y

12 − [y]

12 ={x} − [y]

12 =⇒x= 144{x} −12[y] =⇒[x] + 12[y] = 143{x}. Vậy{x}=k/143vớik = 0,1, ,142 =⇒y= 12k/143 Giả sử12k = 143p+qvới0≤q≤ 142 =⇒ {y} =q/143 =⇒ x = 12q/143 Ta phải chứng minh {12q/143} = k/143, điều tương đương với143|12qk ⇐⇒143|12(12k−143p)−k ⇐⇒143|143k−12.143p(luôn đúng)

(93)

7.9 MỘT SỐ BÀI TOÁN KHÁC 93

Lời Bình Chủ đề đồng hồ cịn nhiều, chẳng hạn câu hỏi ngày có

thời điểm mà đổi chỗ 3kim (giờ, phút, giây) cho ta lại hợp lý hay khơng Ngồi liệu kim tạo với góc 1200 hay khơng Đây các câu hỏi khơng q khó tuơng đối thú vị, việc giải chúng xin dành cho bạn đọc Bài toán 7.9.2. Trong mặt phẳng cho 2001 điểm, khơng có điểm thẳng hàng Một số cặp điểm nối với đoạn thẳng cho điểm nối với nhất 1601 điểm khác Chứng minh điểm nói có điểm đơi nối với nhau, số 1601 đổi thành 1600 thì kết khơng cịn nữa.

lời giải Gọi n là giá trị lớn tập hợp số số lượng tập điểm đôi

một nối với Xétn <2001 Khi với điểm tập conn điểm có khơng nhiều 2001−1−1601 = 399 điểm không nối với điểm Do tập điểm mà khơng nối với n điểm nói có số phần tử ≤399.n

Nếu 399n < 2001− n chắn cịn điểm nối với tất n điểm này, trái với giả thiết tính cực đại củan Vậy ta phải có399n ≥2001−n =⇒n≥2001/400 >5 n≥6 Điều phải chứng minh

Bây ta chứng minh thay 1601 1600 có cách nối mà cách nối điểm tồn cặp điểm nối với Gọi điểm

{Ai|i = 1,2, ,2001} Ta nối AiAj i 6≡j mod Rõ ràng với cách nối

mỗi điểm nối với1600 điểm khác, mà 6số tuỳ ý ln có hai số đồng dư với modulo 5, nên hai điểm có số tương ứng khơng nối với

Lời Bình Kỹ thuật sử dụng tốn gọi nguyên lý cực đại, khởi

đầu cực trị Nội dung nguyên lý tập hữu hạn số ln có số nhỏ số lớn Ngồi tốn tổng quát cách dễ dàng sau

Bài toán 7.9.3 (Tổng quát). Cho tập hợp S điểm (S >1) trên mặt phẳng, khơng điểm thẳng hàng điểm nối với nhất M điểm khác (S < M) Chứng minh tồn tại

S−1

SM

+ điểm đôi nối với nhau.

Sử dụng toán giải tốn sau khơng khó khăn: Bài tốn 7.9.4 (TST VMO 2004). Xét tập hợpS ={a1 < a2 < < a2004}, với phần tử aiS ta gọi f(ai) là số lượng phần tử của S mà nguyên tố với ai Giả

thiết rằng f(ai) < 2003 f(ai) = f(aj) với mọii, j ≤ 2004 Tìm số nguyên dương k

nhỏ có tính chất với tập con k phần tử của S đều chứa hai phần tử mà ước chung lớn chúng lớn hơn

Bài tốn 7.9.5. Giả sử A là tập hợp có n phần tử và A1, A2, , An là tập có

khơng hơn phần tử của A Giả sử với phần tử x, yA đều tồn nhất i thoả mãn x, yAi Chứng minh rằng |AiAj|= với mọi 1≤i < jn.

lời giải Đặt |Ai|=ai vớii = 1,2, , n Giả sử A ={1,2, , n}, di số tập hợp Aj chứa i

Ta có:

n

X

i=1

ai = n

X

i=1

(94)

X

|AiAj|=

X di

2

. (7.2)

Mặc khác với phần tửx, yA tồn ithoả mãn x, yAi nên ta xét

số cặp (x, y, Zi)với x, yAi suy ra: n X i=1 ai = n . (7.3)

Các công thức (7.1),(7.2),(7.3) gọi nội công thức Bây ta cần phải thiết lập hệ thức riêng khác Giả sử Am = {s1, s2, , sp}, với b /Am suy có p tập

khác chứa {b, si} =⇒dbp Do diaj với mọii /A

=⇒ n

X

i=1

di = n

X

i=1 X

j,i /Aj

di

ndi

| {z }

ndigiá trịj

=

n

X

j=1 X

i,i /Aj

aj

naj

| {z }

ncjgiá trịi

=

n

X

j=1

aj.

Để có đẳng thức ta phải có di = aj với i /Ai Từ dễ có P d2i

= n2 suy P

|AiAj|= n2

, mà |AiAj| ≤1 suy |AiAj|= với i, j

Lời Bình Phương pháp sử dụng tốn tạm gọi tên phương

pháp độc lập chuyển động, ý tưởng phải khảo sát nhiều đối tượng tạm cố định số biến lại tận dụng tính độc lập (bình đẳng) biến khảo sát số biến Việc tổng hợp "chuyển động" vừa khảo sát cho đánh giá đầy đủ tất biến ban đầu Dưới hai toán minh họa: Bài toán 7.9.6. Xét số nguyên dương n, k, m thoả mãn n > 2k. S là tập gồm tập con k phần tử của {1,2, , n} mà với tập con k+ phần tử khác rỗng của {1,2, , n}

chứa đúng m phần tử của S.

Bài toán 7.9.7. Tại họp có 12k người, người trao đổi lời chào với đúng 3k+ người khác, với hai người đó, số người trao đổi lời chào với hai người là như Xác định k.

(95)

Chương 8

Góc màu

Nguyễn Quốc Khánh & Lê Hồng Quý

Giới thiệu. Trong Tốn Học việc tìm khái niệm giúp nhìn nhận kết cũ cách rõ ràng sâu sắc Hơn nhờ tạo kết mẻ, ý nghĩa Trong viết này, muốn giới thiệu với bạn khái niệm có nhiều ứng dụng tổ hợp, khái niệm góc màu

8.1 Khái niệm góc màu

Chúng ta bắt đầu với số ví dụ

Bài tốn 8.1.1. Một đa diện lồi khơng gian có tất mặt hình tam giác. Đối với cạnh AB bất kì đa diện, đánh dấu mũi tên theo chiều từ A đến B hoặc từ B đến A sao cho đỉnh có mũi tên vào mũi tên đi Chứng minh tồn mặt ABC của đa diện mà mũi tên các cạnh mặt theo chiều.

a

lời giải Gọi D, M, C lần lượt số đỉnh, số mặt số cạnh đa diện lồi giả

thiết Bây đưa khái niệm: góc ABCd gọi góc màu hai cạnh AB, BC đánh dấu mũi tên theo cách có mũi tên vào B từ B Giả sử phản chứng có cách vẽ mũi tên mà khơng có mặt đa diện có mũi tên chiều Đếm số góc màu cách tơ theo hai cách

(96)

Chúng ta biết mặt đa diện hình tam giác, mà theo giả sử phản chứng mũi tên cạnh tam giác khơng chiều Do mặt có góc màu Và có tổng cộngM góc màu

Mặt khác xét đỉnh đa diện, giả sử đỉnh có x mũi tên y

mũi tên vào với xy Theo giả thiết thìx, y≥1 Số góc màu đỉnh xy Chú ý thêm xy−(x+ 1)(y−1) =xy+ 1≥ Do đỉnh bậc n (tức có n cạnh có đầu mút đỉnh này) có n−1 góc màu Do số góc màu khơng bé 2CD

Vậy ta có bất đẳng thức M ≥ 2CD Ngoài biết hệ thức Euler đa diện lồiD+M =C+ Kết hợp hai kiện này=⇒D+M ≥2C =⇒2≥C Bất đẳng thức rõ ràng sai nên giả sử phản chứng sai Như điều phải chứng minh Bài tốn chứng minh xong hồn tồn

Lời Bình Bài tốn kết đáng ngạc nhiên cách phát biểu nó

rất giản dị Cách giải sử dụng khái niệm góc màu Chúng thực ngồi cách giải vừa trình bày liệu có cịn lời giải sơ cấp đẹp đẽ khác hay khơng Theo cách đó, nói ví dụ đẹp để thể cho khái niệm mẻ Tuy nhiên ví dụ đẹp khơng phải lúc làm trịn nhiệm vụ Chính chúng tơi muốn bạn quay lại với hai toán cổ điển sau Một đề thi IMO năm 1998, xuất kì thi IMO từ cách 20 năm Chúng ta vào chi tiết2 toán

Bài tốn 8.1.2. Trong thi có a thí sinh và b giám khảo, với b là số nguyên dương lẻ, b ≥ Mỗi giám khảo đánh giá thí sinh theo hai mức rớt đậu Gọi k là số ngun dương có tính chất lấy giám khảo tuỳ ý đánh giá của vị giám khảo có kết trùng nhiều cho k thí sinh Chứng minh rằng:

k a

b−1 2b .

lời giải Trước hết ta dịch tốn ngơn ngữ dồ thị, ngơn ngữ thích hợp với khái

niệm góc màu Mỗi giám khảo thí sinh coi điểm Khi với giám khảo A thí sinh B, tô đỏ đoạn thẳng AB giám khảo A cho thí sinh

B đậu, tơ màu xanh trường hợp ngược lại Lại đếm số góc màuS theo hai cách

Chúng ta biết cặp giám khảo có chung đánh giá nhiều k thí sinh, có khơng q k góc màu có hai đầu mút hai cạnh góc hai giám khảo Có tất C2

b cặp giám khảo, ta có:

Skb(b−1)

2 . (8.1)

Bây xét thí sinh Giả sử có x giám khảo cho thí sinh đậu cịn lại bx

giám khảo cho rớt Như số góc màu nhận thí sinh làm đỉnh góc là:

x(x−1) +

(bx)(bx−1) =

(97)

8.1 KHÁI NIỆM GÓC CÙNG MÀU 97

= (xb

2)

+b

4 −

b

2 ≥

b2

4 −

b

2 =

(b−1)2 −

1 4.

Chú ý blà số nguyên lẻ (b−1)2/4 là số nguyên, số góc màu nhận thí sinh làm đỉnh góc khơng bé (b−1)2/4 Và đó:

Sa(b−1)

2

4 . (8.2)

Kết hợp hai kết vừa thu (8.1) (8.2) ta suy rằng:

kb(b−1) ≥

a(b−1)2 =⇒

k a

b−1 2b .

Bài toán 8.1.3. Giả sử k, n là hai số nguyên dương và S là tập hợp n điểm mặt phẳng thoả mãn tính chất ba điểm số khơng thẳng hàng, với điểm P ít nhất k điểm phân biệt của S cách đều P Chứng minh rằng:

k <

2+

2n.

lời giải Chúng ta quy ước góc gọi màu góc có hai cạnh nhau.

Ta ước lượng tổng số góc màu T tạo tập hợp S theo hai cách

Một mặt với điểm Sk điểm phân biệt S cách nó, đỉnh có khơng C2

k góc màu Vì ta có đánh giá:

TnCk2. (8.3)

Mặt khác cặp hai điểmA, BS có khơng q 3góc màu nhận hai điểm làm đầu mút hai cạnh Thật vậy, giả sử phản chứng AXB,d AY B,d AZBd góc màu phân biệt Khi rõ ràng điểm X, Y, Z nằm đường trung trực AB, chúng thẳng hàng Điều trái với giả thiết toán tậpS khơng có điểm thẳng hàng Vậy cặp điểm A, B S có nhiều góc màu AXB,d AY Bd nhậnA, B làm đầu mút hai cạnh góc Từ ta có:

T ≤2Cn2. (8.4)

Kết hợp hai đánh giá (8.3),(8.4) vừa thu ta có2Cn2 ≥nC

2

k suy ra:

n(n−1) ≥ nk(k−1)

2 =⇒k 2−

k−1

4 <2n−2<2n =⇒ (k

2)

<2n =⇒k <

2+

(98)

Rõ ràng ý tưởng để giải hai tốn thực khơng Tuy nhiên nhờ sử dụng khái niệm góc màu mà lời giải trở nên vô sáng sủa Điều cho thấy khái niệm góc màu thực tiềm ẩn suy nghĩ từ lâu, hai tốn có vai trị cụ thể hố ý tưởng Và kết cụ thể hố thật thú vị, nhờ có khái niệm góc màu mà hai tốn cổ điển nhìn nhận theo cách nhẹ nhàng đẹp đẽ

Về mặt tốn học góc màu hiểu đơn giản ba đối tượng có kèm theo số tính chất Và tồn cơng việc ghép số đối tượng thành khảo sát tính chất theo nhiều cách khác Các bạn sử dụng khái niệm để giải số toán sau đây:

Bài tốn 8.1.4 (IMO Shortlist 1986). Cho5 số có 100 chữ số tạo thành bởi Ta xếp số thẳng theo hàng đơn vị, hàng chục, hàng trăm, Biết hai số trong số có chung nhất r hàng hàng sau xếp chứa đủ hai chữ số Chứng minh 40≤r≤60

Bài tốn 8.1.5. Cho bảng vng T kích thước (n2 +n+ 1).(n2+n + 1) với n là số nguyên dương Hãy tìm số tự nhiên k lớn cho tơ màu k ô vuông đơn vị của

T mà số tơ khơng có ơ có tâm tạo thành bốn đỉnh hình chữ nhật.

Bài toán 8.1.6 (IMO Shortlist 2004). Trong trường đại học có n sinh viên với n một số tự nhiên lẻ Các sinh viên tham gia câu lạc (một sinh viên tham gia nhiều câu lạc bộ) Các câu lạc kết hợp với tạo thành hội đồng (mỗi câu lạc thuộc nhiều hội đồng khác nhau) Có đúng k hội đồng giả sử thêm rằng: (i) Mỗi cặp sinh viên thuộc câu lạc bộ.

(ii) Với sinh viên hội đồng sinh viên thuộc câu lạc trong hội đồng.

(iii) Mỗi câu lạc có lẻ sinh viên Và câu lạc đó 2m+ sinh viên thuộc về đúng m hội đồng.

Hãy tìm tất giá trị của k.

Bài tốn 8.1.7 (IMO 2005). Trong kỳ thi có n thí sinh có bài tốn Biết rằng khơng có thí sinh giải hết bài với hai tốn có nhiều hơn 2n/5 thí sinh giải hai tốn Chứng minh có tí hai thí sinh giải được bài toán.

Thay cho lời kết Một khái niệm giúp soi sáng kết cũ Tuy

nhiên khơng phải tất cả, việc tạo khái niệm đem so sánh với vai trị cơng cụ Cơng cụ giúp ta cơng phá vấn đề khó đặt từ trước khái niệm đời Trong viết sử dụng kết kinh điển khác để thể điều cách cụ thể hơn, kết mà sử dụng thường biết tên toán người Dưới cách phát biểu tốn đó:

Bài tốn 8.1.8 (Bài tốn người). Trong nhóm học sinh trường ln có thể chọn nhóm 3học sinh mà học sinh học lớp học ở3 lớp khác nhau.

(99)

8.2 MỞ RỘNG BÀI TOÁN6 NGƯỜI 99

8.2 Mở rộng tốn 6 người

Trong viết tơi sử dụng khái niệm góc màu đề cập tới kỳ trước để giải toán mở rộng toán 6người cổ điển Đây phần nhỏ lớp rộng toán dạng T uran

Bài toán 8.2.1 (Bài tốn người). Trong nhóm học sinh trường ln có thể chọn nhóm3 học sinh mà học sinh học lớp học ở3 lớp khác nhau.

lời giải Ta dịch tốn ngơn ngữ dồ thị, 6 thí sinh tương ứng với 6 điểm phân biệt

trong khơng gian với điều kiện khơng có bộ3điểm chúng đồng phẳng Khi với2 điểm bất kìA, Bta tô đỏ đoạn thẳng ABnếuA, Bhọc lớp tô màu xanh trường hợp ngược lại Cần chứng minh với cách tơ tồn tam giác có ba cạnh tô cùng1màu

Giả sử phản chứng có cách tơ mà cách tơ không tồn tam giác màu Gọi6điểm làA, B, C, D, E, F,5cạnh xuất phát từ đỉnhAAB, AC, AD, AE, AF

được tô bằng2 màu đỏ xanh Theo nguyên lýDirichlet tồn nhất3 cạnh tô màu Giả sử AB, AC, ADcùng màu đỏ, do3tam giácABC, ACD, ADB tam giác màu suy BC, CD, DB phải tô màu xanh, tam giác BCD lại tam giác màu Mâu thuẫn chứng tỏ giả sử phản chứng sai Nghĩa với cách tơ tồn tam giác màu

Có kết mạnh nhiều, với cách tơ tồn hai tam giác màu Với cách thức chứng minh kết phức tạp phải xét xét nhiều trường hợp Cần ý thêm toán sau hồn tồn sử dụng suy luận kiểu để giải Bài toán 8.2.2. Trong không gian cho 7điểm với giả thiết chúng khơng có 3điểm nào đồng phẳng Tơ màu tất đoạn nối điểm số điểm hai màu xanh đỏ Chứng minh với cách tô ta tìm nhất tam giác có ba cạnh tơ màu.

Tất nhiên không vào chi tiết, sau giải tốn điểm, lại chẳng xuất toán 8,9 hay nhiều điểm Một người làm tốn thơng minh chắn khơng giải tốn riêng lẻ kiểu mà họ đặt tốn tổng qt tìm cách giải tổng quát Hãy đặt toán tổng quát để cơng nó, nhớ giải tốn tổng qt đơi dễ dàng việc cố tìm lời giải trường hợp riêng lẻ Với trường hợp lời khun tốt

Bài tốn 8.2.3. Trong khơng gian cho n điểm mà chúng khơng có điểm đồng phẳng Tô tất đoạn thẳng nối điểm số điểm hai màu xanh đỏ Tìm số lớn nhất f(n) sao cho với cách tô tìm ít nhất f(n) tam giác màu.

(100)

Để giải toán toán rõ ràng phép suy luận "ngây thơ" không đem lại hiệu Chúng ta sử dụng khái niệm góc màu để đưa lời giải sáng sủa Cần phải nhắc lại góc ABCd gọi màu cạnh AB, BC tô màu Gọi tổng số góc màu cách tơ T, ta ước lượng T theo hai cách sau

Giả sử cách tơ có x tam giác màu, tam giác cho góc màu, tam giác khác cho 1góc màu Do ta có:

T = 3x+ (Cn3−x) =Cn3+ 2x. (8.5) Bây ta ước lượng số góc màu theo cách khác Giả sử đỉnh co a

cạnh tô màu đỏ b cạnh tô màu xanh Số góc màu đỉnh C2

a +Cb2 Nếu |ab|>1, giả sử a > b+ 1, đó:

(Ca2+Cb2)−(Ca2−1+Cb2+1) = (Ca2−Ca2−1)−(Cb2+1−Cb2) =Ca1−1−Cb1

=ab−1>0.

Từ bất đẳng thức suy số góc màu đỉnh nhỏ số lượng cạnh đỏ số lượng cạnh xanh xuất phát từ đỉnh chênh không Do ta xét hai trường hợp n chẵn n lẻ

1. Nếu n chẵn

Đặt n= 2k số góc màu đỉnh nhỏ là:

Ck2+Ck2−1 = (k−1)2 =⇒Tn(k−1)2. (8.6) Kết hợp hai bất đẳng thức (8.5),(8.6) dẫn tới:

n(n−1)(n−2)

1.2.3 + 2xn(k−1)

.

Ta biến đổi sau

x

2

2k(k−1)2 −2k(2k−1)(2k−2)

6

= k(k−1)

6 6(k−1)−2(2k−1) = k(k−1)(k−2)

3 =

n(n−2)(n−4) 24 .

(101)

8.2 MỞ RỘNG BÀI TOÁN6 NGƯỜI 101

2. Nếu n lẻ

Đặt n= 2k+ đỉnh có 2C2

k =k(k−1) góc màu đó:

Tnk(k−1). (8.7)

Kết hợp hai bất đẳng thức (8.5),(8.7), biến đổi tương tự ta thu được:

x

2

(2k+ 1)k(k−1)−(2k+ 1).2k(2k−1)

6

=

2(2k+ 1)k

(k−1)− 2k−1

3

= (2k+ 1).k(k−2) . 2.1. Nếu k chẵn

Nếu k chẵn biểu thức cuối số nguyên, ta trường hợp đẳng thức: đặt k= 2t+ thìn= 4t+ Khi tôAaAb màu đỏ số dư phép chia|ab|chon

không lớn t lớn 3t tô xanh trường hợp ngược lại Nếu xếp n điểm đỉnh n giác rõ ràng ta tô đỏ 2t đoạn nối đỉnh A với t đỉnh bên trái vàt đỉnh bên phải nó, ngồi 2t cạnh cịn lại đuợc tơ màu xanh Các ước lượng lại trở thành đẳng thức

2.2. Nếu k lẻ

Đặt k= 2t+ thìt= 4k+ 3, lúc biểu thức cuối khơng cịn số ngun nữa, xác có dạngy12 Bây ta chứng minh giá trị nhỏ xy+ 1bằng cách cách tơ thích hợp Để làm điều cố gắng tận dụng cách tô trường hợp ý đến tính đối xứng phép tơ Cụ thể, ta tiến hành cách tơ sau, cách tơ có dựa chút vào trường hợp trước

Đầu tiên ta lấy 4t + đỉnh tô để từ đỉnh có 2t cạnh xanh 2t cạnh đỏ Chọn điểm X 4t+ điểm chia 4t điểm lại thành tập S1, S2 Lấy điểm A, B, ta tiến hành tô theo nguyên tắc sau:

    

As1 đỏ, Bs1 xanh với điểm s1 ∈S1

Bs2 đỏ, As2 xanh với điểm s2 ∈S2

(102)

Khi đó4t+ 3điểm (trừ điểmX) có tính chất từ điểm có đúng2t+ 1cạnh xanh 2t+ cạnh đỏ Riêng đỉnh X có 2t+ cạnh xanh 2t cạnh đỏ đỉnh X dơi góc màu so với đỉnh khác Thêm góc dơi vào ước lượng vế phải cuối có thêm1/2đơn vị Lúc rõ ràng làx=y+1 Điều phải chứng minh

Cuối ta tổng kết lại kết vừa thu được:

f(n) =           

n(n−2)(n−4)

24 n chẵn

n(n−1)(n−5)

24 n chia4 dư

n(n−1)(n−5) 24 +

1

2 n chia4 dư Vậy tốn 8.2.3 giải hồn tồn

Lời Bình Bài tốn 8.2.1khơng phải q khó để chứng minh, thực có ý nghĩa

và mở nhiều vấn đề thú vị Chẳng hạn n điểm cho, tìm số t(n) nhỏ để với cách chọn t(n) cạnh để tơ màu đỏ ln tìm tam giác màu Đây nội dung tốn T uran cổ điển, bạn chứng minh

t(n) = [n2/4] khơng q khó khăn Một câu hỏi khác, tìm số k(n) nhỏ cho với cách chọn rak(n) cạnh để với cách tơ cạnh 2màu xanh đỏ tìm tam giác màu Đây thực tốn hay khó Trọng phần cuối viết giới thiệu cách chứng minh với bạn n = 5p+q với 0< q ≤5 thì:

f(n) = n(n−1) −

5p(p−1)

2 −pq+ 1.

Thay cho lời kết.Việc đếm số lượng góc màu thực chất đếm số ba thoả

mãn tính chất đó, liệu có thực cần tới khái niệm góc màu hay không Câu trả lời để diễn đạt lời giải hồn chỉnh khơng thực cần thiết Tuy nhiên việc sử dụng khái niệm góc màu cách tư giúp cho chuyện rõ ràng hợp lý Và bạn thấy, khó toán 8.2.3sẽ trở nên dễ chịu nhiều sử dụng khái niệm góc màu Điều quan trọng trước bắt tay vào tính tốn, ta cần phải có niềm tin kết chắn đạt

(103)

8.3 PHƯƠNG PHÁP HÀM ĐẾM VÀ VÀI ỨNG DỤNG 103

8.3 Phương pháp hàm đếm vài ứng dụng

Trong viết thứ chúng tơi nói góc màu thực tên đặt cho việc nhóm số đối tượng lại với việc khảo sát tính chất theo nhiều cách tư tưởng phương pháp góc màu Trong viết chúng tơi tiếp tục giới thiệu hình thức hố tất điều Trong số tốn tổ hợp, lập luận thông thường chưa đem lại hiệu quả, thêm vào hàm đếm để tiếp xúc với chất vấn đề cách tường minh

Bài toán 8.3.1 (TST VMO 2000). Trong mặt phẳng cho 2000 đường tròn bán kính đơn vị cho khơng có hai đường tròn tiếp xúc đường tròn cắt hai đường trịn khác Tìm giá trị nhỏ số giao điểm tạo trường trịn này.

lời giải Kí hiệu χ là tập hợp đường tròn, S là tập hợp giao điểm đường

tròn cho Với cặp XS (C)∈χ ta xét hàm đếm:

f(X, C) = (

0 X /∈(C)

kk đường trịn qua X X ∈(C).

Với cách đặt ta nhận thấy với XS hàm f có tính chất: X

Cχ

f(X, C) = 1. (8.8)

Mặt khác với đường tròn(C)thuộcχ Do số đường tròn số giao điểm hữu hạn, suy tồn giá trị nhỏ số f(X, C) Giả sử giá trị nhỏ đạt X0 và:

f(X0, C) =

k0

(k0 ∈N∗).

k0−1 đường trịn(Ci) (vớii= 1,2, , k0−1)) khác (C) qua X0 Mặt khác đường trịn khơng tiếp xúc có bán kính nên k0 −1 đường trịn cắt (C)

k0−1 điểm phân biệt khác X0 Suy theo cách chọnX0 ta có: X

XS

f(X0, C)≥

k0

+ (k0−1)

k0

với (C)∈χ. (8.9)

Kết hợp (8.8) (8.9) suy ra:

|S|=X

XS

X (C)∈χ

f(X, C) = X (C)∈χ

X

XS

f(X, C)≥ |χ|= 2000.

Mặt khác xét 500 nhóm, nhóm gồm đường trịn bán kính mà hai đường trịn thuộc hai nhóm khác khơng cắt Khơng có hai đường trịn tiếp xúc nhóm có đường trịn có tâm đỉnh tam giác cạnh

3 đường tròn thứ tư có tâm tâm tam giác nói Dễ thấy với 2000đường trịn chúng thỏa mãn giả thiết có 2000 giao điểm

(104)

Bài toán 8.3.2. Xét tậpPn={(a1, a2, , ak) |1≤a1 ≤a2 ≤ .ak & k

P

i=1

ai =n, kn}

trong đó n là số nguyên dương Với phần tử π = (a1, a2, , ak)∈ Pn, gọi F(π)là số số

1 trong π G(π) là số số phân biệt trong π Chứng minh rằng: X

π∈Pn

F(π) = X

π∈Pn

G(π).

lời giải Đặtpn=|Pn|vàp0 = Ta chứng minh P

π∈Pn

G(π) =p0+ .+pn−1 = P

π∈Pn

F(π)

Để chứng minh vế trái đẳng thức kép ta xét hàm đếm:

χ(π, m) =

0 m /π

1 mπ.

Vớiπ ∈ Pn cố định n

P

i=1

χ(π, m) =G(π) =⇒ P π∈Pn

G(π) = P

π∈Pn

n

P

m=1

χ(π, m) =

n

P

m=1 P

π∈Pn

χ(π, m)

Mà ta lại có P

π∈Pn

χ(π, m) =pnm =⇒

P

π∈Pn

G(π) =

n

P

m=1

pnm =p0 +p1+ .+pn−1

Bây ta chứng minh vế phải đẳng thức kép nhờ quy nạp theo n Với n = hiển nhiên Giả sử khẳng định tớin−1≥1 Ta chứng minh vớin Thật vây, ta có P

π∈Pn

F(π) = P 1∈π∈Pn

F(π)

Xét ánh xạ: f :π = (a1, a2, , ak)→ψ= (a2, a3, , ak) Suy ψ∈ Pn−1 Dễ có f song

ánh, từ suy ra: X

π∈Pn

F(π) = X

ψ∈Pn−1

F(1, ψ) = X

ψ∈Pn−1

(1 +F(ψ)) =|Pn−1|+ X

ψ∈Pn

F(ψ) =p0 +p1+ .+pn−1.

(ở bước cuối ta sử dụng giả thiết quy nạp) Như theo ngun lí quy nạp ta có đẳng thức với mọin Bài toán chứng minh xong

Với phương pháp ta giải toán thú vị sau đây:

Bài toán 8.3.3 (IMO 2001). Trong kì thi Tốn có 21 nam và 21 nữ tham gia Biết rằng thí sinh giải khơng q bài tốn với cặp nam-nữ, có tốn được giải hai thí sinh Chứng minh có tốn mà giải nhất nam và nữ.

(105)

8.4 MỞ RỘNG MỘT ĐỀ THI IMO 1992 105

8.4 Mở rộng đề thi IMO 1992

Tiếp theo tinh thần viết trước, với hiểu biết góc màu hàm đếm, viết giới thiệu với bạn lời giải cho kết đề cập tới mở rộng toán người Đây mở rộng toán tổ hợp xuất kỳ thi IMO năm1992

Bài toán. Cho đồ thị đầy đủ n đỉnh Chọn k = f(n) cạnh tuỳ ý tô chúng hai màu xanh đỏ Hỏi k nhỏ để phương án tô tạo tam giác có ba cạnh tơ màu

lời giải Chúng ta lưu ý đến kết tốn 6 người: trong 6 người ln ra

được người đôi quen đơi khơng quen Với lưu ý ta phát biểu tốn dạng khác: tìm số Rn để tồn cách tơ màu Rn cạnh đồ

thị đầy đủ n đỉnh mà khơng có điểm rời nhau, cách tơ Rn−1 cạnh

thì bộ6điểm rời ln tồn Và có hệ thức liên hệf(n) = n2−Rn+

Sự tương đương hai cách phát biểu với hệ thức liên hệ suy trực tiếp từ thuật toán (T) sau đây: đồ thịA1, A2, , An có cạnh tơ hai

màu, khơng tơ cả, ta thêm điểmAn+1 ký hiệu phép thêm điểm dựa vào AiT(Ai)

(1≤in) sau:

Đối với f(n) đoạn AiAn+1 khơng tơ, cịn với mọij =6 i 1≤jn thìAiAj

An+1Aj tơ màu

Đối vớiRn AiAn+1 nối nối An+1 với tất điểm nối với Ai

Xuất phát từ đồ thị đầy đủ hình bên, để đạt f(n) ta thực phép tốnT(A1), T(A2), T(A3), T(A4), T(A5) để có đồ thị6,7,8,9,10 đỉnh với f(n)tối thiểu.Tiếp tục thực hiệnT(A1) cho đồ thị10 đỉnh, ta đồ thị 11 đỉnh có f(11) tối thiểu

Đồ thị choRn xem bù màu củaf(n): ta có 5điểm rời nhau, qua5bước

đầu ta có5 cặp điểm đơi nối với nhau, chọn5 điểm cặp thực hiện(T) cho nhận giá trị Rn sau bước nhận điểm đôi

được nối với Lại chọn ra5điểm của5bộ 3và thực hiện(T)cho chúng, sau5bước nhận 5giá trị cuối bộ4 đơi nối với Q trình tiếp tục

Cuối thu công thức sau với n= 5p+q,0< q≤5:

Rn=

5p(p−1)

2 +qp=⇒f(n) =

n(n−1) −

5p(p−1)

(106)

Bây ta cần chứng minh tô Rn− cạnh cho đồ thi n đỉnh

điểm rời tồn tại, giả sử phản chứng

Trường hợp 1: số cạnh tô Rn−1, số đỉnh thuộc cạnh tô Rn−1 với a ≥ 1, ký

hiệu tập đỉnhD Ngồi D cịn có nRn+a đỉnh khác

Nếu D có6 +Rnanđỉnh rời ta có(nRn+a) + (6 +Rnan) =

đỉnh rời nhau, mâu thuẫn

Nếu D +Rnan đỉnh rời nhau, tức cứ6 +Rnan đỉnh

D có cạnh, số cạnh phải là:

g(a) = C

6+Rn−an

Rn−a

(Rna)−2

.

Cần kiểm tra g(Rn+ 3−n)> Rn−1⇐⇒

Cn3−3

n−5 > Rn−1 Thayn = 5p+q ta biến đổi tương đương (5p+q−3)(5p+q−4)

6 >

5p(p−1) +qp

⇐⇒(5p+q−3)(5p+q−4)>15p(p−1) + 6qp

⇐⇒10p(p−2) + (q2+ 12) +q(4p−7) >0.

Bất đẳng thức cuối đúng, số cạnh D lại lớn Rn−1, mâu thuẫn Suy

trường hợp không hợp lý

Trường hợp 2: |D|> Rn−1, hay ký hiệu tập cạnh tơ (C), ta có |D| >|(C)|

(xét n≥11 ta có|(C)| ≥6) Đối với dD c∈(C)ta định nghĩa

f(d, c) =   

0 d /∈(C)

k d∈(C) số lượng cạnh thuộc (C)và chứa dk

=⇒ X c∈(C)

X

dD

f(d, c) =X

dD

X

c∈(C)

(107)

8.4 MỞ RỘNG MỘT ĐỀ THI IMO 1992 107 Theo nguyên lý Dirichlet tồn c1 mà

P

dDf(d, c)≥ |D|/|(C)| >1 c1 chứa

một đỉnh treoA1 Do2.1<|(C)|nên trongDcòn đỉnh nối vớiA1, vậy|D|>|(C)|+ (nếu ngược lại với việc họn A1 điểm rời D cộng với điểm không thuộc vào D cho ta điểm rời nhau, mâu thuẫn) Ký hiệu D2 (C2) tập nhận từD (C) sau bỏ tất thông tin vềA1 Suy ra:

X

c∈(C2) X

dD2

f(d, c)>|(C2)|.

Từ ta lại lấy đỉnh treo A2 rời A1, lại lập luận 2.2<|(C2)| nên lại có: X

c∈(C3) X

dD3

f(d, c)>|(C3)|.

(108)(109)

Phần III

Một số toán khác

(110)(111)

Chương 9 Hình Học

Bài toán 9.1. ChoABC, biết tồn ba điểm X, Y, Z tương ứng thuộc cạnh

BC, CA, AB sao cho AX = BY = CZ BAX = CBY = ACZ Chứng minh rằngABC đều.

Bài toán 9.2. ChoO là điểm nằm tam giác nhọn ABC Các đường tròn có tâm là trung điểm cạnh tam giác qua O cắt điểm thứ hai là M, K, L khác điểm O Chứng minh rằng O là tâm đường tròn nội tiếp củaM KL nếu nếu O tâm đường tròn ngoại tiếpABC.

Bài tốn 9.3. Cho ba đường trịn bán kính1 đơi cắt tạo thành giao điểm, nhưng cả ba đường khơng có điểm chung cả, tức chúng tạo thành lỗ có dạng tam giác cong có ba đỉnh ba số sáu giao điểm Chứng minh đường trịn thứ tư qua ba giao điểm lại có bán kính bé hơn

Bài tốn 9.4. ChoABC với M là trung điểm cạnh BC Điểm PAM thoả mãnP M =

BM = CM Hạ P HBC Kẻ P QBA (QBA) Kẻ P RAC (RAC) Chứng minh đường tròn ngoại tiếpQRH tiếp xúc với BC tại H.

Bài toán 9.5. ĐiểmM nằm tam giácABC Các đường thẳngAM, BM, CM cắt đường tròn ngoại tiếpABC tại A0, B, C0 Gọi r, r0 lần lượt bán kính đường trịn nội tiếp của các tam giác ABC A0B0C0 Chứng minh rằng:

4rr0≤R2 −OM2.

Bài toán 9.6. Giả sử4 đường trịn(O1),(O2),(O3),(O4)có tính chất (O1) tiếp xúc ngồi với (O2), (O2) tiếp xúc với (O3), (O3) tiếp xúc với (O4) (O4) tiếp xúc với (O1) Chứng minh tiếp điểm thuộc đường tròn

Bài tốn 9.7. Cho hình chữ nhật ABCD Về phía ngồi hình chữ nhật cạnh

AB, CD dựng tam giácAM B, CN D thoả mãn điều kiện AM B =6 CN D Đường thẳng

M N cắt cácAB, CD tại P Q, DP cắt BM tại E , BQ cắt DN tại F Hỏi 3đường thẳng

EF, AC, M N có đồng quy khơng.

Bài tốn 9.8. Giả sử đường tròn nội tiếp (I)củaABC tiếp xúc với cạnh BC, CA, AB

(112)

DEF tại M, N, P cà cắt (I) tại X, Y, Z Chứng minh khẳng định sau đây:

(i) AM, BN, CPđồng quy tạiJ. (ii) AX, BY, CZđồng quy tại L. (iii) K, J, Lthẳng hàng.

Bài toán 9.9. ChoA1A2A3 Giả sửPi là điểm cạnh Ai+1Ai+2 sao cho đoạn thẳng

AiPi đồng quy. (I) là đường tròn nội tiếpA1A2A3 ti là tiếp tuyến qua Pi đến (I) khác

với Ai+1Ai+2 Gọi Qi là giao điểm của ti Pi+1Pi+2 Chứng minh rằng điểm Q1, Q2, Q3 thẳng hàng (các số tính theo mod 3)

Bài tốn 9.10. Tứ giác ABCD nội tiếp đường trong (O) Giả sử E I lần lượt giao điểm cặp đường thẳng AB, CD AC, BD Chứng minh rằng:

i) EA.EB=IA.IC+IE2 ii) O là trực tâmEIF.

Bài tốn 9.11. Giả sử nửa đường trịn tâm O đường kính BC cắt hai cạnh AB, AC củaABC tại E D. ED cắt BC tại F. H là trực tâmABC Chứng minh rằng F HAO. Bài toán 9.12. Cho tứ giác nội tiếp ABCD và điểm M nằm cạnh CD sao choADM và tứ giác ABCM có diện tích chu vi Chứng minh hai cạnh nào đó tứ giác ABCD có độ dài.

Bài tốn 9.13. Cho tam giác nhọn ABC có đường cao AD, BE và phân giác trong

AP, BQ Ký hiệu I, O lần lượt tâm đường tròn nội tiếp ngoại tiếp củaABC Chứng minh rằng D, E, F thẳng hàng nếu P, Q, O thẳng hàng.

Bài toán 9.14. Sử dụng AB, AC làm đường kính vẽ phía ngồi tam giác hai nửa đường trịn.AH là đường cao củaABC D là điểm cạnh BC (D 6=B, C) Qua D vẽ

DE k AC, DF kAB (E, F nằm trên nửa đường tròn vẽ) Chứng minh rằng D, E, F, H

cùng nằm đường trịn.

Bài tốn 9.15. Chứng minh rằngA1, A2, , An n đỉnh một n giác trong

hai tính chất sau thoả mãn:

i) Các điểm thuộc đường trịn bán kính và với điểm M nằm hình trịn ta có bất đẳng thức M A1M A2 M An≤2

ii) Đa giác lồi A1A2 An có góc và A1A2 ≤A2A3 ≤ AnA1

Bài toán 9.16. Giả sử X, Y là hai điểm nằm đường trịn (O) H là điểm bất kì nằm đường thẳng XY Một đường thẳng l bất kỳ qua H cắt (O) tại M, N. M X, N Y

cắt (O) tại P, Q tương ứng Chứng minh rằng P Qđi qua điểm cố định khi l thay đổi.

Bài tốn 9.17. Cho trước đường trịn tâmO và đường thẳngd bất kỳ. H là hình chiếu của O

trên d. M là điểm cố định trên (O) A, B thay đổi trên d sao cho H luôn trung điểm của AB. M A, M B cắt (O) tại P, Q Chứng minh rằng P Qluôn qua điểm cố định khi

(113)

113

Bài toán 9.18. Giả sử M, N, P là điểm cạnh BC, CA, AB của tam giác ABC

sao cho bán kính đường trịn nội tiếp tam giác N AP, P BM, M CN bằng bằng nửa bán kính đường trịn nội tiếpABC Chứng minh rằng M, N, P là trung điểm các cạnh BC, CA, AB.

Bài tốn 9.19. Cho hai đường trịn (O1),(O2) cắt ở A, B Tiếp tuyến chuyến chung

EF với E ∈(O1), F ∈ (O2) Một cát tuyếnM N kEF với M ∈(O1), N ∈(O2) M E cắt N F

S Chứng minh rằng SBM =6 SBN.

Bài toán 9.20. ChoABC với trung tuyến AM Về hai phía của A trên đường phân giác trong lấy hai điểm E, F thoả mãn AE =

bc Chứng minh rằng F M C =6 EM C.

Bài toán 9.21. Giả sử E là điểm nằm trung tuyến kẻ từ C củaABC Đường tròn qua E và tiếp xúc với AB tại A cắt AC tại M, đường tròn qua E và tiếp xúc AB tại B

cắtBC tại N Chứng minh đường tròn ngoại tiếp tam giácCM N tiếp xúc với hai đường tròn nói trên.

Bài tốn 9.22. Giả sử A, B, C, D, E, F điểm nằm đường tròn cho AE k

BD, BC k DF Điểm X đối xứng với D qua CE Chứng minh rằng d(X, EF) = d(B, AC), với d(M, N P) là khoảng cách từ điểm M tới đường thẳng N P.

Bài toán 9.23. Xét đường thẳng đồng phẳng, khơng có đường thẳng chúng song song hay đồng quy, khơng có đường thẳng chúng tạo thành tam giác Chứng minh có đường thẳng chúng song song với đường thẳng Eulercủa tam giác tạo bởi đường thẳng cịn lại đường thẳng số4 đường thẳng cho song song với đường thẳngEuler của tam giác tạo bởi 3đường thẳng cịn lại.

Bài tốn 9.24. Lục giác A1A2 A6 cạnh bẳng giả sử ta có đẳng thức A1+ A3+6 A5 =6 A2+6 A4+6 A6 Chứng minh rẳng lục giác có góc đối diện nhau, nghĩa là A1 =6 A4,6 A2 =6 A5,6 A3 =6 A6.

Bài tốn 9.25. Hai đường trịn đường kính nhau (O1),(O2) cắt tại P, Q và hai tâm không nằm phần chung hai đường trịn đó. O là trung điểm P Q Hai đường thẳng AB, CD vẽ qua P (6≡ P Q) sao cho A, C ∈ (O1) B, D ∈(O2) M, N là trung điểm của AD, BC tương ứng Chứng minh rằng M, N, O thẳng hàng.

Bài toán 9.26. Cho trướcABC nội tiếp đường tròn (O) và đường thẳngbất kỳ.a,b,c là ảnh củaqua phép đối xứng trục qua BC, CA, AB Đặt A0 = ∆b ∩∆c, xác

định B0, C0 một cách tương tự Chứng minh rằng AA0, BB0, CC0 đồng quy điểm đồng quy đó trùng với tâm nội tiếp hay tâm bàng tiếp bàng tiếp góc A0 củaA0B0C0 tuỳ theoABC

nhọn hay tù ở A.

Bài toán 9.27. Giả sử (I, r)là đường tròn nội tiếpABC (Oi, ri), i= 1,2,3là đường

trịn tiếp xúc ngồi với (I) và tiếp xúc với trong cạnh củaABC Chứng minh rằng:

r =√r1r2+

r2r3+

(114)

Bài toán 9.28. Giả sử a, b, c là ba đường thẳng song song qua đỉnh A, B, C củaABC Gọi a0, b0, c0 3 đường thẳng đối xứng với a, b, c qua BC, CA, AB tương ứng Chứng minh rằng a0, b0, c0 đồng quy nếu a, b, c song song với đường thẳng Euler củaABC.

Bài toán 9.29. Ngũ giác ABCDE nội tiếp đường tròn tam O và thoả mãn CB = DE. Chứng minh rằng p(ABCDE) ≤ BE +AD+AC trong đó p(ABCDE) là chu vi ngũ giác ABCDE.

Bài toán 9.30.Lấy2điểmP, Qtuỳ ý cạnhBC củaABC Chứng minh rằngr(ABP) =

r(AQC) nếu nếu r(ABQ) = r(AP C) Trong đór(XY Z) chỉ bán kính đường trịn nội tiếpXY Z.

Bài tốn 9.31.ChoABC và điểmPBC Tiếp tuyến đường tròn nội tiếpABP

mà song song vớiBC cắt AP ởQ, AC ởR Chứng minh rằngr(AQR) +r(ABP) =r(ABC) Bài tốn 9.32. Tìm số thựck lớn cho nếu P nằm tam giác nhọn ABC thoả mãn P AB = P BC = P CA AP, BP, CP là tia cắt (P BC),(P CA),(P AB) tại

A1, B1, C1 tương ứng thì S(A1BC) +S(B1CA) +S(C!AB)≥kS(ABC)

Bài tốn 9.33. Cho tứ giác ABCD có hai đường chéo ACBD Trung trực AB cắt trung trựcCD tại O nằm tứ giác Chứng minh hai tam giácABO, CDO có diện tích bằng tứ giác ABCD nội tiếp.

Bài tốn 9.34. Cho trước đường trịn đường kính AB, nửa đuờng tròn ta chọn n

điểm P1, P2, , Pn sao cho P1 nằm giữa A P2, P2 nằm giữa P1 P3, , Pn nằm giữa

Pn−1 B Tìm điểm C trên nửa đường trịn cịn lại cho tổng diện tích tam giác

CP1P2, CP2P3, , CPn−1Pn là lớn nhất.

Bài toán 9.35. Giả sử điểm E nằm trongABC thoả mãn EBA=6 ECA Gọi M, N hình chiếu của E trên phân giác ngồi đỉnh A Chứng minh rằng M N đi qua trung điểm của BC.

Bài toán 9.36. Cho trướcABC và gọi M là tiếp điểm đường tròn nội tiếp (I) tam giác cạnh BC điểm NBC Chứng minh tồn đường tròn tiếp xúc với cả đường tròn nội tiếp tam giác BM N, M N A, CAN.

Bài toán 9.37. Tứ giác ABCDcó tính chấtAB.CD=AD.BC GọiH, K là trung điểm của

AC, BD Chứng minh nếu BD là phân giác AKC thì suy ra AK+KC =BH+HD. Bài toán 9.38. Giả sử tứ giác ABCD ngoại tiếp đường trịn tâmI và thoả mãn(IA+IB)2+ (IC+ID)2 = (AB+CD)2 thì ABCD là hình thang cân.

Bài tốn 9.39. Cho trướcABC nội tiếp đường tròn (S) (O) nằm góc BAC tiếp xúc với AB tại P, AC tại Q, đồng thời tiếp xúc với (S) Chứng minh trung điểm của P Q là tâm đuờng trịn nội tiếpABC.

Bài tốn 9.40. Trên cạnh BC, CA, AB củaABC ta lấy điểm X, Y, Z. Chứng minh bất đẳng thức:

1

S(AY Z) +

S(BZX) +

S(CXY) ≥

(115)

115

Bài toán 9.41. Giả sử (I),(O) là đường tròn nội tiếp đường tròn ngoại tiếp củaABC. Các tiếp điểm của (I) tại BC, CA, AB lần lượt là D, E, F Vẽ đường tròn ωa, ωb, ωc tiếp

xúc với (I),(O) tại D, K đối với ωa, E, M đối với ωb F, N đối với ωc.

i) Chứng minh rằng DK, EM, F N đồng quy tại P.

ii) Chứng minh trực tâmDEF nằm trên OP.

Bài tốn 9.42. Trên mặt phẳng cho hai đường trịn (O1),(O2) cắt hai điểm A, B. Tiếp tuyến của (O1) tại A B cắt tạiK Cho điểm M tuỳ ý(6=A, B) trên (O1) Đường thẳng M A cắt (O2) lần tại P Đường thẳng M K cắt (O1) lần tại C Đường thẳng CA cắt (O2) lần tại Q Chứng minh trung điểm của P Q nằm đường thẳng M C và đường thẳng P Q đi qua điểm cố định khi M di chuyển trên (O1)

Bài toán 9.43. Cho trước lục giác lồi ABCDEF Lấy sáu trung điểm A1, B1, C1, D1, E1, F1 của cạnh AB, BC, CD, DE, EF, F A Ký hiệu p p1 lần lượt chu vi hai lục giác lồi nói Giả sử tất góc lục giác A1B1C1D1E1F1 đều Chứng minh rằng:

p ≥ √

3 ·p1.

Bài toán 9.44. Tam giác ABC nội tiếp đường trịn tâm O có ba đường cao là

AH, BK, CL Lấy A0, B0, C0 lần lượt trung điểm đường cao Đường trịn nội tiếp với tâm I củaABC tiếp xúc với cạnh BC, CA, AB lần lượt tại D, E, F Chứng minh bốn đường thẳng A0D, B0E, C0F, OI đồng quy Khi OI ta coi như IO là đường thẳng tuỳ ý quay quanh O.

Bài toán 9.45. Trên cạnh củaABC lấy điểm M1, N1, P1 sao cho đường thẳng

M M1, N N1, P P1 đều chia chu vi củaABC thành phần nhau, đó M, N, P trung điểm cạnh BC, CA, AB Chứng minh đường thẳng M M1, N N1, P P1 đồng quy tại K và chứng minh:

max

KA BC,

KB CA,

KC AB

≥ √1

3.

Bài toán 9.46. Giả sửABC là tam giác có góc C nhọn, lấy H là chân đường cao hạ từ

A, M là trung điểm của BC, x, y là đường chia góc A (khi BAx= xAy =6 yAC), N, P lần lượt giao điểm trung trực cạnhBC và tia x, y Tìm tất cácABC

có tính chất AB=N P = 2HM.

Bài tốn 9.47. Hai đường trịn cắt tại A, B Gọi l là tiếp tuyến của đường tròn, với các tiếp điểm là P, T Tiếp tuyến đường ngoại tiếpAP T tại P T cắt tại

S Lấy H là điểm đối xứng điểm B qua đường thẳng l Chứng minh rằng A, S, H thẳng hàng.

Bài tốn 9.48. Hai đường trịn (C1),(C2) cắt tại P, Q Tiếp tuyến chung chúng (gần P hơn Q) tiếp xúc (C1) tại A (C2) tại B Tiếp tuyến của (C1) (C2) tại P cắt đường tròn tại E, F 6= P Lấy H, K là điểm tia AF, BE thoả mãn

(116)

Bài toán 9.49. Cho trước đường trong A bấn kính đường trịn B đi qua tâm của A

và tiếp xúc với A Ký hiệu H là họ đường tròn C tiếp xúc với B và tiễp xúc trong với A Giả sử C, C0 là hai đường tròn trong H với bán kinh tương ứng là p, p0 Với số nguyên n > thì tồn dãy n đường tròn C =C1, C2, , Cn= C0 thoả mãn Ci tiếp xúc với

Ci+1 với 1≤in−1 nếu nếu:

(pp0)2 = (n−1)2(2p+ 2p0−(n−1)2 −8).

Bài tốn 9.50. ChoABC Một đường trịn tiếp xúc với (O) và tiếp xúc với AB, AC tại

M1, N1 Các điểm M2, N2, M3, N3 được định nghĩa tương tự Chứng minh đoạn thẳng

M1N1, M2N2, M3N3 đồng quy trung điểm đoạn.

(117)

Chương 10 Giải Tích

Bài toán 10.1. Xét dãy số thực {an} thoả mãn am+nan+am với số m Hỏi có

tồn tại lim

n→+∞

an

n hay khơng?

Bài tốn 10.2. Xét hai dãy số {un} {vn} thoả mãn:

      

u0>0

un+1 =une−1/u

2

n

vn=

u3

ne

1/u2

n

n

Tìm giới hạn dãy {vn}.

Bài toán 10.3. Cho số tự nhiên m Với số tự nhiên n, ký hiệu an là số số trong cách

viết của n trong hệ số m Đặt:

S(m) =

n

X

j=1

(m−1)(m2 −m−1)aj

j3 .

Chứng minh rằng S(m)→0 khi m →0

Bài toán 10.4. Giả sửP là đa thức hệ số thực có hệ số amà a

π/ Q Chứng

minh tập {sin(f(n))|nN} trù mật trong [−1,1]

Bài toán 10.5. Dãy số a1, a2, , an thoả mãn với số thực δ >1:

lim

n→∞a[δ

n] = 0.

Chứng minh rằng:

lim

n→∞an= 0. Bài toán 10.6. Xét hai dãy số {an},{bn} xác định bởi:

    

a0 =b0 =

an+1=an+bn với số tự nhiên n.

bn+1 =an+ 3bn

(118)

Tính:

lim

n→∞

an

bn

.

Bài tốn 10.7. Giả sử ta có đẳng thức sau với n tự nhiên tuỳ ý:

(1 +

2 +

3)n=pn+rn

2 +sn

3 +tn

6.

Trong đó pn, rn, sn, tnZ Tìm giới hạn sau khi n tiến đến vô cùng:

limrn

pn

limsn

pn

limtn

pn

.

Bài toán 10.8. Giả sử P là đa thức hệ số nguyên bất khả quy với hệ số bậc cao nhất P(0) khơng có ước số phương Với nghiệm phức của P modun lớn hơn Chứng minh đa thức h(x) =f(x3) cũng bất khả quy.

Bài toán 10.9. Cho hàm tuần hoàn f :RR khác số liên tục điểm nào đó Chứng minh rằng f tuần hồn có chu kì sở.

Bài tốn 10.10. Tính giới hạn dãy số {un} sau đây:

un= n

X

k=0

cos√

n+k −1

.

Bài toán 10.11. Cho dãy số {xn} với điều kiện xn+1 =

x2n−1

2xn

n ≥0 Tìm giá trị x0 sao cho |xn| ≤1 ∀n.

Bài toán 10.12. Chứng minh rằng { n

10k|n, kN} trù mật trong (0,+∞)

Bài toán 10.13. Tìm điều kiện cần đủ để tập hợp điểm A ={({},{})|nN}

trù mật hình vng đơn vị [0,1]×[0,1] Bài tốn 10.14. Cho số thực x >1 thoả mãn lim

n→∞x

x{x}n= 0 thì xZ.

Bài tốn 10.15. Cho dãy số {xn} với x1 = 1, xn+1 =

n xn

+xn

n với n ≥1 Chứng minh

tồn tại N sao cho với mọi n > N ta có nxnn+

Bài tốn 10.16. Cho 0< cn<1 ∀n Chứng minh đặt:

(

xn= (1−c1)(1−c2) (1−cn)

yn = (1 +c1)(1 +c2) (1 +cn)

tbì khẳng định sau tương đương với nhau: (1)

∞ X

n=1

(119)

119

Bài tốn 10.17. Tính giới hạn sau đây:

lim

n→+∞

2+ sin

2 π sin

1/nπ

7 + sin 2π

7 sin 1/n 2π

7 + sin 3π

7 sin 1/n3π

7 n

.

Bài toán 10.18. Các số dương c1, c2, , cm gọi thông ước với nếu:

ci

cj

Q với số i, j.

i) Cho dãy tăng số thực dương {an} Với số thực Ai 6= với i= 1,2, , n xét hàm số

sau đây:

f(x) =

n

X

i=1

Aicosaix.

Giả sử f(x) là hàm số tuần hoàn, chứng minh số {an} thông ước.

ii) Với n, mN và số thực Ai Bj khác với i = 1, n j = 1, m Các số dương {a1, a2, , an} đôi khác nhau, số dương {b1, b2, , bm} cũng đôi khác nhau.

Xét hàm số g(x) sau đây:

g(x) =

n

X

i=1

Aicosaix+ n

X

j=1

Bicosbix.

Chứng minh hàm số g(x) tuần hồn thì {a1, a2, , an, b1, b2, , bm} là số thông

ước với nhau.

Bài toán 10.19. Với số nguyên dương n gọi f(n) là số chữ số trong cách viết của n

dưới dạng thập phân Cho a >0 Chứng minh rằng: ∞

X

n=1

af(n)

n2 <∞ ⇐⇒a <91.

Bài toán 10.20. Cho dãy {an} với a1 =x an+1 =

a2

n+y

2 với mọi n Tìm tất cặp số (x, y)để dãy số {an} hội tụ.

Bài toán 10.21. Giả sử = x0 < x1< < xn < xn+1 = và:

n+1 X

j=0,j6=i

1

xixj

= với mọi 1≤in.

Chứng minh rằng xi+xn+1−i = với mọi 0≤in+

Bài toán 10.22. Xét dãy số thực dương an > với n = 1,2,3, Đặt Sn =a1+a2+

+an Chứng minh với mọi n ≥2 ta có:

an+1 ≤

Sn+1

(Sn−1)an+an−1

thì suy ra:

lim

(120)

Bài toán 10.23. Cho0< x1 <1 vàxn+1 =xn+

xn

n

2

(n≥1) Chứng minh dãy có giới hạn là a và chứng minh rằng:

limn(axn) =a2.

Bài tốn 10.24. Chứng minh không tồn giới hạn lim

n→∞(sinn)

n2 . Bài toán 10.25. Cho 0≤r < và hàm số:

F(r) = lim

n→∞

X

±1±r ±rn−1

.

Chứng minh ta có đẳng thức:

F

5−1

= 6−

5 F √ = 6.

Bài toán 10.26. Cho hàm số f :RR liên tục, tồn tại M > 0sao cho:

|f(x+y)−f(x)−f(y)|< M với mọi x, yR.

i) Chứng minh với số thực x đều tồn giới hạn hữu hạn:

g(x) = lim

nN,n→∞

f(nx)

n .

ii) Chứng minh hàm số g(x) liên tục điểm x=

iii) Chứng minh tồn giới hạn hữu hạn: lim

x→0

f(x)

x .

Bài toán 10.27. Chứng minh tồn hàm số liên tục toàn tập số thực R

nhưng không khả vi điểm đó.

Bài tốn 10.28. Giả sử bZ, b >1 Tập hợp SZ thoả mãn : (

0∈S

x 6≡y mod bx6=y, x, yS

Cho knSn thoả mãn

∞ P

n=1

kn

bn = Chứng minh kn = với mọin ≥1

Bài toán 10.29. Cho hàm số f :RR Chứng minh mệnh đề sau:

lim

x→0f(x) = limx→0

f(3x)−f(x)

x = =⇒xlim→0

f(x)

(121)

121

Bài toán 10.30. Cho đa thức P(x) = xn+a1xn−1 + +an ≥ ∀x Giả thiếta1 ≥

a2 an≥0 Gọi λ là nghiệm phức của P |λ| ≥1 Chứng minh tồn số tự nhiên

m λm = 1.

Bài toán 10.31. Cho dãy số thực {an} Chứng minh mệnh đề:

lim

n→∞an Xn

j=1

a2j

= =⇒ lim

n→∞3n·a

n= 1.

Bài toán 10.32. Cho hàm số f : RR khả vi liên tục cấp Chứng minh tồn tại

aR thoả mãn:

f(a)f0(a)f00(a)f000(a)≥0.

Bài toán 10.33. Chứng minh công thức giới hạn sau tìm ý nghĩa hình học của

chúng:        lim

n→∞n

1−cosπ

n = π 2 lim n→∞ π2

n2 −2

1−cos π

n

= 0.

Bài toán 10.34. Cho dãy số {an} thoả mãn a0 = 0, a1 = và:

an+1 = (

an+ sinan−1 nếu an> an−1

an+ cosan−1 nếu anan−1.

Tìm giới hạn lim

n→∞an.

Bài toán 10.35. Với song ánh p :Z+ →Z+ xây dựng hàm số f : (0,1) →R như sau:

f(0, a1a2, a3 ) = 0, ap(1)ap(2)ap(3) Hỏi với song ánh p nào thì f khả vi điểm đó.

Bài toán 10.36. Cho số thực α > n số thực dương b1, b2, , bn Chứng minh với

các số thực bất kỳ x1, x2, , xk ta có:

n X i=1 n X j=1

xixj

(bi+bj)α ≥0.

Bài toán 10.37. Tìm giá trị lớn của R −1

x3g(x)dx biết hàm g thoả mãn:          R −1

(g(x))2dx= 1

R −1

g(x)dx= R −1

xg(x)dx= R −1

(122)

Bài toán 10.38. Cho hàm số f liên tục trên R tuần hoàn chu kỳ Chứng minh với mọi số vơ tỷ α ta có:

lim n→∞ N N X n=1

f() = Z

f(t)dt.

Bài toán 10.39. Cho dãy số thực dương an>0 Chứng minh rằng:

∞ X N=1 N N X n=1 anp p−1

X∞

n=1

apn.

Bài toán 10.40. Xây dựng hàm số liên tục fn: [0,1]→R thoả mãn:

     R

fn(t)dt= với số tự nhiên n bất kỳ

lim

n→∞fn(x) = với số thực x

∈[0,1]

Bài tốn 10.41. Cho m số vơ tỷ a1, a2, , am có tính chất khơng tồn tại m số nguyên

t1, t2, , tm không đồng thời bằng t1a1+t2a2+ +tmam = Chứng minh với

0< s < t <1 tồn số tự nhiên n thoả mãn {nai} ∈(s, t) với mọi 1≤in.

Bài toán 10.42. Chứng minh tồn số thực dương k sao cho với mọi nN: n X i=1 sini i < k. Xét tổng riêng sau:

Sn+ =

X sini>0

sini i

Sn−=

X sini<0

sini i

. Chứng minh rằng:

lim

n→∞

Sn+

S

n

=−1.

Bài toán 10.43. Cho hàm số f :RR liên tục toàn R và thoả mãn:

lim

h→0+

f(x+h)−f(xh) 2h ≥0.

Chứng minh rằng f là hàm số tăng.

Bài toán 10.44. Chứng minh không tồn giới hạn lim

n→∞sin

n.

Bài toán 10.45. Hàm số Dirichlet xác định sau:

D(x) = (

1 x /Q

0 xQ.

Chứng minh không tồn dãy hàm liên tục {fn} sao cho

lim

(123)

123

Bài toán 10.46. Với số thực x ký hiệu E(x) là phần nguyên của x Cho số hũu tỷ a. Hàm số f :Z →[0,1] được định nghĩa sau:

f(m) =amE(am) với số nguyên mZ.

Chứng minh với mọi >0đoạn [0, ] chứa vô số phần tử của f(Z) (tập hợp toàn các giá trị của f) Từ suy có vơ số số hữu tỷ p

q thoả mãn:

ap q <

|q|.

Bài toán 10.47. Cho số thực a1, a2, , ak Với số tự nhiên n tuỳ ý đặt:

bn = k

Y

i=1

sin(nai).

Biết rằng lim

n→∞bn = Chứng minh tồn số i

ai

πZ.

Bài toán 10.48. Giả sử rằng {an} là dãy số thực bị chặn thoả mãn:

     n n P i=1

aib

1 ln(n)

n

P

i=1

aic.

Chứng minh rằng b=c.

Bài toán 10.49 (Bổ đề Dirichlet - 1842) Cho số thực α nN Chứng minh tồn tại số nguyên pZ và số tự nhiên qN thoả mãn:

i) αp

q < qn. ii) αp q <

q(n+ 1).

Từ suy với số vô tỷ α đều tồn vô số phân số p

q thoả mãn:

αp q <

q2.

Với điều kiện là q→+∞.

Lời Bình Có thể mở rộng bổ đề Dirichlet cho nhiều số Hãy chứng minh trường hợp

hai số sau Chứng minh với hai số thực α, β và số tự nhiên n tồn hai số nguyên

(124)

Bài toán 10.50. i) Xét dãy số {xn} xác định bởi:

(

x1 =t6=

xn+1(xn+t) =t+ với số tự nhiên n.

Tính giới hạn:

lim

n→∞xn. ii) Xét dãy số {yn} xác định bởi:

    

y1 =a6=−1

yn+1 =

3p2y2

n+ 2−2

2yn+

p 2y2

n+

với số tự nhiên n.

Tính giới hạn:

lim

(125)

Chương 11 Đại Số

Bài toán 11.1. Ký hiệu Nm là tập hợp tất số nguyên không bé số nguyên m cho

trước Tìm tất hàm f :NmNm thoả mãn:

f(x2 +f(y)) =y+ (f(x))2 ∀x, yNm.

Bài toán 11.2. Số thực c được gọi giá trị bội dãy số (xn) nếu tồn hai số k, l

thoả mãn xk =xl =c Với cặp số thực (a, b) ta lập dãy số:

U(a, b) : u0 =a, u1 =bun+1 =un+un−1 với số tự nhiên n Chứng minh tồn tại a, b nguyên cho dãy U(a, b) có hơn 2006 giá trị bội.

Bài toán 11.3. Xét dãy số {an} thoả mãn a1, a2, a3 là số nguyên và an+3 = an+1+an

với số tự nhiên n Chứng minh với số nguyên tố p ta có p là ước số:

an+3p+1−an+p+1−an+1. Bài toán 11.4. Với số tự nhiên n lớn hơn 1, xét đa thức:

Pn(x) =

[n−23 ] X

k=0

Cn3k+2.xk.

Tìm tất số nguyên a thoã mãn

n−1

|Pn(a3) với mọi n≥2

Bài toán 11.5. Xét dãy số (an) xác định sau:

(

a1 =a2 =a3 =a4 =

an.an−4 =an−1an−3 +a2n−2 với mọi n >4 Chứng minh rằng anZ với mọinN.

Bài toán 11.6. Với điều kiện xi >0 với mọii = 1, n Tính giá trị sau:

c= max{x1,

x1

+x2, ,

xn−1

, xn

}.

Giả sử có thêm điều kiện x1+x2+ +xn= Tính:

c= max{ x1

1 +x1

, x2

1 +x1+x2

, , xn

1 +x1+x2+ +xn }.

(126)

Bài toán 11.7. Cho dãy tăng số tự nhiên {ai} thoả mãn tính chất với hai tập con

I, J ∈ {1,2, , n} I 6=J thì ta có: X

iI

ai 6=

X

iJ

ai.

Tính giá trị lớn của:

n

X

i=1

ai

.

Bài tốn 11.8. Tìm tất hàm số f : (1,+∞)→R thoả mãn:

f(x)−f(y) = (yx)f(xy) với mọix, y >1.

Bài toán 11.9. Tồn hay khơng số thực ucó tính chất [un]−n là số phương với mọi số tự nhiên n.

Bài toán 11.10. Cho dãy số dương {an} thoả mãn:

(

a0 =a2005

ai = 2· √

ai−1ai+1 ∀1≤i≤2005.

Chứng minh rằng an=a2005−n với mọi 0≤n ≤2005

Bài toán 11.11. Cho dãy số {an} thoả mãn a1 =a2000 và với mọi nN:

xn+2 =

xnxn+1+ 5x4n

xnxn+1

.

Chứng minh rằng x2 6=x1999

Bài toán 11.12. Xét hàm số f(x) = 3(|x+|x−1| − |x+ 1|) và đặt xn+1 = f(xn) với mọi

n ≥0 Hỏi có số thực x0 thoả mãn x0 =x2007 và số x0, x1, , x2006 là đôi một phân biệt.

Bài tốn 11.13. Hỏi có tồn hay khơng đa thức P(x)bậc n mà đa thức hợp m lần của P

P( (P(x)) ) | {z }

mlầnP

nhận đủ nghiệm là 1,2, , mn.

Bài toán 11.14. Cho số nguyên n >1 n số thực a1, a2, , an Đặt:

    

S =

n

P

i=1

a2

i

P =

i<j (ai

aj)2

Chứng minh ta có bất đẳng thức:

S P

(127)

127

Bài toán 11.15. Cho số tự nhiên n >1 n số thực a1, a2, , an Chứng minh tồn tại

n số thực b1, b2, , bn thoả mãn tính chất:

    

aibiZ với mọi1≤in

P 1≤i<jn

(bibj)2 ≤

n2−1 12 .

Bài toán 11.16.Choa, b, c, x, y, zlà sáu số thực dương thoả mãn đẳng thứcax+by+cz =xyz. Chứng minh bất đẳng thức:

x+y+z >

q

4(a+b+c) +p8(ab+bc+ca).

Bài toán 11.17. Trên mặt phẳng cho n vecto sau đây v~1, ~v2, , ~v3 có:

n

X

i=1

|v~i|=h.

Chứng minh có k vecto v~i1, ~vi2, , ~vin trong số vecto {v~i1} sao cho:

| k

X

j=1

~ vij| ≥

h π.

Bài tốn 11.18. Giả sử số tự nhiên n có nhất 2ước số nguyên tố khác Chứng minh rằng tồn hoán vị (a1, a2, , an) của (1,2, , n) mà:

n

X

k=1

kcos2πak

n = 0.

Bài toán 11.19. Cho số nguyên dương p, thoả mãn p = 2n + là số nguyên tố và a

không chia hết cho p Chứng minh mệnh đề sau:

n

X

k=1

sin2πak

p

chẵn ⇐⇒ p|an−1.

Bài toán 11.20. Tìm điều kiện cần đủ số tự nhiên b1, b2, , bnsao cho ta có đẳng

thức sau với 1≤kn−1:

n X i=1 cos 2

n bi

= n X i=1 sin 2

n bi

.

Bài toán 11.21. Cho đa thức f(x) =xn+a1xn−1 + +an−1x+anR[x] Cho n số thực

phân biệt b1, b2, , bn thoả mãn n

P

i=1

=−a1 Chứng minh rằng:

n

X

i=1

f(bi)

Q

j6=i

(bibj)

(128)

Bài toán 11.22. Chứng minh bất đẳng thức:

n

X

k=1

(1 +x2

k) n/2 Q

j6=k

(bkbj) ≥n.

Bài toán 11.23. Cho số tự nhiên n ui = cos

2i−1

2n+ ·π với 1≤in Chứng minh rằng:

2n =

n

X

i=1

1 p

1−u2

i

Q

j6=i,1≤jn+1

|uibj|

.

Từ suy định lý M arkov: giả sử đa thức hệ số thựcf(x) =xn+a

1xn−1+ +an−1x+an

thoả mãn:

1−x2 · |f(x)| ≤1 ∀x∈[−1,1]. Chứng minh rằng:

|a0| ≤2n.

Bài toán 11.24. Ký hiệu phép toánnhư sau Với hai số thực dương x, y:

xy= x+y +xy.

Tính giá trị biểu thức 1∗2∗3∗ ∗2006 với thứ tự phép toán tuỳ ý. Bài toán 11.25. Chứng minh tồn phân hoạch:

N ={[]|nN} ∪ {[]|nN}.

Với hai số vô tỷ dương α, β thoả mãn

α +

1

β = Tuy nhiên không tồn ba số vô tỷ dương α, β, λ sao cho ta có phân hoạch:

N ={[]|nN} ∪ {[]|nN} ∪ {[]|nN}.

Bài toán 11.26. Trong bảng số m.n có tính chất tổng hàng hay cột số nguyên. Chứng minh thay số bảng hai số nguyên gần sao cho tổng hàng cột khơng đổi.

Bài tốn 11.27. Cho tập n số thực tuỳ ý{an} Chứng minh tồn tập con TA sao

cho tổng số trong T là số thực sai khác với số nguyên gần khơng q

n+ 1. Bài toán 11.28. Cho n số thực bất kỳ {an} Chứng minh tìm số {bi} bi

là hai số nguyên gần ai nhất với k bất kỳ:

k X j=1

aij −

k

X

j=1

bij

(129)

129

Bài toán 11.29. Cho số thực dương a, b, c, d thoả mãn a > b > c > d > e (với e là số của logarith tự nhiên) Chứng minh rằng:

aeb +bec+ced +dea < bea+ceb +dec +aed.

Bài toán 11.30. Với mội số nguyên dương n tìm số thực dương q = q(n) tốt cho với dãy n số thực x1, x2, , xn ta có bất đẳng thức:

(1) n X i=1 Xi j=1 xi

q· n

X

i=1

x2i

(2) n X i=1 Xi j=1 xi

q· n

X

i=1

x2i.

Bài toán 11.31. Xét dãy số {an} xác định sau:

  

a1 =

an+1 =

3an

2

với số tự nhiên n.

Chứng minh rằng

(1) Trong dãy số có vô hạn số chẵn vô hạn số lẻ. (2) Tồn số thực α sao cho an+1 =

α n + (3) Số 0, a1a2, là số vô tỷ hay hữu tỷ.

Ngồi có tồn hay khơng số thực α sao cho an+1 = 2α n + 1?

Bài tốn 11.32. Chứng minh khơng tồn hàm số f :RR mà:

f(f(x)) =x2−3x−2 với số thực x.

Bài toán 11.33. Chứng minh đẳng thức sau đối với n tự nhiên tuỳ ý: + r n − 12 = s n+ 2+ r n − 12 .

Bài toán 11.34. Chứng minh đẳng thức sau với số tự nhiên n:

n X p=1 p X q=1

− +

p

8q+ (2p−1)2

=−n(n+ 1)(n+ 2)

3 .

Bài toán 11.35. Giả sử P Q là đa thức thoả mãn P3 6≡Q2 Chứng minh rằng:

deg(P3−Q2)≥deg(P) + 2. Nếu đặt F =P3−Q4 thì ta có:

deg(F)≥

(130)

Bài tốn 11.36. Tìm tất hàm số f :RR thoả mãn:

f(xf(y)) = 4f(x)−f(y)−3x với mọi x, yR.

Bài toán 11.37. Cho số thực a, b, c thoả mãn (b−1)2−4ac= Xét dãy đa thức: (

f(1, x) =ax2+bx+c

f(n+ 1, x) =f(1, f(n, x)) với số tự nhiên n.

Tìm số nghiệm thực xR của phương trình f(n, x) =

Bài tốn 11.38. Tìm tất số thực (x1, x2, , xn) thoả mãn:

(x1+x2+ +xk)(xk + +xn) = với 1≤kn.

Bài toán 11.39. Cho a, b số tự nhiên khác Chứng minh mệnh đề:

bn−1|an−1 ∀nN =⇒ ∃kN a=bk.

Bài toán 11.40. Giả sử f :RR là hàm số thoả mãn với số thực dương x tồn tại đa thức Pc(x) có tính chất:

|f(x)−Pc(x)| ≤cx2006 ∀xR.

Chứng minh rằng f là đa thức.

Bài tốn 11.41. Tìm tất đa thức P hệ số nguyên cho đa thức:

Q(x) = (x2+ 6x+ 10)(P(x))2−1 = (R(x))2 là bình phương đa thức hệ số nguyên.

Bài toán 11.42. Cho số tự nhiên m > và số p1, p2, , pn là tất số nguyên tố

không vượt quá m Chứng minh rằng:

n

X

k=1

1

pk

+

p2

k

ln(ln(n)).

Bài tốn 11.43. Tìm tất số thực a, b sao cho với mọi n xn là nghiệm phương

trình cosx

x =n ta ln có cosaxn+ cosbxn ≥2−x

2

n.

Bài tốn 11.44. Tìm tất số thựck sao cho tồn hàm số f khả vi toànR thoả mãn với số thực x thì: (

f(x)≤

(f(x))2+ (f0(x))2 =k.

Bài tốn 11.45. Tìm tất hàm số f : [0,1]→[0,1] thoả mãn:

(131)

131

Bài tốn 11.46. Tìm tất toàn ánh f :RR thoả mãn:

f(f(xy)) =f(x)−f(y) với mọix, yR.

Bài toán 11.47. Cho x là số thực thoả mãn [nx2] = [x[xn]] + 1 với số tự nhiên n bất kỳ Chứng minh rằng x= +

5 .

Chứng minh nếu x3 =x2+ 1 x >0thì tồn số {Cn} nhận giá trị0,1,2 mà:

[nx] + [nx2] + [nx3] = [x[n4]] +Cn.

Bài toán 11.48. Một học sinh chơi với hệ số phương trình bậc hai sau Lấy hai số p, q bất kỳ, xét phương trình x2+px+q = Nếu phương trình có hai nghiệm p1, q1 thi lại xét phương trình x2+p

1x+q1 Hỏi học sinh chơi q lượt hay khơng (khơng tính phương trình đầu tiên).

Bài tốn 11.49. Cho tập hợp hữu hạn Acó khơng hơn 6phần tử cho nếu a, b, c, d, e, f

phần tử phân biệt của A thì ab+cd+ef cũng thuộc vào A Tìm giá trị lớn số các phần tử của A.

Bài toán 11.50. Trên mặt phẳng toạ độ cho 101 đường thẳng đánh dấu tất giao điểm chúng Hỏi xảy hay khơng tình đường thẳng có đúng 50 điểm đánh dấu có hoành độ dương và 50 điểm đánh dấu khác có hồnh độ âm.

Ngày đăng: 28/04/2021, 10:22

TỪ KHÓA LIÊN QUAN

w